You are on page 1of 132

Polos Olmpicos de Treinamento

Curso de Teoria dos Nmeros - Nvel 2

Aula

Samuel Barbosa Feitosa

Divisibilidade I
Teorema 1. (Algoritmo da Divis
ao) Para quaisquer inteiros positivos a e b, existe um u
nico
par (q, r) de inteiros n
ao negativos tais que b = aq + r e r < a. Os n
umeros q e r s
ao
chamados de quociente e resto, respectivamente, da divis
ao de b por a.
Exemplo 2. Encontre um n
umero natural N que, ao ser dividido por 10, deixa resto 9, ao
ser dividido por 9 deixa resto 8, e ao ser dividido por 8 deixa resto 7.
O que acontece ao somarmos 1 ao nosso n
umero? Ele passa a deixar resto 0 na divisao por
10, 9 e 8. Assim, um possvel valor para N e 10 9 8 1.
Exemplo 3. a) Verifique que an 1 = (a 1)(an1 + an2 + . . . + a + 1)
b) Calcule o resto da divis
ao de 42012 por 3.
Para o item a), usando a distributividade e efetuando os devidos cancelamentos no lado
direito, podemos escrever:
an + an1 + . . . + a2 + a an1 an2 . . . a 1 = an 1.
Para o item b), veja que 3 = 41 e assim e natural substituir os valores dados na express
ao
do primeiro item:
42012 1 = 3(42011 + . . . + 4 + 1).
Isso significa que q = (42011 + . . . + 4 + 1) e que r = 1.
Observac
ao 4. O teorema anterior admite um enunciado mais geral: Para quaisquer inteiros a e b, com a 6= 0, existe um u
nico par de inteiros (q, r) tais que b = aq + r, 0 r < |a|.
Por exemplo, o resto da divis
ao de 7 por 3 e 2 e o quociente e 3.
Iremos agora estudar propriedades a respeito das operacoes com restos.
Teorema 5. (Teorema dos Restos) Se b1 e b2 deixam restos r1 e r2 na divis
ao por a,respectivamente,
ent
ao:

POT 2012 - Teoria dos N


umeros - Nvel 2 - Aula 1 - Samuel Feitosa

b1 + b2 deixa o mesmo resto que r1 + r2 na divis


ao por a
b1 b2 deixa o mesmo resto que r1 r2 na divis
ao por a.
Demonstraca
o. Por hip
otese, existem q1 , q2 e q tais que: b1 = aq1 + r1 , b2 = aq2 + r2 e
r1 + r2 = aq + r, logo:
b1 + b2 = a(q1 + q2 + q) + r.
Como 0 < r < |a|, b1 + b2 deixa resto r quando dividido por a. A demonstracao para o
produto e deixada ao cargo do leitor.
Observac
ao 6. Em alguns casos, e prefervel que o professor faca uma demonstraca
o do
resultado anterior para a = 3 ou a = 5 apenas com o intuito de deixar os alunos mais
prefervel que mais tempo seja gasto resolvendo
confort
aveis a respeito do resultado. E
exemplos e problemas. Na seca
o de congruencias, os alunos ter
ao um contato mais apropriado com o enunciado anterior.
Exemplo 7. Qual o resto que o n
umero 1002 1003 1004 deixa quando dividido por 7?
Como 1002 deixa resto 1 por 7, o n
umero acima deixa o mesmo resto que 1 2 3 = 6 por 7.
Exemplo 8. Qual o resto que o n
umero 45000 deixa quando dividido por 3?
Como 4 deixa resto 1 por 3, 45000 deixa o mesmo resto que 1| 1 {z. . . 1} = 1 por 3.
5000

Exemplo 9. Qual o resto que o n


umero 22k+1 deixa quando dividido por 3?

Note que 20 deixa resto 1 por 3, 21 deixa resto 2 por 3, 22 deixa resto 1 por 3, 23 deixa
resto 2 por 3, 24 deixa resto 1 por 3. Precebeu alguma coisa? Como 100 e par, o resto
devera ser 1. Como 22 deixa resto 1, entao 22k = 2| 2 22 {z
. . . 2}2 deixa o mesmo resto que
k

1| 1 {z. . . 1} = 1 e 22k+1 = 22k 2 deixa o mesmo resto que 1 2 = 2 por 3.


k

Exemplo 10. Qual o resto de n3 + 2n na divis


ao por 3?

Se o resto de n por 3 e r, o resto de n3 + 2n e o mesmo de r3 + 2r. Para r = 0, esse


resto seria 0. Para r = 1, seria o mesmo resto de 3 que e 0. Finalmente, para r = 2, o
resto seria o mesmo de 8 + 4 = 12 que tambem e 0. Assim, nao importa qual o resto de n
por 3, o n
umero n3 + 2n sempre deixara resto 0. Uma ideia importante nessa solucao foi
divid-la em casos. Tambem poderamos ter resolvido esse exemplo apelando para alguma
fatoracao:
n3 + 2n = n3 n + 3n = n(n2 1) + 3n = n(n 1)(n + 1) + 3n.
Como n 1, n e n + 1 s
ao consecutivos, um deles e m
ultiplo de 3. Assim, o u
ltimo termo da
igualdade anterior e a soma de dois m
ultiplos de 3 e consequentemente o resto procurado
e 0.

POT 2012 - Teoria dos N


umeros - Nvel 2 - Aula 1 - Samuel Feitosa

Observac
ao 11. Fatoraco
es podem ser muito u
teis para encontrarmos os valores explcitos
de q e r.
Exemplo 12. Prove que, para cada n natural,
(n + 1)(n + 2) . . . (2n)
e divisvel por 2n .
Veja que

1 2 2n
.
1 2n
Para cada n
umero natural k no produto escrito no denominador, temos uma aparicao de
2k no produto escrito no numerador. Basta efetuarmos os cancelamentos obtendo:
(n + 1)(n + 2) . . . (2n) =

(n + 1)(n + 2) . . . (2n) = 2n 1 3 (2n 1).


Exemplo 13. (Olimpada de Leningrado 1991) Cada um dos naturais a, b, c e d e divisvel
por ab cd, que tambem e um n
umero natural. Prove que ab cd = 1.
Se chamarmos p = ab cd, teremos a = px, b = py, c = pz e d = pt onde x, y, z e t s
ao
inteiros. Assim, p = p2 (xy zt). Consequentemente 1 = p(xy zt) e conclumos que p = 1,
pois p e natural.
Exemplo 14. A soma digital D(n) de um inteiro positivo n e definida recursivamente como
segue:

n
se 1 n 9,
D(n) =
D(a0 + a1 + . . . + am ) se n > 9,
onde a0 , a1 , . . . , am s
ao todos os dgitos da express
ao decimal de n na base 10, i.e.,
n = am 10m + am1 10m1 + . . . + a1 10 + a0
Por exemplo, D(989) = D(26) = D(8) = 8. Prove que: D((1234)n) = D(n), para n =
1, 2, 3 . . .
Como 10n 1n = (10 1)(10n1 + 10n2 + . . . + 1), podemos concluir que 10n sempre deixa
resto 1 na divisao por 9. Assim, n = am 10m + am1 10m1 + . . . + a1 10 + a0 , deixa o mesmo
resto que am + am1 + . . . + a0 na divisao por 9. Desse modo, D(n) nada mais e do que
o resto na divisao por 9 do n
umero n. Como 1234 deixa resto 1 por 9, o n
umero (1234)n
deixa o mesmo resto que 1 n por 9, ou seja, D((1234)n) = D(n).
Observac
ao 15. O exemplo anterior contem o criterio de divisibilidade por 9, i.e., n deixa
o mesmo resto que D(n) na divis
ao por 9. O criterio de divisibilidade por 3 e an
alogo pois
10n tambem sempre deixa resto 1 por 3.
Exemplo 16. Encontre todos os pares de inteiros positivos a e b tais que 79 = ab + 2a + 3b.

POT 2012 - Teoria dos N


umeros - Nvel 2 - Aula 1 - Samuel Feitosa

Fatoremos a expressao anterior. Somando 6 aos dois lados da equacao, obtemos:


85 = 6 + ab + 2a + 3b
= (3 + a)(2 + b)
Assim, (3 + a) e (2 + b) s
ao divisores positivos de 85 maiores que 1. Os u
nicos divisores
positivos de 85 s
ao 1, 5, 19, 85. Logo, os possveis pares de valores para (3 + a, 2 + b) s
ao
(5, 19) ou (19, 5) que produzem as solucoes (a, b) = (2, 17) e (16, 3).
2n 2
22 1 2
e um inteiro, ent
ao
n
2n 1
n

Problema 17. (Olimpada Russa) Prove que se


tambem e um inteiro.
Se k =

2n 2
, entao
n
22 1 2
2n 1
n

2(22 2 1)
=
2n 1
 nk

2 1
= 2
2n 1
n

= 2

(2n 1)(2n(k1) + 2n(k2) + . . . + 2n + 1)


2n 1

= 2(2n(k1) + 2n(k2) + . . . + 2n + 1),


e um n
umero inteiro.

Problemas Propostos

Problema 18. Encontre os inteiros que, na divis


ao por 7, deixam um quociente igual ao
resto.
Problema 19. Determinar os n
umeros que divididos por 17 d
ao um resto igual ao quadrado
do quociente correspondente.
Problema 20. (OCM 1985) Encontre o quociente da divis
ao de a128 b128 por
(a64 + b64 )(a32 + b32 )(a16 + b16 )(a8 + b8 )(a4 + b4 )(a2 + b2 )(a + b)
Problema 21. (OCM 1994) Seja A = 777 . . . 77 um n
umero onde o dgito 7aparece 1001
vezes. Determinar o quociente e o resto da divis
ao de A por 1001.
Problema 22. Encontre um inteiro que deixa resto 4 na divis
ao por 5 e resto 7 na divis
ao
por 13
Problema 23. Encontre o menor inteiro que, dividido por 29 deixa resto 5, e dividido por
31 d
a resto 28.
4

POT 2012 - Teoria dos N


umeros - Nvel 2 - Aula 1 - Samuel Feitosa

Problema 24. Prove que, para todo inteiro positivo n o n


umero n5 5n3 + 4n e divisvel
por 120.
Problema 25. (Fatoraco
es Importantes)
a) Seja S = 1 + z + z 2 + z 3 + . . . + z n1 . Veja que S + z n = 1 + zS ent
ao S(z 1) = z n 1.
Conclua que, para quaisquer x e y vale:
xn y n = (x y)(xn1 + xn2 y + xn3 y 2 + . . . + x2 y n3 + xy n2 + y n1 )
b) Mostre que se n e mpar vale:
xn + y n = (x + y)(xn1 xn2 y + xn3 y 2 . . . + x2 y n3 xy n2 + y n1 )
Problema 26. Prove que, o n
umero 199 + 299 + 399 + 499 + 599 e m
ultiplo de 5.
Problema 27. Mostre que o n
umero 1n + 8n 3n 6n e multiplo de 10 para todo natural
n.
Problema 28. Encontre o resto da divis
ao 3710 1 por 11.
Problema 29. Prove que 22225555 + 55552222 e divisvel por 7.
Problema 30. Encontre o u
ltimo dgito do n
umero 19891989 .
Problema 31. Mostre que se n divide a ent
ao 2n 1 divide 2a 1.
Problema 32. (Cone Sul 1996) Provar que o n
umero
1995 19971996 1996 19971995 + 1
19962
e um inteiro.
Problema 33. Mostre que para n mpar, n divide 1n + 2n + . . . + (n 1)n
Problema 34. Existe um natural n tal que nn + (n + 1)n e divisvel por 2011?
Problema 35. Quantos n
umeros inteiros positivos n existem tais que n + 3 divide n2 + 7?
Problema 36. Encontre o n
umero de inteiros n tais que
1. 1000 < n < 8000.
2. nn+1 + (n + 1)n e divisvel por 3.
Problema 37. Sejam m e n naturais tais que mn + 1 e m
ultiplo de 24, mostre que m + n
tambem e m
ultiplo de 24.

POT 2012 - Teoria dos N


umeros - Nvel 2 - Aula 1 - Samuel Feitosa

Problema 38. (Irlanda 1997) Encontre todos os pares de inteiros (x, y) tais que 1+1996x+
1998y = xy.

Dicas e Solu
c
oes

18. Os n
umeros s
ao {0, 8, 16, 24, . . . , 8 7}.
18. Escreva n = 17q + q 2 e note que 0 q 2 < 17. Assim, q = 0, 1, 2, 3, 4.
19. Use a diferenca de quadrados sucessivas vezes para obter (a b) como quociente.
21. O n
umero do problema e igual a
100

10999 +1
103 +1

7(101001 1)
.
9

Alem disso,

10999 +1
103 +1

e inteiro e

101001 1
103 +1

100
.
103 +1

22. Os n
umeros que satisfazem essa propriedade s
ao os n
umeros da forma 65k + 59.
24. Basta mostrar que n5 5n3 + 4n e m
ultiplo de 3, 8 e 5. Na divisao por 5, temos
quatro restos possveis: {0, 1, 2, 3, 4}. Assim, o n
umero n5 5n3 + 4n possui o mesmo
resto na divisao por 5 que um dos cinco n
umeros: {05 5 03 + 40, 15 5 13 + 4, 25
5 23 + 8, 35 5 33 + 12, 45 5 43 + 16}. Como todos esses n
umeros s
ao m
ultiplos
de 5, segue que n5 5n3 + 4n e m
ultiplo de 5 para todo n inteiro. O procedimento
com 3 e 8 e semelhante.
25. Para o item a), troque z por xy . Para o item b), substitua y por y no item anterior.
26. Pelo problema anterior, como 99 e mpar temos: 199 + 499 = (1 + 4)(198 + 197 4 +
. . . + 1 497 + 498 ). Da, segue que 199 + 499 e m
ultiplo de 5. Analogamente podemos
99
99
mostrar que 2 + 3 e m
ultiplo de 5.
27. O n
umero em quest
ao e m
utiplo de 2 pois e a soma de dois mpares e dois pares.
Para ver que tambem e m
ultiplo de 5, basta notar que 5 divide 1n 6n e 8n 3n .
Isso pode ser facilmente mostrado usando a fatoracao do exerccio 25.
31. Se a = nk, temos (2n 1)(2n(k1) + 2n(k2) + . . . + 2n + 1) = 2nk 1.
32. Veja que 199519971996 199619971995 +1 = 1995(19971996 1)1996(19971995 1).
Pela fatoracao de xn y n ,
1996 (19971995 1)
= (19971994 + 19971993 + . . . + 1),
19962
e inteiro. Alem disso, pela mesma fatoracao,


1997 1 1996
19971995 1 19971994 1
1995 (19971996 1)
=
1995
+
+
.
.
.
+
+
,
19962
1996
1996
1996
1996
e uma soma de n
umeros inteiros.
6

33. Como n e impar,


(n i)n + in = ((n i) + i)((n i)n1 (n i)n2 i + . . . (n i)in2 + in1 ).
34. Faca n = 1005 e use a fatoracao de xn + y n .
37. Fatore a expressao como:
(x 1998)(y 1996) = xy 1998y 1996x + 1998 1996 = 19972 .
Os divisores de 19972 s
ao {1, 1997, 19972 }. Resolvendo os sistemas correspondentes `
a essas possibilidades, temos: (x, y) = (1999, 19972 + 1996), (1997, 19972 +
1996), (3995, 3993), (1, 1), (19972 + 1998, 1997), (19972 + 1998, 1995).

Refer
encias
[1] F. E. Brochero Martinez, C. G. Moreira, N. C. Saldanha, E. Tengan - Teoria dos
N
umeros um passeio com primos e outros n
umeros familiares pelo mundo inteiro,
Projeto Euclides, IMPA, 2010.
[2] E. Carneiro, O. Campos and F. Paiva, Olimpadas Cearenses de Matem
atica 1981-2005
(Nveis J
unior e Senior), Ed. Realce, 2005.
[3] S. B. Feitosa, B. Holanda, Y. Lima and C. T. Magalhaes, Treinamento Cone Sul 2008.
Fortaleza, Ed. Realce, 2010.
[4] D. Fomin, A. Kirichenko, Leningrad Mathematical Olympiads 1987-1991, MathPro
Press, Westford, MA, 1994.
[5] D. Fomin, S. Genkin and I. Itenberg, Mathematical Circles, Mathematical Words, Vol.
7, American Mathematical Society, Boston, MA, 1966.
[6] I. Niven, H. S. Zuckerman, and H. L. Montgomery, An Introduction to the Theory of
Numbers.

Polos Olmpicos de Treinamento


Curso de Teoria dos Nmeros - Nvel 2

Aula

Prof. Samuel Feitosa

Divisibilidade II

Definic
ao 1. Dados dois inteiros a e b, com a 6= 0, dizemos que a divide b ou que a e
um divisor de b ou ainda que b e um m
ultiplo de a e escrevemos a | b se o r obtido pelo
algoritmo de divis
ao aplicado `
a a e b e 0, ou seja, se b = aq para algum inteiro q.
Lema 2. Sejam a, b, c, d inteiros. Temos
i) (d divide) Se d | a e d | b, ent
ao d | ax + by para quaisquer x e y inteiros.
ii) (Limitaca
o) Se d | a, ent
ao a = 0 ou |d| |a|.
iii) (Transitividade) Se a | b e b | c, ent
ao a | c.
Em particular, segue da propriedade i) que d | a + b e d | a b.
Exemplo 3. (Olimpada de Maio 2006) Encontre todos os naturais a e b tais que a|b + 1 e
b|a + 1.
Pela propriedade da Limitacao, temos a b + 1 e b a + 1. Da, a 1 b a + 1.
Vejamos os casos:
(i) a = b. Como a|b + 1 e a | b(pois b = a) temos que a | [(b + 1) b] = 1. Assim, a = 1
Nesse caso, s
o temos a solucao (a, b) = (1, 1)
(ii) a = b + 1. Como b|a + 1 e b|a 1(pois b = a 1) temos que b|[(a + 1) (a 1)] = 2.
Assim, b = 1 ou b = 2 e nesse caso, s
o temos as solucoes (3, 2) e (2, 1).
(iii) a = b 1. Esse caso e an
alogo ao anterior e as solucoes para (a, b) s
ao (1, 2) e (2, 3).
Exemplo 4. (Criterio de Divisibilidade por 7) Existem alguns metodos pr
aticos para decidirmos se um n
umero e m
ultiplo de outro. Certamente o leitor j
a deve ter se deparado com
algum criterio de divisibilidade. Existe um criterio por 7 bastante popular: Para saber se
um inteiro e multiplo de 7, basta apagar seu u
ltimo dgito, multiplic
a-lo por 2 e o subtrair
do n
umero que restou. Se o resultado e m
ultiplo de 7, ent
ao o n
umero original tambem e
m
ultiplo de 7.

POT 2012 - Teoria dos N


umeros - Nvel 2 - Aula 1 - Samuel Feitosa

Podemos aplicar esse algoritmo sucessivas vezes ate que o resultado obtido seja facilmente verificavel como um m
ultiplo de 7. Por exemplo, para o n
umero 561421 podemos
escrever:
56142 2 = 56140
5614 0 = 5614
561 8 = 553
55 6 = 49

Como 49 e m
ultiplo de 7, nosso n
umero original tambem e. Por que esse processo funciona?
Se o nosso n
umero original esta escrito na forma 10a + b, entao o n
umero obtido ap
os a
operacao descrita e a 2b. Basta mostrarmos que se 7 | a 2b, entao 7 | 10a + b. Se
7 | a 2b, pela propriedade (i) do lema, conclumos que 7 | 10a 20b. Como 7 | 21b,
tambem temos que 7 | [(10a 20b) + 21b] = 10a + b.
Exemplo 5. Mostre que se 7 | 3a + 2b ent
ao 7 | 4a 2b.
Veja que 7 | 7a e 7 | 3a + 2b, entao 7 | [7a (3a + 2b)] = 4a 2b. Na pratica, o que
fizemos foi multiplicar o n
umero 3a + 2b por algum inteiro para posteriormente subtramos
um m
ultiplo de 7 conveniente e obtermos o n
umero 4a 2b. Existem outras formas de
fazermos isso. Observe os n
umeros 3 0, 3 1, 3 2, 3 3, 3 4, 3 5, 3 6. O n
umero 3 6 deixa
o mesmo resto que 4 por 7, pois 3 6 = 7 2 + 4. Como 7|3a + 2b podemos concluir que
7|(18a + 12b) e consequentemente 7 | [18a + 12b 14a)] = 4a + 12b. Mas 7 | 14b, entao
7 | [4a + 12b 14b] = 4a 2b.
Para o proximo exemplo, o leitor precisara lembrar dos criterios de divisibilidade por 9
e 3 vistos na aula passada.
Exemplo 6. Usando os dgitos 1, 2, 3, 4, 5, 6, 7, construmos v
arios n
umeros de sete dgitos
distintos. Existem dois deles, distintos, tais que um divide o outro?
N
ao. Suponha, por absurdo, que m < n sejam dois desses n
umeros, com m | n. Claramente
m | n m e 9 | n m, pois n e m possuem a mesma soma dos dgitos e consequentemente
possuem o mesmo resto na divisao por 9. Por outro lado, sabemos a soma dos dgitos de
m: 1 + 2 + + 7 = 3 9 + 1. Da, m nao possui fator 9 e podemos garantir que 9m | n m.
Mas entao 9m n m 10m n n tem pelo menos oito dgitos, uma contradicao.
Exemplo 7. (Leningrado 1989) Seja A um n
umero natural maior que 1, e seja B um
n
umero natural que e um divisor de A2 + 1. Prove que se B A > 0, ent
ao B A > A.
Seja B A = q. Assim, A + q | A2 + 1. Como (A q)(A + q) = A2 q 2 e divisvel por
A + q, podemos concluir que A + q | [(A2 + 1) (A2 q 2 )] = q 2 + 1. Pela propriedade de
limitacao, A + q q 2 + 1. Nessa desigualdade, nao podemos
ter q = 1 pois A > 1. Usando

2
2
entao que q > 1, temos A q q + 1 < q , ou seja, A < q.
Problema 8. (AIME 1986) Qual e o maior inteiro n para o qual n3 + 100 e divisvel por
n + 10?
2

POT 2012 - Teoria dos N


umeros - Nvel 2 - Aula 1 - Samuel Feitosa

Para achar explicitamente o quociente de n3 + 100 por n + 10 podemos fazer uso de alguma
fatoracao. Utilizaremos a soma dos cubos n3 + 103 = (n + 10)(n2 10n + 100). Como,
n3 + 100 = (n + 10)(n2 10n + 100) 900,
podemos concluir que o n
umero 900 deve ser m
ultiplo de n + 10. O maior inteiro n para o
qual n + 10 divide 900 e 890. Veja que se n = 890, o quociente da divisao de n3 + 100 por
n + 10 e n2 10n + 100 1 = 8902 10 890 + 99.
Exemplo 9. (Extrado de [1]) Encontre todos os inteiros positivos n tais que 2n2 + 1 |
n3 + 9n 17.
Utilizando o 2n2 + 1 divide para reduzir o grau de n3 + 9n 17, temos que
(
2n2 + 1 | n3 + 9n 17
2n2 + 1 | 2n2 + 1
= 2n2 + 1 | (n3 + 9n 17) 2 + (2n2 + 1) (n)

2n2 + 1 | 17n 34

Como o grau de 17n 34 e menor do que o de 2n2 + 1, podemos utilizar a limitacao


para obter uma lista finita de candidatos a n. Temos 17n 34 = 0 n = 2 ou
|2n2 + 1| |17n 34| n = 1, 4 ou 5. Destes candidatos, apenas n = 2 e n = 5 s
ao
solucoes.
Exemplo 10. (Leningrado 1990) Sejam a e b n
umeros naturais tais que b2 + ba + 1 divide
2
a + ab + 1. Prove que a = b.
Pela propriedade de limitacao, b2 +ba+1 a2 +ab+1 e da b a. Alem disso, b2 +ab+1 >
a b. A igualdade b(a2 + ab + 1) a(b2 + ba + 1) = b a implica que a b e divisvel por
b2 + ba + 1. Se a b 6= 0, entao b2 + ab + 1 a b. Mas isso e um absurdo, logo a b = 0.

Problemas Propostos

Problema 11. Mostre que se 3 | a + 7b ent


ao 3 | a + b.
Problema 12. Mostre que se 7 | a + 3b ent
ao | 13a + 11b
Problema 13. Mostre que se 19 | 3x + 7y ent
ao 19 | 43x + 75y
Problema 14. Mostre que se 17 | 3a + 2b ent
ao 17 | 10a + b
Problema 15. Encontre todos os inteiros positivos n tais que n + 2009 divide n2 + 2009 e
n + 2010 divide n2 + 2010.
Problema 16. Seja n > 1 e k um inteiro positivo qualquer. Prove que (n 1)2 |(nk 1) se,
e somente se , (n 1)|k.
3

POT 2012 - Teoria dos N


umeros - Nvel 2 - Aula 1 - Samuel Feitosa

Problema 17. (OBM 2005) Prove que a soma 1k + 2k + . . . + nk , onde n e um inteiro e k


e mpar, e divisvel por 1 + 2 + . . . + n.
Problema 18. O n
umero de seis dgitos X = abcdef satisfaz a propriedade de que abcdef
e divisvel por 7. Prove que X tambem e divisvel por 7.
Problema 19. (Bielor
ussia 1996) Inteiros m e n, satisfazem a igualdade
(m n)2 =

4mn
.
m+n1

a) Prove que m + n e um quadrado perfeito.


b) Encontre todos os pares (m, n) satisfazendo a equaca
o acima.
Problema 20. (Olimpada de Leningrado) Os n
umeros naturais a,b e c tem a propriedade
3
3
3
que a e divisvel por b, b e divisvel por c e c e divisvel por a. Prove que (a + b + c)13 e
divisvel por abc.
possvel encontrar duas potencias de 2, distintas e com o
Problema 21. (OBM 2000) E
mesmo n
umero de algarismos, tais que uma possa ser obtida atraves de uma reordenaca
o
dos dgitos da outra? (Dica: Lembre-se do criterio de divisibilidade por 9)
Problema 22. (IMO 1998) Determine todos os pares de inteiros positivos (x, y) tais que
xy 2 + y + 7 divide x2 y + x + y.

Dicas e Solu
c
oes

11. Como 3 | 6b, segue que 3 | [(a + 7b) 6b] = a + b.


12. Como 7 | a + 3b, segue que 7 | 13a + 39b = (13a + 11b) + 28b. Mas 7 | 28b, portanto
7 | [(13a + 11b) + 28b 28b] = 13a + 11b.
13. Como 19 | 3x + 7y, segue que 19 | 27(3x + 7y) = (43x + 75y) + (38x + 114y). Mas
19 | 19(2x + 6y), portanto 19 | [(43x + 75y) + (38x + 114y) 19(2x + 6y)] = 43x + 75y.
14. Como 17 | 3a + 2b, segue que 17 | 27a + 18b = (10a + b) + 17(a + b).
16. Veja que
nk 1
=
(n 1)2

nk1 1 nk2 1
n1
k
+
+ ... +
+
n1
n1
n1 n1

1
sempre s
ao inteiros, o n
umero do lado esquerdo da equacao
Como os n
umeros nn1
k
sera inteiro se, e somente se, o n
umero n1 for inteiro.

POT 2012 - Teoria dos N


umeros - Nvel 2 - Aula 1 - Samuel Feitosa

17. Comece dividindo o problema quando em dois casos: n e par ou n e mpar. Sabemos
. Para n mpar, basta mostrar que o n
umero em questao
que 1 + 2 + . . . + n = n(n+1)
2
e divisvel por n e n+1
.
O
pr
o
ximo
passo

e
lembrar
do
problema
33
da aula 1. Pela
2
n
n
k
k
fatoracao de x + y , temos que i + (n i) e divisvel por n. Faca outros tipos de
pares para mostrar a divisibilidade por n2 . O caso quando n e par e an
alogo.
18. Veja que X = 103 abc + def = 1001abc (abc def ). Como 1001 e multiplo de 7,
conclumos que X e a soma de dois m
ultiplos de 7.
19. Somando 4mn em ambos os lados, obtemos:
4mn
+ 4mn
m+n1
4mn(m + n)
=

m+n1
4mn
(m + n) =
m+n1
= (m n)2 .

(m + n)2 =

Assim, m + n e o quadrado de um inteiro. Se m n = t, entao m + n = t2 e


2
2
facil verificar que para qualquer t inteiro esse par e solucao
(m, n) = ( t 2+t , t 2t ). E
do problema.
20. Analise a expans
ao pelo bin
omio de Newton.
21. N
ao. Suponha, por absurdo, que existam duas potencias de 2, 2m < 2n , satisfazendo
o enunciado. Como 2n e um m
ultiplo de 2m , podemos ter: 2n = 22m , 42m , 82m , . . ..
n
Alem disso, como ambos possuem a mesma quantidade de dgitos, temos 1 < 22m <
10. Assim, as u
nicas possibilidade s
ao 2n = 2 2m , 4 2m , 8 2m . Pelo criterio de
divisibilidade por 9, como 2m e 2n possuem os mesmos dgitos, podemos concluir
que 2n 2m e um m
ultiplo de 9. Entretanto, nenhuma das possibilidade anteriores
satisfaz essa condicao e chegamos em um absurdo.
22. Comecaremos usando a ideia do exemplo 10. A igualdade y(x2 y + x + y) x(xy 2 +
y + 7) = y 2 7x implica que y 2 7x e divisvel por xy 2 + y + 7. Se y 2 7x 0,
como y 2 7x < xy 2 + y + 7, segue que y 2 7x = 0. Assim, (x, y) = (7t2 , 7t) para
facil checar que esses pares s
algum t N. E
ao realmente solucoes. Se y 2 7x < 0,
entao 7x y 2 > 0 e divisvel por xy 2 + y + 7. Da, xy 2 + y + 7 7x y 2 < 7x, que
nos permite concluir que y 2. Para y = 1, temos x + 8 | 7x 1 e consequentemente
x + 8 | 7(x + 8) (7x 1) = 57. Entao as u
nicas possibilidades s
ao x = 11 e x = 49,
cujos pares correspondentes s
ao (11, 1), (49, 1). Para y = 2, temos 4x + 9 | 7x 4
e consequentemente 7(4x + 9) 4(7x 4) = 79 e divisvel por 4x + 9. Nesse caso,
nao obtemos nenhuma solucao nova. Todas as solucoes para (x, y) s
ao: (7t2 , 7t)(t
N), (11, 1) e (49, 1).

Refer
encias
[1] F. E. Brochero Martinez, C. G. Moreira, N. C. Saldanha, E. Tengan - Teoria dos
N
umeros um passeio com primos e outros n
umeros familiares pelo mundo inteiro,
Projeto Euclides, IMPA, 2010.
[2] E. Carneiro, O. Campos and F. Paiva, Olimpadas Cearenses de Matem
atica 1981-2005
(Nveis J
unior e Senior), Ed. Realce, 2005.
[3] S. B. Feitosa, B. Holanda, Y. Lima and C. T. Magalhaes, Treinamento Cone Sul 2008.
Fortaleza, Ed. Realce, 2010.
[4] D. Fomin, A. Kirichenko, Leningrad Mathematical Olympiads 1987-1991, MathPro
Press, Westford, MA, 1994.
[5] D. Fomin, S. Genkin and I. Itenberg, Mathematical Circles, Mathematical Words, Vol.
7, American Mathematical Society, Boston, MA, 1966.
[6] I. Niven, H. S. Zuckerman, and H. L. Montgomery, An Introduction to the Theory of
Numbers.

Polos Olmpicos de Treinamento


Curso de Teoria dos Nmeros - Nvel 2

Aula

Prof. Samuel Feitosa

O Algoritmo de Euclides

Exemplo 1. Seja S um conjunto infinito de inteiros n


ao negativos com a seguinte propriedade: dados dois quaisquer de seus elementos, o valor absoluto da diferenca entre eles
tambem pertence a S. Se d e o menor elemento positivo de S, prove que S consiste de
todos os m
ultiplos de d.
Considere um elemento m qualquer de S. Pelo algoritmo da divisao, m = qd + r com
0 r < d. Como todos os n
umeros m d, m 2d, m 3d, . . . , m qd = r pertencem
a S e d e o menor elemento positivo de tal conjunto, devemos ter obrigatoriamente que
r = 0. Sendo assim, podemos concluir que todos os elementos de S s
ao m
ultiplos de d.
Resta mostrarmos que todos os m
ultiplos de d estao em S. Seja kd um m
ultiplo positivo
qualquer de d. Como S e infinito, existe um inteiro m S tal que m = qd > kd. Assim
todos os n
umeros m d, m 2d, . . . , m (q k)d = kd estao em S.
Definic
ao 2. Um inteiro a e um divisor comum de b e c se a | b e a | c. Se b e c n
ao s
ao
ambos nulos, denotaremos por mdc(b, c) o m
aximo divisor comum de b e c.
Como um inteiro nao nulo possui apenas um n
umero finito de divisores, se b e c s
ao ambos
nao nulos, o n
umero mdc(b, c) sempre existe, isto e, sempre esta bem definido.
Lema 3. (Euclides) Se x 6= 0, mdc(x, y) = mdc(x, x + y)
Demonstraca
o. Seja d um divisor comum de x e y. Entao d | x + y e consequentemente d
tambem a um divisor comum de x e x + y. Reciprocamente, se f e um divisor comum de
x + y e x, f tambem divide (x + y) y = x e assim f e um divisor comum de x e y. Como
os conjuntos de divisores comuns dos dois pares de n
umeros mencionados s
ao os mesmos,
o maior divisor comum tambem e o mesmo.
Entao podemos calcular:
mdc(123, 164) = mdc(123, 41) = mdc(41, 123) = mdc(41, 82) = mdc(41, 41) = 41.

POT 2012 - Teoria dos N


umeros - Nvel 2 - Aula 3 - Samuel Feitosa

Exemplo 4. Tres m
aquinas I, R, S imprimem pares de inteiros positivos em tickets. Para
a entrada (x, y), as m
aquinas I, R, S imprimem respectivamente (x y, y), (x + y, y), (y, x).
Iniciando com o par (1, 2) podemos alcancar
a) (819, 357)?
b) (19, 79)?
Para o item a), calculemos inicialmente mdc(819, 357):
mdc(819, 357) = mdc(462, 357) = mdc(105, 357) = mdc(105, 252) = . . . = mdc(21, 21) = 21.
Pelo Lema de Euclides, o mdc entre os dois n
umeros em um ticket nunca muda. Como
mdc(1, 2) = 1 6= 21 = mdc(819, 357), nao podemos alcancar o par do item a).
Para o item b), indiquemos com uma operacao de alguma das m
aquinas. Veja que:
R

(2, 1) (3, 1) (1, 3) (4, 3) . . . (19, 3) (3, 19) (22, 19) (41, 19)
R

(60, 19) (79, 19).


Observac
ao 5. Procurar invariantes sempre e uma boa estrategia para comparar configuraco
es diferentes envolvidas no problema. Confira o problema proposto 31.
Definic
ao 6. Dizemos que dois inteiros p e q s
ao primos entre si ou relativamente primos
se mdc(p, q) = 1. Dizemos ainda que a fraca
o pq e irredutvel se p e q s
ao relativamente
primos.
Exemplo 7. (IMO 1959) Prove que

21n + 4
e irredutvel para todo n
umero natural n.
14n + 3

Pelo lema de Euclides, mdc(21n+4, 14n+3) = mdc(7n+4, 14n+3) = mdc(7n+1, 7n+2) =


mdc(7n + 1, 1) = 1.
O seguinte lema sera provado na proxima aula.
Lema 8. (Propriedades do MDC) Seja mdc(a, b) = d, ent
ao:
i) Se k =
6 0, mdc(ka, kb) = kd.


a b
ii) mdc
,
= 1.
d d
iii) Se mdc(a, c) = 1, ent
ao mdc(a, bc) = d.
Exemplo 9. (Olimpada Inglesa) Se x e y s
ao inteiros tais que 2xy divide x2 + y 2 x, prove
que x e um quadrado perfeito

POT 2012 - Teoria dos N


umeros - Nvel 2 - Aula 3 - Samuel Feitosa

Se d = mdc(x, y), entao x = da e y = db, com mdc(a, b) = 1. Do enunciado, temos:


2abd2 | d2 a2 + d2 b2 da
d2 | d2 a2 + d2 b2 da
d2 | da
d | a.
Logo, a = dc, para algum c. Como x | y 2 , obtemos d2 c | d2 b2 , ou seja, c|b2 e mdc(c, b2 ) = c.
Usando que mdc(a, b) = 1 e que todo divisor comum de b e c tambem e um divisor comum
de a e b, podemos concluir que mdc(c, b) = 1. Usando o item iii) do lema anterior,
mdc(c, b2 ) = 1. Assim, c = 1 e x = d2 c = d2 .

Exemplo 10. No planeta X, existem apenas dois tipos de notas de dinheiro: $5 e $78. E
possvel pagarmos exatamente $7 por alguma mercadoria? E se as notas fossem de $ 3 e $
78?
Veja que 2 78 31 5 = 1 e consequentemente 14 78 217 5 = 7. Basta darmos
14 notas de de $ 78 para recebermos 217 notas de $ 5 como troco na compra de nossa
mercadoria. Usando as notas de $3 e $78 nao e possvel pois o dinheiro pago e recebido
como troco por algo sempre e m
ultiplo de 3 e 7 nao e m
ultiplo de 3.
Queremos estudar a versao mais geral desse exemplo. Quais s
ao os valores que podemos
pagar usando notas de $a e $b? Em particular, estaremos interessados em conhecer qual o
menor valor que pode ser pago. Para responder essa pergunta, precisaremos do algoritmo
de Euclides:
Teorema 11. (O Algoritmo de Euclides) Para os inteiros b e c > 0, aplique sucessivamente
o algoritmo da divis
ao para obter a serie de equaco
es:
b = cq1 + r1 , 0 < r1 < c,
c = r1 q 2 + r2 , 0 < r2 < r1 ,
r1 = r2 q 3 + r3 , 0 < r3 < r2 ,
..
.
rj2 = rj1 qj + rj , 0 < rj < rj1 ,
rj1 = rj qj+1
A sequencia de restos n
ao pode diminuir indefinidamente pois 0 ri < ri1 e existe apenas
um n
umero finito de naturais menores que c. Assim, para algum j, obteremos rj+1 = 0.
O maior divisor comum de b e c ser
a rj , ou seja, o u
ltimo resto n
ao nulo da sequencia de
divis
oes acima.
Demonstraca
o. Pelo Lema de Euclides,
mdc(x + qy, y) = mdc(x + (q 1)y, y) = mdc(x + (q 2)y, y) = . . . = mdc(x, y).
3

POT 2012 - Teoria dos N


umeros - Nvel 2 - Aula 3 - Samuel Feitosa

Entao,
mdc(b, c) = mdc(c, r1 ) = mdc(r1 , r2 ) = . . . = mdc(rj1 , rj ) = rj .

Exemplo 12. Calcule mdc(42823, 6409).


Pelo Algoritmo de Euclides,
42823 = 6 6409 + 4369
6409 = 1 4369 + 2040
4369 = 2 2040 + 289
2040 = 7 289 + 17
289 = 17 17.
Portanto, mdc(42823, 6409) = 17.
Podemos extrair mais informacoes do Algoritmo de Euclides. Para isso, iremos organizar
as equacoes do exemplo acima de outra forma.
Essencialmente, a equacao mdc(x+qy, y) = mdc(x, y) nos diz que podemos subtrair q vezes
um n
umero de outro sem alterar o m
aximo divisor comum do par em questao. Realizando
esse procedimento sucessivas vezes, subtraindo o n
umero menor do maior, podemos obter
pares com n
umeros cada vez menores ate que chegarmos em um par do tipo (d, d). Como o
m
aximo divisor comum foi preservado ao longo dessas operacoes, d sera o m
aximo divisor
comum procurado. Iremos repetir o exemplo anterior registrando em cada operacao quantas
vezes um n
umero e subtraido do outro. Isso sera feito atraves de dois pares de n
umeros
auxiliares:
(42823, 6409) | (1, 0)(0, 1)
(4369, 6409) | (1, 6)(0, 1)
(4369, 2040) | (1, 6)(1, 7)
(289, 2040) | (3, 20)(1, 7)
(289, 17) | (3, 20)(22, 147)
(17, 17) | (355, 2372)(22, 147)

Da primeira linha para a segunda, como subtramos 6 vezes o n


umero 6409 de 42823,
subtramos 6 vezes o par (0, 1) de (1, 0), obtendo: (1, 0) 6(0, 1) = (1, 6). Se em uma
dada linha, temos:
(x, x + qy)) | (a, b)(c, d);
entao, a proxima linha devera ser:
(x, y) | (a, b)(c aq, d bq);
4

POT 2012 - Teoria dos N


umeros - Nvel 2 - Aula 3 - Samuel Feitosa

porque representar
a a operacao de subtrairmos q vezes o primeiro n
umero do segundo. Veja
que o par (a, b) foi subtraido de (c, d) exatamente q vezes. Os n
umeros escritos nos u
ltimos
dois pares representam os coeficientes dos n
umeros originais para cada n
umero do primeiro
par. Por exemplo, analisando a linha:
(289, 2040) | (3, 20)(1, 7);
obtemos que:
289 = 3 42823 20 6409,
2040 = 1 42823 + 7 6409.
Em cada linha, essa propriedade e mantida pois a mesma subtracao que e realizada no
primeiro par tambem e realizada entre os dois u
ltimos pares. Analisando o u
ltimo par,
podemos escrever 17 como combinacao de 42823 e 6409 de duas formas diferentes:
17 = 22 42823 + 147 6409,
17 = 355 42823 + 2372 6409,

Assim, se no planeta X tivessemos apenas notas de $42823 e $6409, poderamos comprar


algo que custasse exatamente $17.
Como conclusao da discuss
ao anterior e do algoritmo de Euclides, podemos concluir que:
Teorema 13. (Bachet-B`ezout) Se d = mdc(a, b), ent
ao existem inteiros x e y tais que
ax + by = d.
De fato, a discuss
ao anterior tambem nos mostra um algoritmo para encontrarmos x e y.
Voltando `a discuss
ao sobre o planeta X, podemos concluir em virtude do teorema anterior
que qualquer valor m
ultiplo de d poder
a ser pago usando apenas as notas de $a e $b.
Como todo valor pago, necessariamente e um m
ultiplo do m
aximo divisor comum de a e
b, descobrimos que o conjunto que procur
avamos consiste precisamente do conjunto dos
m
ultiplos de d.
Observac
ao 14. (Para professores) A prova mais comum apresentada para o teorema anterior baseia-se na an
alise do conjunto de todas as combinaco
es lineares entre a e b e quase
sempre se preocupa apenas com mostrar a existencia de x e y. Acreditamos que o algoritmo
para encontrar x e y facilite o entendimento do teorema para os alunos mais jovens. Entretanto, frequentemente utilizemos apenas a parte da existencia descrita no enunciado. Alem
disso, preferimos discutir um exemplo numerico ao inves de formalizarmos uma prova e
sugerimos que o professor faca o mesmo com mais exemplos em aula.
Exemplo 15. (Olimada Russa 1995) A sequencia a1 , a2 , ... de naturais satisfaz mdc(ai , aj ) =
mdc(i, j) para todo i 6= j Prove que ai = i para todo i.

POT 2012 - Teoria dos N


umeros - Nvel 2 - Aula 3 - Samuel Feitosa

Para qualquer inteiro n, mdc(a2n , an ) = mdc(2n, n) = n, consequentemente n | an . Seja


d um divisor qualquer de an diferente de n, entao d | mdc(ad , an ). De mdc(ad , an ) =
mdc(d, n), podemos concluir que d | n. Sendo assim, todos os divisores de an que s
ao
diferentes de n s
ao divisores de n. Como ja sabemos que an = nk, para algum k, nao
podemos ter k > 1 pois nk nao divide n e assim conclumos que an = n.
Exemplo 16. Mostre que mdc(2120 1, 2100 1) = 220 1.
Pelo lema de Euclides,
mdc(2120 1, 2100 1) = mdc(2120 1 220 (2100 1), 2100 1),
= mdc(220 1, 2100 1),
= mdc(220 1, 2100 1 280 (220 1)),
= mdc(220 1, 280 1),
= mdc(220 1, 280 1 260 (220 1)),
= mdc(220 1, 260 1),
= mdc(220 1, 260 1 240 (220 1)),
= mdc(220 1, 240 1),
= mdc(220 1, 240 1 220 (220 1)),
= mdc(220 1, 220 1) = 220 1.

Exemplo 17. (Olimpada Russa 1964) Sejam x, y inteiros para os quais a fraca
o
a=

x2 + y 2
xy

e inteira. Ache todos os possveis valores de a.


A primeira estrategia e cancelar
ao caso em que x e y s
ao primos

x
y

os fatores comuns com o objetivo de reduzir o problema


entre si. Seja d = mdc(x, y), com
= d x0
, mdc(x0 , y0 ) = 1,
= d y0

entao
a=

x 0 2 + y0 2
x2 + y 2
=

xy
x 0 y0

Nessa condicao, como x0 divide y02 e y0 divide x20 , cada um deles e igual a 1, donde
a=

12 + 12
= 2.
11

POT 2012 - Teoria dos N


umeros - Nvel 2 - Aula 3 - Samuel Feitosa

Definic
ao 18. Os inteiros a1 , a2 , . . . , an , todos diferentes de zero, possuem m
ultiplo comum
b se ai |b para i = 1, 2, . . . , n(note que a1 a2 . . . an e um m
ultiplo comum). O menor m
ultiplo
comum positivo para tal conjunto de inteiros e chamado de mnimo m
ultiplo comum e ser
a
denotado por mmc(a1 , a2 , . . . , an ).
Proposic
ao 19. Se a e b s
ao n
ao nulos, ent
ao: mmc(a, b) mdc(a, b) = |ab|.
(A prova desta proposicao tambem sera deixada para a pr
oxima secao)
Exemplo 20. (Olimpada Russa 1995) Sejam m e n interios positivos tais que:
mmc(m, n) + mdc(m, n) = m + n.
Prove que um deles e divisvel pelo o outro.
Se d = mdc(m, n), entao podemos escrever m = da e n = db. Pela proposicao anterior,
mmc(m, n) =

d2 ab
= dab.
d

Temos:
mmc(m, n) + mdc(m, n) m n = 0
dab + d da db = 0
ab + 1 a b = 0
(a 1)(b 1) = 0.
Portanto, ou a = 1 e m | n ou entao b = 1 e n | m.
Exemplo 21. (Torneio das Cidades 1998) Prove que, para quaisquer inteiros positivos a e
b, a equaca
o mmc(a, a + 5) = mmc(b, b + 5) implica que a = b.
Para o item a), como (a + 5) a = 5, temos mdc(a, a + 5) e igual a 1 ou 5. O mesmo vale
para mdc(b, b + 5). Pela proposicao anterior, temos:

mmc(a, a + 5) =
mmc(b, b + 5) =

a(a + 5)
,
mdc(a, a + 5)
b(b + 5)
.
mdc(b, b + 5)

Suponha que mdc(a, a + 5) = 5 e mdc(b, b + 5) = 1, entao a(a + 5) = 5b(b + 5). Consequentemente, a e m


ultiplo de 5 e a(a + 5) e m
ultiplo de 25. Isso implica que b(b + 5) tambem e
m
ultiplo de 5 e que mdc(b, b + 5) > 1. Uma contradicao. Analogamente, nao podemos ter
mdc(a, a + 5) = 1 e mdc(b, b + 5) = 5. Sendo assim, mdc(a, a + 5) = mdc(b, b + 5) e:
a(a + 5) b(b + 5) = 0
(a b)(a + b + 5) = 0.
Como a + b + 5 > 0, conclumos que a = b.
7

POT 2012 - Teoria dos N


umeros - Nvel 2 - Aula 3 - Samuel Feitosa

Exemplo 22. Uma m


aquina f executa operaco
es sobre o conjunto de todos os pares de
inteiros positivos. Para cada par de inteiros positivos, ela fornece um inteiro dado pelas
regras:
f (x, x) = x, f (x, y) = f (y, x), (x + y)f (x, y) = yf (x, x + y).
Determine f (2012, 2012! + 1).
Claramente mmc(x, x) = x e mmc(x, y) = mmc(y, x). Usando a proposicao anterior e o
lema de Euclides temos:
(x + y)mmc(x, y) = (x + y)

xy
x(x + y)
=y
= y mmc(x, x + y)
mdc(x, y)
mdc(x, x + y)

Temos entao uma forte suspeita de que f = mmc. Seja S o conjunto de todos os pares de inteiros positivos (x, y) tais que f (x, y) 6= mmc(x, y), e seja (m, n) o par em S
com a soma m + n minima. Note que todo par da forma (n, n) nao esta em S pois
f (n, n) = n = mmc(n, n). Assim, devemos ter m 6= n. Suponha sem perda de generalidade
que n > m. Portanto:
nf (m, n m) = [m + (n m)]f (m, n m)
= (n m)f (m, m + (n m))
nm
f (m, n m) =
f (m, n)
n
Como o par (m, m n) nao esta em S, dado que a soma de seus elementos e menor que
m + n, temos:
f (m, n m) = mmc(m, n m)

nm
f (m, n) = (n m)mmc(m, m + (n m))
n
f (m, n) = mmc(m, n)

Uma contradicao. Desse modo, S deve ser um conjunto vazio e f (x, y) = mmc(x, y)
para todos os pares de inteiros positivos. Como 2012 | 2012!, mdc(2012, 2012! + 1) = 1 e
consequentemente mmc(2012, 2012! + 1) = 2012(2012! + 1).

Problemas Propostos

Problema 23. Calcule:


a) mdc(n, n2 + n + 1).
b) mdc(3 2012, 2 2012 + 1).

POT 2012 - Teoria dos N


umeros - Nvel 2 - Aula 3 - Samuel Feitosa

c) mdc


240 + 1 8
,2 + 1 .
28 + 1

Problema 24. Encontre mdc(2n + 13, n + 7)


Problema 25. Prove que a fraca
o

12n+1
30n+2

e irredutvel.

Problema 26. Sejam a, b, c, d inteiros n


ao nulos tais que ad bc = 1. Prove que
fraca
o irredutvel.

a+b
c+d

e uma

Problema 27. Mostre que mdc(am 1, an 1) = amdc(m,n) 1.


Problema 28. Mostre que se mdc(a, b) = 1, ent
ao:
mdc(a + b, a2 ab + b2 ) = 1 ou 3
Problema 29. Dado que mdc(a, 4) = 2, mdc(b, 4) = 2, prove que:
mdc(a + b, 4) = 4.
Problema 30. Prove que, para todo natural n,
mdc(n! + 1, (n + 1)! + 1) = 1.
Problema 31. No exemplo 4, determine todos os pares que podem ser obtidos comecando-se
com o par (1, 2).
Problema 32. Qual o m
aximo divisor comum do conjunto de n
umeros:
{16n + 10n 1, n = 1, 2, 3 . . .}?
a
Problema 33. A sequencia Fn de Farey e a sequencia de todos as fraco
es irredutveis
b
com 0 a b n arranjados em ordem crescente.
F1
F2
F3
F4
F5
F6

=
=
=
=
=
=

{0/1, 1/1}
{0/1, 1/2, 1/1}
{0/1, 1/3, 1/2, 2/3, 1/1}
{0/1, 1/4, 1/3, 1/2, 2/3, 3/4, 1/1}
{0/1, 1/5, 1/4, 1/3, 2/5, 1/2, 3/5, 2/3, 3/4, 4/5, 1/1}
{0/1, 1/6, 1/5, 1/4, 1/3, 2/5, 1/2, 3/5, 2/3, 3/4, 4/5, 5/6, 1/1}

a em algum Fn . Mostre que se m/n


Claramente, toda fraca
o ab < 1 com mdc(a, b) = 1, est

e m /n s
ao fraco
es consecutivas em Fn temos |mn nm | = 1.
Problema 34. (Resvista Quantum - Jornal Kvant) Todas as fraco
es irredutveis cujos denominadores n
ao excedem 99 s
ao escritas em ordem crescente da esquerda para a direita:
1 1
a 5 c
, ,..., , , ,...
99 98
b 8 d
Quais s
ao as fraco
es

5
a c
e em cada lado de ?
b d
8
9

POT 2012 - Teoria dos N


umeros - Nvel 2 - Aula 3 - Samuel Feitosa

Problema 35. (OBM) Para cada inteiro positivo n > 1, prove que 1 +
e inteiro.

1
2

1
3

+...+

Problema 36. Determine todas as soluco


es em inteiros positivos para

1
a

1
b

= 1c .

1
n

n
ao

Problema 37. Inteiros positivos a e b, relativamente primos, s


ao escolhidos de modo que
a+b
seja tambem um inteiro positivo. Prove que pelo menos um dos n
umeros ab + 1 e
ab
4ab + 1 e um quadrado perfeito.
Problema 38. (IMO 1979) Sejam p, q n
umeros naturais primos entre si tais que:
p
1 1
1
1
= 1 + ...
+
.
q
2 3
1318 1319
Prove que p e divisvel por 1979.

Respostas, Dicas e Solu


c
oes

23. (a)
mdc(n, n2 + n + 1) = mdc(n, n2 + n + 1 n(n + 1)),
= mdc(n, 1),
= 1.
(b)
mdc(3 2012, 2 2012 + 1) = mdc(3 2012 (2 2012 + 1), 2 2012 + 1),
= mdc(2012 1, 2 2012 + 1),
= mdc(2012 1, 2 2012 + 1 2(2012 1)),
= mdc(2012 1, 3),
= mdc(2012 1 3 670, 3),
= mdc(2, 3) = 1.
Outra opcao seria observar que o mdc procurado deve dividir o n
umero 3(2
2012 + 1) 2(3 2012) = 3 e que 2 2012 + 1 nao e m
ultiplo de 3.
(c)
mdc



240 + 1 8
32
24
16
8
8
,
2
+
1
=
mdc
2
+
2
+
2
+
2
+
1,
2
+
1
,
28 + 1


= mdc (232 1) + (224 + 1) + (216 1) + (28 + 1) + 1, 28 + 1 ,

= mdc(1, 28 + 1) = 1.

10

POT 2012 - Teoria dos N


umeros - Nvel 2 - Aula 3 - Samuel Feitosa

24.
mdc(2n + 13, n + 7) = mdc(2n + 13 2(n + 7), n + 7),
= mdc(2n + 13 2(n + 7), n + 7),
= mdc(1, n + 7) = 1
25.
mdc(12n + 1, 30n + 2) = mdc(12n + 1, 30n + 2 2(12n + 1)),
= mdc(12n + 1, 6n),
= mdc(12n + 1 2(6n), 6n),
= mdc(1, 6n) = 1

26. Seja f = mdc(a + b, c + d). Entao f | d(a + b) b(c + d) = 1 e consequentemente


f = 1.
27. Veja que
mdc(am 1, an 1) = mdc(amn 1 + (an 1)amn , an 1)
= mdc(amn 1, an 1)

O resultado segue aplicando o Algoritmo de Euclides aos expoentes.


28. Seja f = mdc(a + b, a2 ab + b2 ). Entao f | (a + b)2 (a2 ab + b2 ) = 3ab. Se
mdc(f, a) > 0, devemos ter mdc(f, b) > 0 pois f | a + b. O mesmo argumento vale
para mdc(f, b) > 0. Assim, mdc(f, a) = mdc(f, b) = 1. Portanto, f | 3.
30. Pelo lema de Euclides,

mdc(n! + 1, (n + 1)! + 1) = mdc(n! + 1, (n + 1)! + 1 (n + 1)(n! + 1))


= mdc(n! + 1, n)
= mdc(n! + 1 n[(n 1)!], n) = 1
34. Sejam l = mmc{1, 2, . . . , n} e ai = l/i. A soma considerada e
a1 + a2 + . . . + an
.
l
Queremos analisar o expoente do fator 2 no numerador e no denominador. Seja k tal
que 2k n < 2k+1 . Entao 2k ||l e ai e par para todo i 6= 2k . Como a2k e mpar, segue
que o numerador e mpar enquanto que o denominador e par. Consequentemente a
fracao anterior nao representa um inteiro.
11

36. Sejam d = mdc(a, b), a = dx, b = dy. Consequentemente mdc(x, y) = 1 e podemos


escrever a equacao como:
1 1
1
+
=

a b
c
bc + ac = ab
dyc + dxc = d2 xy
c(x + y) = dxy

Como mdc(xy, x + y) = 1 pois mdc(x, y) = 1, devemos ter xy | c e consequentemente


c = xyk. Assim, d = k(x + y). O conjunto solucao e formado pelas triplas (a, b, c)
onde (a, b, c) = (kx(x + y), ky(x + y), xyk) com mdc(x, y) = 1 e x, y e k inteiros
positivos.
38. Use a identidade de Catalao:
1

1
1
1
1
1 1 1
+ + ...
=
+
+ ... +
2 3 4
2n
n+1 n+2
2n

Em seguida, agrupe os termos da forma


fracao obtida.

1
1
+
e analise o numerador da
n + i 2n i + 1

Refer
encias
[1] S. B. Feitosa, B. Holanda, Y. Lima and C. T. Magalhaes, Treinamento Cone Sul 2008.
Fortaleza, Ed. Realce, 2010.
[2] D. Fomin, A. Kirichenko, Leningrad Mathematical Olympiads 1987-1991, MathPro
Press, Westford, MA, 1994.
[3] D. Fomin, S. Genkin and I. Itenberg, Mathematical Circles, Mathematical Words, Vol.
7, American Mathematical Society, Boston, MA, 1966.
[4] I. Niven, H. S. Zuckerman, and H. L. Montgomery, An Introduction to the Theory of
Numbers.

Polos Olmpicos de Treinamento


Curso de Teoria dos Nmeros - Nvel 2

Aula

Prof. Samuel Feitosa

N
umeros Primos, MDC e MMC.

Definic
ao 1. Um inteiro p > 1 e chamado n
umero primo se n
ao possui um divisor d
satisfazendo 1 < d < p. Se um inteiro a > 1 n
ao e primo, ele e chamado de n
umero
composto. Um inteiro m e chamado de composto se |m| n
ao e primo.
O proximo teorema nos diz que os primos s
ao as pecasfundamentais dos n
umeros inteiros:
Teorema 2. Todo inteiro n, maior que 1, pode ser expresso como o produto de n
umero
primo.
Demonstraca
o. Se o inteiro n e um primo, entao ele mesmo e o produto de um u
nico fator primo. Se o inteiro n nao e primo, existe uma decomposicao do tipo: n = n1 n2 com
1 < n1 < n e 1 < n2 < n. Repetindo o argumento para n1 e n2 , podemos escrever n como
o produto de primos ou podemos obter parcelas menores escrevendo n como um produto
de naturais. Como nao existe uma sucess
ao infinita de naturais cada vez menores, ap
os um
n
umero finito de operacoes desse tipo, poderemos escrever n como um produto de n
umeros
primos.
Quantos n
umeros primos existem?
Teorema 3. (Euclides) Existem infinitos n
umeros primos.
Demonstraca
o. Suponha, por absurdo, que exita apenas uma quantidade finita de primos:
p1 , p2 , . . . , pn . Considere o n
umero X = p1 p1 . . . pn + 1. Pelo teorema anterior, esse n
umero
deve ser o produto de alguns elementos do conjunto de todos os n
umeros primos. Entretanto, nenhum dos primos pi divide X.
Exemplo 4. Existe um bloco de 1000 inteiros consecutivos n
ao contendo nenhum primo?
Sim. Um exemplo e o conjunto 1001! + 2, 1001! + 3, . . . , 1001! + 1001. Veja i | 1001! + i para
todo i = 2, 3, . . . , 1001.

POT 2012 - Teoria dos N


umeros - Nvel 2 - Aula 4 - Samuel Feitosa

Exemplo 5. (Torneio das Cidades) Existe um bloco de 1000 inteiros consecutivos contendo
apenas um primo?
Para cada bloco de 1000 n
umeros consecutivos, contemos sua quantidade de n
umeros primos. Por exemplo, no bloco 1, 2, 3, . . . , 1000, temos 168 n
umeros primos (mas s
o usaremos
o fato de que existem mais de dois primos nesse bloco). Comparando os blocos consecutivos k + 1, k + 2, . . . , k + 1000 e k + 2, k + 3, . . . , k + 1001, ou o n
umero de n
umeros primos
aumenta em uma unidade, ou fica constante ou diminui em uma unidade. Analisando todos os blocos consecutivos desde 1, 2, . . . , 1000 ate 1001! + 2, 1001! + 3, . . . , 1001! + 1001,
o n
umero de n
umeros primos deve ser igual `a 1 em algum deles. Para ver isso, usaremos um argumento de continuidade discreta: Comecando com o n
umero 168 e realizando
alteracoes de no m
aximo uma unidade na quantidade de primos em cada bloco, para chegarmos no n
umero 0, necessariamente deveremos passar pelo n
umero 1 em algum momento.
Relembremos um importante resultado da aula passada:
Teorema 6. (Bachet- B`ezout) Se d = mdc(a, b), ent
ao existem inteiros x e y tais que
ax + by = d.
Proposic
ao 7. Sejam a, b e c inteiros positivos com a | bc e mdc(a, b) = 1. Ent
ao, a | c.
Demonstraca
o. Pelo teorema anterior, existem x e y inteiros tais que ax + by = 1. Assim,
acx + bcy = c. Como a | acx e a | bcy, podemos concluir que a | c.
Em particular, se p e um n
umero primo e p | ab, entao p | a ou p | b. Podemos usar esse
fato para garantir a unicidade em nosso primeiro teorema, obtendo o importante:
Teorema 8. (Teorema Fundamental da Aritmetica) A fatoraca
o de qualquer inteiro n > 1,
em fatores primos, e u
nica a menos da ordem dos fatores.
possvel colocarmos 1995 n
Exemplo 9. (R
ussia 1995) E
umeros naturais ao redor de um
crculo de modo que para quaisquer dois n
umeros vizinhos a raz
ao entre o maior e o menor
seja um n
umero primo?
N
ao, e impossvel. Suponha, por absurdo, que isso seja possvel e denotemos por
a
a0 , a1 , . . . , a1995 = a0 tais inteiros. Entao, para k = 1, . . . , 1995, k1
e primo ou o inak
verso de um primo. Suponha que a primeira situacao ocorra m vezes e a segunda ocorra
1995 m vezes entre esses quocientes. Como o produto de todos os n
umeros da forma
ak1
,
para
k
=
1,
.
.
.
,
1995

e
igual
a
`
1,
podemos
concluir
que
o
produto
de
m primos deve
ak
ser igual ao produto de 1995 m primos. Em virtude da fatoracao u
nica, m = 1995 m.
Um absurdo pois 1995 e mpar.
Proposic
ao 10. Se as fatoraco
es em primos de n e m s
ao:
n = p1 1 p2 2 . . . pk k ,
m = p1 1 p2 2 . . . pk k .

POT 2012 - Teoria dos N


umeros - Nvel 2 - Aula 4 - Samuel Feitosa

Ent
ao, mdc(m, n) = p11 p22 . . . pkk e mmc(m, n) = p11 p22 . . . pkk , onde i e o menor dentre
{i , i } e i e o maior dentre {i , i }.
Proposic
ao 11. Se a e b s
ao inteiros positivos, mostre que mmc(a, b)mdc(a, b) = ab.
Demonstraca
o. Basta usar a proposicao anterior e observar que:
max{x, y} + min{x, y} = x + y.
possvel que mmc(a, b) = mmc(a + c, b + c)
Exemplo 12. (Torneio das Cidades 1998) E
para alguma conjunto {a, b, c} de inteiros positivos?
N
ao. Suponha que a + c e b + c possuem algum divisor primo p. Como p | mmc(a + c, b + c),
caso existam tais inteiros, devemos ter que p | mmc(a, b). Assim, usando que pelo menos
um dentre a e b e divisvel por p podemos concluir que c tambem e divisvel por p. Entao,
podemos cancelar o fator p:




a b
a+c b+c
mmc(a + c, b + c)
mmc(a, b)
mmc
,
=
= mmc
,
=
.
p p
p
p
p
p
Efetuando alguns cancelamentos, podemos supor entao que a+c e b+c nao possuem fatores
primos em comum. Obtivemos um absurdo pois:
mmc(a + c, b + c) = (a + c)(b + c) > ab mmc(a, b).
Exemplo 13. (OCM 2005) Determinar os inteiros n > 2 que s
ao divisveis por todos os
primos menores que n.
Como mdc(n, n 1) = 1, se n 1 possui algum fator primo, ele nao dividira n. Assim,
n 1 < 2. Consequentemente nao existe tal inteiro.

Exemplo 14. Mostre que n4 + n2 + 1 e composto para n >1.

Veja que n4 + n2 + 1 = n4 + 2n2 + 1 n2 = (n2 + 1)2 n2 = (n2 + n + 1)(n2 n + 1).


Para n > 1, n2 n + 1 = n(n 1) + 1 > 1 e assim n4 + n2 + 1 e o produto de dois inteiros
maiores que 1.
Exemplo 15. Mostre que n4 + 4n e composto para todo n > 1.
Se n e par, certamente o n
umero em quest
ao e divisvel por 4. Para o caso em que n e
impar, iremos usar a fatoracao:
a4 + 4b4 = a4 + 4a2 b2 + 4b4 4a2 b2 = (a2 + 2b2 ) 4b2 b2 = (a2 2ab + 2b2 )(a2 + 2ab + 2b2 ).
Para n da forma 4k + 1, faca a = n e b = 4k . Para n da forma 4k + 3, faca a = n e
b = 22k+1 .
Exemplo 16. Se 2n + 1 e um primo mpar para algum inteiro positivo n, prove que n e uma
potencia de 2.

POT 2012 - Teoria dos N


umeros - Nvel 2 - Aula 4 - Samuel Feitosa

Ja vimos que an 1 = (a 1)(an1 + an2 + . . . + 1). Se n e impar,


(a)n 1 = (a 1)((a)n1 + (a)n2 + . . . + 1)
an + 1 = (a + 1)(an1 an2 + . . . a + 1)

Sendo assim, se n possusse algum divisor primo mpar p com n = pb, poderamos escrever:
2n + 1 = (a + 1)(an1 an2 + . . . a + 1), onde a = 2b . Como an1 an2 + . . . a + 1 > 1,
o n
umero 2n + 1 nao seria primo.
Exemplo 17. Dados que p, p + 10 e p + 14 s
ao n
umeros primos, encontre p.
Vamos analisar os possveis restos na divisao por 3 de p. Se p deixa resto 1, entao p + 14
e um m
ultiplo de 3 maior que 3 e consequentemente nao poder
a ser um n
umero primo. Se
o resto e 2, entao p + 10 e um m
ultiplo de 3 maior que 3 e tambem nao poder
a ser um
n
umero primo. Assim, o resto de p por 3 e 0 e consequentemente p = 3.
n

Exemplo 18. (Austria-Pol


onia) Dados naturais n e a > 3 mpar, mostre que a2 1 tem
pelo menos n + 1 divisores primos distintos.

Usando a fatoracao da diferenca de quadrados, temos que:


k

k1

a2 1 = (a2
m

k2

+ 1)(a2

+ 1) . . . (a + 1)(a 1).

Assim, a2 + 1 | a2 1 se k > m. Como a e impar, podemos concluir que:


k

mdc(a2 + 1, a2 + 1) = mdc(a2 1 + 2, a2 + 1) = mdc(2, a2 + 1) = 2.


Sendo assim, na fatoracao:
n

n2

n1

a2 1
+ 1)
(a2
+ 1) (a2
(a + 1) (a 1)
=
...
,
n
2
2
2
2
2
temos o produto de pelo menos n inteiros primos entre si e consequentemente seus fatores
2i

primos s
ao distintos. Para cada termo (a 2+1) , temos um fator primo pi+1 diferente de 2.
n
Da, a2 1 possui pelo menos n + 1 fatores primos distintos, a saber, {2, p1 , p2 , . . . , pn }.
Exemplo 19. (Rioplatense 1999) Sejam p1 , p2 , . . . , pk primos distintos. Considere todos os
inteiros positivos que utilizam apenas esses primos (n
ao necessariamente todos) em sua
fatoraca
o em n
umeros primos, formando assim uma seq
uencia infinita
a1 < a2 < < an < .
Demonstre que, para cada natural c, existe um natural n tal que
an+1 an > c.

POT 2012 - Teoria dos N


umeros - Nvel 2 - Aula 4 - Samuel Feitosa

Suponha, por absurdo, que exista c > 0 tal que an+1 an c, n N. Isso significa que
as diferencas entre os termos consecutivos de (an )n1 pertencem ao conjunto {1, 2, . . . , c},
logo s
ao finitas. Sejam d1 , d2 , . . . , dr essas diferencas. Seja i o maior expoente de pi que
aparece na fatoracao de todos os dj .
claro que M pertence `a seq
Considere entao o n
umero M = p1 1 +1 p2 2 +1 pk k +1 . E
uencia,
ou seja, M = an , para algum n. Vejamos quem sera an+1 . Por hip
otese, existe i tal que
an+1 an = di . Como an+1 > an , existe um primo pj que divide an+1 com expoente maior
ou igual a j + 1. Caso contrario,
an < an+1 < p1 1 +1 p2 2 +1 pk k +1 = an ,
+1

absurdo. Da, pj j

+1

|an pj j

|di , novamente um absurdo, pela maximalidade de j .

Logo, o conjunto de todas as diferencas nao pode ser finito e, portanto, dado qualquer
c > 0, existe um natural n tal que an+1 an > c.

Problemas Propostos
Problema 20. Dado que p, 2p + 1 e 4p2 + 1 s
ao n
umeros primos, encontre p.
Problema 21. Dado o par de primos p e 8p2 + 1, encontre p.
Problema 22. Dado o par de primos p e p2 + 2, prove que p3 + 2 tambem e um n
umero
primo.
Problema 23. Dado que p, 4p2 + 1 e 6p2 + 1 s
ao n
umeros primos, encontre p.
n

Problema 24. Os n
umeros de Fermat s
ao os n
umeros da forma 22 + 1. Prove que o
conjunto dos divisores primos dos termos da seq
uencia de Fermat e infinito.
Problema 25. Mostre que todo inteiro n pode ser escrito de maneira u
nica na forma n = ab,
onde a e um inteiro livre de quadrado e b e um quadrado perfeito. Um inteiro e dito livre
de quadrado se n
ao e divisvel por nenhum quadrado perfeito maior que 1.
Problema 26. Prove que todo primo maior que 3 e da forma 6k+1 ou 6k+5.
Problema 27. Prove que todo inteiro da forma 3k+2 tem um fator primo da mesma forma.
Problema 28. Prove que existem infinitos primos da forma 4k+3 e 6k +5.
Problema 29. Prove que se n e composto, ent
ao possui um fator primo p

n.

Problema 30. (OBM 1998) S


ao dados 15 n
umeros naturais maiores que 1 e menores que
1998 tais que dois quaisquer s
ao primos entre si. Mostre que pelo menos um desses 15
n
umeros e primo.
Problema 31. Mostre que n|(n-1)! para todo n
umero composto n.

POT 2012 - Teoria dos N


umeros - Nvel 2 - Aula 4 - Samuel Feitosa

Problema 32. Suponha que n >1. Mostre que a soma dos inteiros dos inteiros positivos
n
ao excedendo n divide o produto dos inteiros positivos n
ao excedendo n se, e somente se,
n e composto.
Exemplo 33. (R
ussia 1995) Encontre todos os primos p para os quais p2 + 11 tenha exatamente seis divisores distintos, incluindo 1 e p2 + 11.
Problema 34. (Irlanda 2002 ) Encontre todas as soluco
es inteiras positivas de p(p + 3) +
q(q + 3) = n(n + 3), onde p, q s
ao primos.
Exemplo 35. Prove que qualquer quadrado perfeito positivo tem mais divisores que deixam
resto 1 na divis
ao por 3 do que divisores que deixam resto 2 na divis
ao por 3.

Dicas e Solu
c
oes
19. Analisemos o resto de p na divisao por 3. Se p deixar resto 1, o n
umero 2p + 1 sera
divisvel por 3. Se p deixar resto 2, o n
umero 4p + 1 sera divisvel por 3. Em ambos
os casos, 2p + 1, 4p + 1 > 3 e obtemos assim um absurdo.
20. Analisemos o resto de p na divisao por 3. Se p deixa resto 1 ou 2, p2 deixa resto 1
e consequentemente 8p2 + 1 deixa resto 0 por 3 mas certamente e maior que 3. Um
absurdo, logo p = 3.
21. Analisemos o resto na divisao por 3. Se p nao e m
ultiplo de 3, p2 + 2 e divisvel por
3
3 e maior que 3. Um absurdo, logo p = 3 e p + 2 = 29.
22. Analise os restos na divisao por 5.
23. Iremos usar a fatoracao do exemplo 17:
n

n1

22 1 = (22

n2

+ 1)(22

+ 1) . . . (2 + 1)(2 1).

Assim, se k > m,
k

mdc(22 + 1, 22 + 1) = mdc(22 1 + 2, 22 + 1) = mdc(2, 22 + 1) = 1,


produzindo que quaisquer dois n
umeros de Fermat distintos s
ao primos entre si e isso
necessariamente implica que o conjunto de seus divisores primos e infinito.
24. Analise os restos na divisao por 2 e 3.
27. Tente imitar a prova de Euclides para a existencia de infinitos primos.
29. Se n e composto, podemos escrever n = ab com 1 < a b <. Assim, a2 n e

a n. Para terminar, basta considerar qualquer divisor primo de a.


30. Dado 1 < n < 1998, se ele nao for primo, usando o exerccio anterior, ele tem que
ter um fator primo menor que 1998, ou seja, um fator primo menor que 45. Como s
o
existem 14 primos menores que 45, e s
ao 15 n
umeros, um deles sera primo.
6

31. Escreva n = ab e analise as aparicoes de a e b no produto (n 1) (n 2) . . . 2 1.


33. Se p 6= 3, 3 | p2 + 11. Analogamente, se p 6= 2, 4 | p2 + 11. Assim, exceto nesses dois casos, 12 | p2 + 11 e podemos encontrar mais que 6 divisores distintos:
{1, 2, 3, 4, 6, 12, p2 + 11}. Agora, teste p = 2 e p = 3 para verificar que p = 3 e a u
nica
solucao.
34. Seja

m
n = 3 p1 1 pnn q11 qm

a decomposicao de n em fatores primos, onde cada pi deixa resto 1 por 3 e cada qj


deixa resto 2 por 3. Entao
2m
.
n2 = 32 p121 pn2n q121 qm

Um divisor de n2 deixa resto 1 por 3 se e somente se possuir uma quantidade par de


primos qj , contados com repeticao. Mais especificamente, se e somente se a soma dos
expoentes de q1 , . . . , qm for par. Assim, a quantidade de divisores dessa forma e igual
a
D1 = (21 + 1) (2n + 1)


1
(21 + 1)(22 + 1) (2m + 1) + 1 .
2

Enquanto para se obter um divisor que deixe resto 2 por 3, precisamos de uma
quantidade mpar de fatores primos da forma 3k+2. Assim, a quantidade de divisores
dessa forma e:


1
D2 := (21 + 1)(22 + 1) (2n + 1)
(21 + 1)(22 + 1) (2m + 1) .
2
Da, segue facilmente que D1 > D2 .

Refer
encias
[1] E. Carneiro, O. Campos and F. Paiva, Olimpadas Cearenses de Matem
atica 1981-2005
(Nveis J
unior e Senior), Ed. Realce, 2005.
[2] S. B. Feitosa, B. Holanda, Y. Lima and C. T. Magalhaes, Treinamento Cone Sul 2008.
Fortaleza, Ed. Realce, 2010.
[3] D. Fomin, A. Kirichenko, Leningrad Mathematical Olympiads 1987-1991, MathPro
Press, Westford, MA, 1994.
[4] D. Fomin, S. Genkin and I. Itenberg, Mathematical Circles, Mathematical Words, Vol.
7, American Mathematical Society, Boston, MA, 1966.
[5] I. Niven, H. S. Zuckerman, and H. L. Montgomery, An Introduction to the Theory of
Numbers.

Polos Olmpicos de Treinamento


Curso de Teoria dos Nmeros - Nvel 2

Aula

Prof. Samuel Feitosa

Congru
encias I

Definic
ao 1. Dizemos que os inteiros a e b s
ao congrentes m
odulo m se eles deixam o
mesmo resto quando divididos por m. Denotaremos isso por a b (mod m).
Por exemplo, 7 2 (mod 5), 9 3 (mod 6), 37 7 (mod 10) mas 5 6 3 (mod 4). Veja
que a b (mod m) se, e somente se, m | a b.
Teorema 2. Se a b (mod m) e c d (mod m), ent
ao:
i) a + c b + d (mod m)
ii) a c b d (mod m)
iii) ka kb (mod m) k Z
iv) ac bd (mod m)
v) ak bk (mod m) k N
vi) Se mdc(k, m) = d, ent
ao ka kb (mod m) a b (mod m/d)
Demonstraca
o. Sejam q1 e q2 tais que:
a b = q1 m
c d = q2 m
Entao, (a + c) (b + d) = (q1 + q2 )m. Logo, a + c e b + d deixam o mesmo resto por m e
consequentemente a + c b + d (mod m). Usando que a b (mod a)k bk e que m | a b,
conclumos que m (mod a)k bk . Os demais itens serao deixados para o leitor.
Em termos praticos, podemos realizar quase todas as operacoes elementares envolvendo
igualdade de inteiros. Uma das diferencas cruciais e a operacao de divisao como mostra o
u
ltimo item do teorema anterior.

POT 2012 - Teoria dos N


umeros - Nvel 2 - Aula 5 - Samuel Feitosa

Exemplo 3. Calcule o resto de 4100 por 3.


Como 4 1 (mod 3), temos 4100 1100 = 1 (mod 3).
Exemplo 4. Calcule o resto de 4100 por 5.
Como 4 1 (mod 5), temos 4100 (1)100 = 1 (mod 5).
Exemplo 5. Calcule o resto de 4100 por 7.
Voce deve ter percebido que encontrar relacoes do tipo a 1 (mod m) podem simplificar
bastante o calculo de ak (mod m). Procuremos alguma relacao como essa para 4 e 7. Veja
que:
40 1 (mod 7), 41 4 (mod 7), 42 2 (mod 7), 43 1 (mod 7).
Assim,
499 = (43 )33 133 = 1 (mod 7).
Como 43 1 (mod 7), os restos das potencias de 4 na divisao por 7 se repetem periodicamente de 3 em 3 pois 43k+r 43k 4r 4r (mod 7).
Exemplo 6. Qual o resto de 3636 + 4141 na divis
ao por 77?
Inicialmente devemos perceber que existe uma relacao entre os n
umeros do problema: 36 +
41 = 77. Assim:
36 41 (mod 77),
(36)41 4141
36

36

36 (1 36 ) 36

(mod 77),
+ 4141

(mod 77).

Nosso proximo passo e encontrar o resto de 365 na divisao por 77. Como 36 1 (mod 7),
365 1 (mod 7). Alem disso, 36 3 (mod 1)1 produzindo 365 35 1 (mod 1)1.
Como mdc(7, 11) = 1 e ambos dividem 365 1, podemos concluir que 77 | 365 1. Logo,
3636 + 4141 deixa resto 0 na divisao por 77.
Exemplo 7. Prove que p2 1 e divisvel por 24 se p e um primo maior que 3.
Se p e um primo maior que 3, p 1 (mod 3) e p 1 (mod 2). Da, p2 1 (mod 3).
Alem disso, se p = 2k + 1, segue que p2 = 4k(k + 1) + 1 1 (mod 8) pois k(k + 1) e par.
Como mdc(8, 3) = 1 e ambos dividem p2 1, segue que 24 | p2 1.
Exemplo 8. (OCM-2001) Achar o menor natural n tal que 2001 e a soma dos quadrados
de n inteiros
Podemos concluir da solucao do problema anterior que todo todo inteiro mpar ao quadrado
deixa resto 1 por 8. Usemos isso para estimar o valor de n. Sejam x1 , x2 , . . . , xn inteiros
mpares tais que:
x21 + x22 + . . . x2n = 2001.

POT 2012 - Teoria dos N


umeros - Nvel 2 - Aula 5 - Samuel Feitosa

Analisando a congruencia m
odulo 8, obtemos:
x21 + x22 + . . . x2n = 2001

(mod 8)

1 + 1 + ... + 1 1

(mod 8)

n 1

(mod 8)

Como 2001 nao e quadrado perfeito, nao podemos ter n = 1. O proximo candidado para
n seria 1 + 8 = 9. Se exibirmos um exemplo para n = 9, teremos achado o valor mnimo.
Veja que:
2001 = 432 + 112 + 52 + 12 + 12 + 12 + 12 + 12 + 12 .
Exemplo 9. (IMO) Seja s(n) a soma dos dgitos de n. Se N = 44444444 , A = s(N ) e
B = s(A). Quanto vale s(B)?
Pelo criterio de divisibilidade por 9, N A B (mod 9). Inicialmente calculemos o
resto de N por 9. Como 4444 16 7 (mod 9), precisamos encontrar 74444 (mod 9).
Seguindo os metodos dos primeiros exemplos, seria interessante encontrarmos um inteiro r
tal que 7r 1 (mod 9). O menor inteiro positivo com essa propriedade e r = 3. Como
4444 = 1481 3 + 1, temos:
74444 714813+1 (73 )1481 7 7

(mod 9).

Nosso proximo passo e estimar o valor de s(B). Como N = 44444444 < 1054444 , A =
s(N ) 5 4444 9 = 199980. Alem disso, B = s(A) 1 + 9 5 = 46 e s(B) 12. O u
nico
inteiro menor ou igual a 12 com resto 7 por 9 e o proprio 7, da s(B) = 7.
Exemplo 10. Prove que 11n+2 + 122n+1 e divisvel por 133 para qualquer natural n.
Duas relacoes que podemos extrair dos n
umeros envolvidos s
ao: 14411 = 133 e 13312 =
121. Assim:
144 11 (mod 133),
122 11 (mod 133),
122n 11n
2n+1

12

2n+1

12

2n+1

12

(mod 133),

11 12

(mod 133),

11 (121) + 133 11n


n+2

11

(mod 133),

(mod 133).

Exemplo 11. Prove que n5 + 4n e divisvel por 5 para todo inteiro n


Inicialmente note que n5 + 4n = n(n4 + 4). Se n 0 (mod 5), nao ha o que fazer. Se
n 1 (mod 5), n4 +4 1+4 = 0 (mod 5). Finalmente, se n 2 (mod 5), n2 4 1
(mod 5) e consequentemente n4 + 4 1 + 4 = 0 (mod 5).
Exemplo 12. Seja n > 6 um inteiro positivo tal que n 1 e n + 1 s
ao primos. Mostre que
n2 (n2 + 16) e divisvel por 720. A recproca e verdadeira?

POT 2012 - Teoria dos N


umeros - Nvel 2 - Aula 5 - Samuel Feitosa

Veja que n e da forma 6k, pois n 1 e n + 1 s


ao primos maiores que 3, portanto da forma
6k 1 e 6k + 1, respectivamente. Logo,
n2 (n2 + 16) = 144(9k 4 + 4k 2 ).
Resta provar que 9k 4 + 4k 2 e um m
ultiplo de 5. Vamos analisar a igualdade acima m
odulo
5.
i) Se k 0, 2 ou 3 (mod 5), temos 9k 4 + 4k 2 0 (mod 5);
ii) Se k 1 (mod 5) n 1 (mod 5), temos n 1 0 (mod 5), um absurdo;
iii) Se k 4 (mod 5) n 4 (mod 5), temos n+1 0 (mod 5), novamente um absurdo.
Isso conclui a demonstracao. A recproca nao e verdadeira. Basta tomar, por exemplo,
n = 90.

Problemas Propostos
Problema 13. Determine o resto de 220 1 na divis
ao por 41.
Problema 14. Qual o resto de 12000 + 22000 + . . . + 20002000 na divis
ao por 7?
Problema 15. Qual o resto na divis
ao de 270 + 370 por 13?
Problema 16. Qual o resto de 3200 por 100?
Problema 17. (Est
onia 2000) Determine todos os possveis restos da divis
ao do quadrado
de um n
umero primo com o 120 por 120.
Problema 18. Qual o u
ltimo dgito de 777777 ?
Exemplo 19. Prove que 22225555 + 55552222 e divisvel por 7.
Problema 20. Prove que o n
umero n3 + 2n e divisvel por 3 para todo natural n.
Problema 21. Prove que n2 + 1 n
ao e divisvel por 3 para nenhum n inteiro.
Problema 22. Prove que n3 + 2 n
ao e divisvel por 9 para nenhum n inteiro.
Problema 23. Prove que p2 q 2 e divisvel por 24 se p e q s
ao primos maiores que 3.
Problema 24. Prove que se 2n + 1 e 3n + 1 s
ao ambos quadrados perfeitos, ent
ao n e
divisvel por 40.
Problema 25. Se n e mpar, prove que 7|22n+1 + 3n+2 .
Problema 26. Seja d(n) a soma dos dgitos de n. Suponha que n + d(n) + d(d(n)) = 1995.
Quais os possveis restos da divis
ao de n por 9?
Problema 27. Prove que n
ao existem inteiros positivos x1 , x2 , . . . , x14 tais que:
x41 + x42 + . . . + x414 = 1599.
4

POT 2012 - Teoria dos N


umeros - Nvel 2 - Aula 5 - Samuel Feitosa

Problema 28. Escreva uma u


nica congruencia que e equivalente ao par de congruencias
x 1 (mod 4) e x 2 (mod 3).
Problema 29. Prove que 2015 1 e divisvel por 11 31 61
Problema 30. (Alemanha 1997) Determine todos os primos p para os quais o sistema
p + 1 = 2x2
p2 + 1 = 2y 2
tem uma soluca
o nos inteiros x, y.
Problema 31. Mostre que se n divide um n
umero de Fibonacci ent
ao ele dividir
a uma
infinidade.

Dicas e Solu
c
oes
13. Veja que
25 = 32 9 (mod 41)
210 81 1 (mod 42)
220 1 (mod 41).
Assim, o resto procurado e zero.
14. Como i2000 (i + 7k)2000 (mod 7), podemos simplificar o problema calculando primeiramente o valor de:
12000 + 22000 + 32000 + 42000 + 52000 + 62000 + 72000

(mod 7).

Outra observacao importante que simplificara o calculo e perceber que 23 1 (mod 7).
Assim,
23k 1 (mod 7), 23k+1 2 (mod 7), e 23k+2 4 (mod 7).
Usando isso e o fato de que 2000 e par, temos:
12000 + 22000 + 32000 + 42000 + 52000 + 62000 + 72000
12000 + 22000 + (4)2000 + 42000 + (2)2000 + (1)2000 + 02000
1+4+2+2+4+1+0
0 (mod 7).
Dentre os primeiros 2000 naturais consecutivos, podemos formar 285 grupos de 7
n
umeros consecutivos cuja soma e m
ultipla de 7, em virtude da soma anterior. Os
cinco n
umeros restantes possuem como resto na divisao por 7 o n
umero:
19962000 + 19972000 + 19982000 + 19992000 + 20002000 1 + 4 + 2 + 2 + 4
6 (mod 7).
Assim, o resto da soma na divisao por 7 e 6.
5

POT 2012 - Teoria dos N


umeros - Nvel 2 - Aula 5 - Samuel Feitosa

15. Inicialmente e interessante buscarmos alguma relacao entre os n


umeros envolvidos no
problema. Como 13 = 4 + 9, podemos escrever:
9 4 (mod 13)
35

(4)35

70

9
70

+2

(mod 13)

(mod 13).

17. Use a fatoracao 120 = 3 5 23 e analise a congruencia m


odulo 3, 5 e 8 separadamente.
18. Se n nao e m
ultiplo de 3, sabemos que n2 1 (mod 3). Assim n2 + 2 0 (mod 3).
Se n e m
ultiplo de 3, n 0 (mod 3). Em qualquer caso, n(n2 + 2) 0 (mod 3).
19. Basta repetir a an
alise do problema anterior
20. Podemos montar uma tabela de congruencias na divisao por 9:
n 0 1 2 3 4 5 6 7 8
n3 0 1 8 0 1 8 0 1 8
Como nenhum cubo perfeito diexa resto 7 na divisao por 9, n3 + 2 6 0 (mod 9).
23. Proceda como no exemplo 7.
25.
22n+1 + 3n+2 4n 2 + 3n 9
(3)n 2 + 3n 2
0

(mod 7).

26. Seja r o resto na divisao por 9 de n. Pelo criterio de divisibilidade por 9, temos:
n + d(n) + d(d(n)) 3r 1995 (mod 9).
Assim, r 2 (mod 3) (Pela propriedade vi do teorema 2). Alem disso,
n 1995
d(n) 27 = d(1989)
d(d(n)) 10 = d(19).
Consequentemente, n 1995 d(n) d(d(n)) 1958. Basta procurarmos nos
conjunto {1958, 1959, . . . , 1995} os inteiros que deixam resto 2 por 3 e que satisfazem
a equacao do problema. Nesse conjunto, apena o inteiro 1967 cumpre essas condicoes.
27. Estudando a congruencia m
odulo 16, podemos mostrar que x4 0 ou 1 (mod 1)6.
Assim, a soma
x41 + x42 + . . . + x414
e congruente a um dos n
umeros do conjunto {0, 1, . . . , 14} m odulo 16 enquanto que
1599 15 (mod 16). Um absurdo.
6

28. x 5 (mod 12).


30. Suponha sem perda de generalidade que x, y 0. Como p + 1 e par, p 6= 2. Alem
disso,
2x2 1 2y 2 (mod p)
e consequentente, usando que p e mpar, x y (mod p). Como x < y < p, temos
p2 + 1 = 2(p x)2 = 2p2 4px + p + 1,
de modo que p = 4x 1, 2x2 = 4x. Podemos concluir que x e 0 ou 2 e que a u
nica
possibilidade para p e p = 7.
31. Em virtude da formula recursiva da sequencia de Fibonacci, e possvel mostrarmos
que os restos de seus termos na divisao por qualquer n
umero formam uma sequencia
peri
odica.

Refer
encias
[1] E. Carneiro, O. Campos and F. Paiva, Olimpadas Cearenses de Matem
atica 1981-2005
(Nveis J
unior e Senior), Ed. Realce, 2005.
[2] S. B. Feitosa, B. Holanda, Y. Lima and C. T. Magalhaes, Treinamento Cone Sul 2008.
Fortaleza, Ed. Realce, 2010.
[3] D. Fomin, A. Kirichenko, Leningrad Mathematical Olympiads 1987-1991, MathPro
Press, Westford, MA, 1994.
[4] D. Fomin, S. Genkin and I. Itenberg, Mathematical Circles, Mathematical Words, Vol.
7, American Mathematical Society, Boston, MA, 1966.
[5] I. Niven, H. S. Zuckerman, and H. L. Montgomery, An Introduction to the Theory of
Numbers.

Polos Olmpicos de Treinamento


Aula

Curso de Teoria dos Nmeros - Nvel 2

Prof. Samuel Feitosa

Congru
encias II
Na aula de hoje, aprenderemos um dos teoremas mais importantes do curso: o pequenoteorema de Fermat. Comecaremos relembrando um resultado da aula passada:
Lema 1. Se ka kb (mod m) e mdc(m, k) = 1, ent
ao a b (mod m).
Demonstraca
o. Como m | k(a b) e mdc(m, k) = 1, segue que m | a b.
Teorema 2. (Teorema de Fermat) Seja p um primo. Se p n
ao divide a ent
ao
ap1 1

(mod p).

Alem disso, para todo inteiro a, ap a (mod p)


Demonstraca
o. Considere o conjunto de inteiros B = {a, 2a, 3a, . . . , (p 1)a} onde a e um
inteiro satisfazendo mdc(a, p) = 1. Nenhum deles e divisvel por p e quaisquer dois deles
s
ao incongruentes m
odulo p, em virtude do lema anterior. Assim, o conjunto dos restos
dos elementos de B coincide com o conjunto dos restos nao nulos na divisao por p, a saber,
{1, 2, 3, . . . , p 1}. Portanto,
a 2a 3a . . . (p 1)a 1 2 3 . . . (p 1)
a

p1

(p 1)! (p 1)!

(mod p),

(mod p).

Podemos cancelar o termo (p 1)! em ambos os lados pois mdc((p 1)!, p) = 1, concluindo
assim a demonstracao do teorema.
n5 n3 7n
+
+
e um inteiro para todo inteiro n.
5
3
15
n5
n3
7n
3n5 + 5n3 + 7n
Primeiramente note que
+
+
=
. Como mdc(3, 5) = 1, basta
5
3
15
15
mostrarmos que o numerador e m
utiplo de 3 e 5. Pelo teorema de Fermat:
Exemplo 3. Prove que

3n5 + 5n3 + 7n 5n3 + 7n 5n + 7n = 12n 0 (mod 3),


3n5 + 5n3 + 7n 3n5 + 7n 3n + 7n = 10n 0 (mod 5).

POT 2012 - Teoria dos N


umeros - Nvel 2 - Aula 5 - Samuel Feitosa

Problema 4. Mostre que n7 n (mod 42), n N


Pelo teorema de Fermat,
n7 n
n

(mod 7)
3 2

(n ) n n2 n = n3 n
2 3

2 2

(mod 3)

(n ) n n n = (n ) n2 n

(mod 2)

Como 2, 3 e 7 s
ao primos entre si, n7 n (mod 2 3 7 = 42).
Exemplo 5. (Bulg
aria 95) Encontre o n
umero de inteiros n > 1 para os quais o n
umero
a25 a e divisvel por n para cada inteiro a.
Se n satisfaz o enunciado, p2 (p primo) nao pode divid-lo, pois p25 p nao e divisvel
por p2 . Assim, n e m
ultiplo de primos diferentes. Os fatores primos de n s
ao fatores de
225 2 = 2 32 5 7 13 17 241. Entretanto, n nao e divisvel por 17 e 241 pois 325 3
(mod 17) e 325 32 (mod 241). Seguindo o exemplo anterior, podemos usar o teorema de
Fermat para mostrar que a25 a (mod p) para p {2, 3, 5, 7, 13}. Portanto, n deve ser
igual a um dos divisores de 2 3 5 7 13 diferente de 1. A quantidade de tais divisores e
25 1 = 31.
Exemplo 6. Prove que para cada primo p, a diferenca
111 . . . 11222 . . . 22333 . . . 33 . . . 888 . . . 88999 . . . 99 123456789
(onde cada digito est
a escrito exatamente p vezes) e m
ultiplo de p.
Uma boa maneira de associar os n
umeros do problema com o teorema de Fermat e perceber
que:
10p 1
111
.
.
.
11
.
=
| {z }
9
p uns

Assim, podemos escrever o n


umero S = 111 . . . 11222 . . . 22333 . . . 33 . . . 888 . . . 88999 . . . 99
como:
10p 1
10p 1
10p 1
108p + 2
107p + . . . 9
9
9
9
9S = (10p 1) 108p + 2 (10p 1) 107p + . . . 9 (10p 1)
S =

Para p = 2 ou p = 3, o resultado do enunciado segue dos criterios de divisibilidade por 2 e


3. Podemos entao nos concentrar no caso p > 3. Nesse caso, e suficiente mostrarmos que
9(S 123456789) e divisvel por p pois mdc(p, 9) = 1. Pelo teorema de Fermat:
9S = (10p 1) 108p + 2 (10p 1) 107p + . . . 9 (10p 1)
(10 1) 108 + 2 (10 1) 107 + . . . 9 + (10 1)
9 123456789 (mod p).
2

(mod p)

POT 2012 - Teoria dos N


umeros - Nvel 2 - Aula 5 - Samuel Feitosa

Exemplo 7. Dado um primo p, prove que existem infinitos naturais n tais que p divide
2n n.
Se p = 2, n pode ser qualquer n
umero par. Suponha que p > 2. Considere (p 1)2k , pelo
teorema de Fermat temos:
2(p1)

2k

(2p1 )(p1)

2k1

1(p1)

2k1

= 1 (p 1)2k

(mod p).

Assim, para qualquer k, n = (p 1)2k satisfaz o problema.


Lema 8. Se mdc(a, m) = 1 ent
ao existe um inteiro x tal que
ax 1 (mod m).
Tal x e u
nico m
odulo m. Se mdc(a, m) > 1 ent
ao n
ao existe tal x.
Demonstraca
o. Pelo teorema de Bachet-Bezout, existem inteiros x e y tais que ax+my = 1.
Analisando essa congruencia m
odulo m, obtemos ax 1 (mod m). Se y e outro inteiro
que satisfaz a congruencia, temos ax ay (mod m). Pelo primeiro lema, x y (mod m).
Se d = mdc(a, m) > 1, nao podemos ter d | m e m | ax 1 pois d ax 1.
Teorema 9. (Teorema de Wilson) Se p e primo, ent
ao
(p 1)! 1 (mod p)
Demonstraca
o. Em virtude do lema anterior, para cada a {2, 3, . . . , p 2}, existe um
resto x {0, 1, 2, . . . , p 1} tal que ax 1 (mod p). Se x = 1 ou x = p 1, teramos a = 1
ou p 1. Alem disso, nao podemos ter a = x pois os u
nicos restos que satisfazem a2 1
(mod p) s
ao 1 e p 1 (Veja o problema 20). Com isso, podemos agrupar os n
umeros de
{2, 3, . . . , p 2} em pares onde o produto deixa resto 1 por p, o que nos permite concluir
que o produto de todos eles tambem deixa resto 1 por p. Logo,
(p 1)! 1 (p 1) 1 (mod p).
Exemplo 10. (Est
onia 2000) Prove que n
ao e possvel dividir qualquer conjunto de 18
inteiros consecutivos em dois conjuntos disjuntos A e B tais que o produtos dos elementos
de A seja igual ao produto dos elementos de B.
Suponha, por absurdo, que existam tais conjuntos. Considere o primo p = 19. Como o
produtos dos elementos de A e igual ao produtos dos elementos de B, se um dos conjuntos
contem um m
ultiplo de 19, o outro necessariamente tambem conter
a. Como entre 18
inteiros consecutivos nao existem dois m
ultiplos de 19, nenhum dos conjuntos do problema
contem tais n
umeros. Seja x o resto na divisao por 19 dos produtos dos elementos de A.
Calculemos entao o resto na divisao por 19 do produto de todos os 18 inteiros consecutivos:
x x n(n + 1)(n + 2)(n + 3) . . . (n + 17)
1 2 3 . . . 18
1 (mod 19)(Pelo teorema de Wilson).
Como x2 1 (mod 19), x18 (1)9 1 (mod 1)9. Isso contraria o teorema de Fermat
e obtemos um absurdo.
3

POT 2012 - Teoria dos N


umeros - Nvel 2 - Aula 5 - Samuel Feitosa

Definic
ao 11. Um conjunto S e chamado de sistema completo de resduos m
odulo n, denotado abreviadamente por scr, se para cada 0 i n 1, existe um elemento de s S
tal que i s (mod n). Para qualquer a, o conjunto {a, a + 1, a + 2, . . . , a + (n 1)} e um
exemplo de scr.
Exemplo 12. Se mdc(m, s) = 1, mostre que {t, t + s, t + 2s, . . . t + (m 1)s} e um scr.
Pelo primeiro lema, se t + is t + js (mod m), temos is js (mod m) e i j (mod m).
Como i, j {0, 1, . . . , m 1}, i = j. Isso nos diz que temos m inteiros que deixam restos
distintos na divisao por m. Como existem exatamente m restos na divisao por m, o conjunto
e um scr.
Exemplo 13. Seja m um inteiro positivo par. Suponha que {a1 , a1 , . . . , am } e {b1 , b2 , . . . , bm }
s
ao dois sistemas completos de resduso m
odulo m. Prove que
S = {a1 + b1 , a2 + b2 , . . . , am + bm }
n
ao e um sistema completo de resduos.
Suponha que S seja um scr, entao:
1 + 2 + . . . + m (a1 + b1 ) + (a2 + b2 ) + . . . + (an + bn )

(mod m)

(a1 + a2 + . . . + an ) + (b1 + b2 + . . . + bn )
2(1 + 2 + . . . + n)
2(1 + 2 + . . . + m)
m(m + 1)
m+1
, ou seja,
e inteiro. Um absurdo pois m e par.
2
2
Exemplo 14. (Pol
onia 1997) Prove que a sequencia an definida por a1 = 1 e

Isso implica que m |

an = an1 + a n 
2
contem infinitos termos divisveis por 7.
Uma maneira natural para mostrarmos que existem infinitos inteiros m
ultiplos de 7 na
sequencia e verificar que o aparecimento de um m
ultiplo de 7 acarreta o aparecimento de
outro m
ultiplo na sequencia com um ndice maior. Suponha que ak e m
ultiplo de 7. Seja
a2k1 = s. Entao:
a2k1 = s
a2k = s + ak s
a2k+1 = a2k + ak s

(mod 7)
(mod 7)

POT 2012 - Teoria dos N


umeros - Nvel 2 - Aula 5 - Samuel Feitosa

Ou seja, o aparecimento de um inteiro m


ultiplo de 7 implica no aparecimento de 3 inteiros
com o mesmo resto por 7. Exploremos essa ideia mais uma vez.
a4k3 = t
a4k2 t + a2k1 t + s

(mod 7)

a4k1 t + s + a2k1 t + 2s

(mod 7)

a4k t + 2s + a2k t + 3s

(mod 7)

a4k+1 t + 3s + a2k t + 4s

(mod 7)

a4k+2 t + 4s + a2k+1 t + 5s

(mod 7)

a4k+3 t + 5s + a2k+2 t + 6s

(mod 7)

Se s e m
ultiplo de 7, ja teremos conseguido outro m
ultiplo de 7 na sequencia. Em caso
contrario, o conjunto {t, t + s, t + 2s, . . . , t + 6s} e um scr e conter
a um m
ultiplo de 7.
Exemplo 15. Sejam x, y inteiros. Prove que 3x2 + 4y 2 e 4x2 + 3y 2 n
ao podem ser ambos
quadrados perfeitos.
Comecemos com um lema bastante u
til:
Lema 16. Seja p um n
umero primo da forma 4k + 3. Ent
ao
p | m2 + n2 p | m e p | n.
Facamos inicialmente a primeira implicacao. Se p m, entao mp1 1 (mod p), e da
temos as equivalencias m
odulo p
n2 m2

(nmp2 )2 (mp1 )2
1
p1
p2
p1
(nm )
(1) 2
(1)2k+1
1,
o que contraria o teorema de Fermat. Assim, p | m e p | n.
A recproca e
obvia. Voltando ao problema, suponha que existam w, z inteiros positivos
tais que
3x2 + 4y 2 = w2
2

4x + 3y

= z .

Entao 7x2 + 7y 2 = w2 + z 2 (). Afirmamos que a equacao () nao possui solucao. Para isso,
seja S o conjunto formado pelas solucoes inteiras (x, y, w, z) de (), e tome (a, b, c, d) S

POT 2012 - Teoria dos N


umeros - Nvel 2 - Aula 5 - Samuel Feitosa

com c2 + d2 mnimo. Pelo lema, temos que 7|c e 7|d, e da c = 7c e d = 7d . Mas entao
a2 + b2 = 7c2 + 7d2 (c , d , a, b) S, com
a2 + b2 < 7(a2 + b2 ) = c2 + d2 ,
o que contraria a minimalidade de (a, b, c, d).

Problemas Propostos
Problema 17. Prove que se p e primo ent
ao
a p bp

(mod p) ap bp

(mod p2 )

Problema 18. Encontre os restos da divis


oes de:
a) 3003000 1 por 1001
b) 7120 1 por 143
Problema 19. Encontre o resto de 111
. . . 11} por p, onde p e um primo maior que 5.
| {z
p1 uns

Problema 20. Prove que se n e mpar, ent


ao n5 n (mod 240).
Problema 21. Sejam p e q primos distintos. Mostre que
i) (a + b)p ap + bp (mod p)
ii) pq + q p p + q (mod pq)
 q

p + pq
iii)
e par se p, q 6= 2.
pq

Problema 22. Mostre que se p e primo e a2 b2 (mod p), ent


ao a b (mod p).
Problema 23. Encontre os u
ltimos tres dgitos de 79999
Problema 24. Prove que 2015 1 e divisvel por 11 31 61
Problema 25. Sejam {a1 , a2 , ..., a101 } e {b1 , b2 , ..., b101 } sistemas completos de resduos
m
odulo 101. Pode {a1 b1 , a2 b2 , ..., a101 b101 } ser um sistema completo de resduos m
odulo
101?
Problema 26. (Balc
anica 2003) Existe um conjunto B de 4004 inteiros positivos tal que,
para cada subconjunto A de B com 2003 elementos, a soma dos elementos em A n
ao e
divisvel por 2003?
Problema 27. Para um inteiro mpar n > 1, seja S o conjunto de inteiros x,1 x n,
tal que ambos x e x + 1 s
ao relativamente primos com n. Mostre que o produto de todos
os elementos de S deixa resto 1 na divis
ao por n.
6

POT 2012 - Teoria dos N


umeros - Nvel 2 - Aula 5 - Samuel Feitosa

Problema 28. Sejam n um inteiro positivo maior que 1 e p um primo positivo tal que n
divide p 1 e p divide n3 1. Mostre que 4p 3 e um quadrado perfeito.

Dicas e Solu
c
oes

17. Pelo teorema de Fermat, a ap bp b (mod p). Assim,


ap1 + ap2 b + . . . + abp2 + bp1 ap1 + ap1 + . . . + ap1
pap1
0 (mod p)
Como a b 0 (mod p), temos:
ap bp = (a b)(ap1 + ap2 b + . . . + abp2 + bp1 ) 0 (mod p2 )
19. Veja que:
111
. . . 11} =
| {z
p1 uns

999 . . . 99
9
10p1 1
9

Pelo teorema de Fermat, o numerador 10p1 1 e divisvel por p visto que p 6= 5.


p1
Alem disso, usando que p 6= 2 e 3, segue que 10 9 1 tambem e m
ultiplo de p.
20. Proceda como no exemplo 20.
21. i)Pelo teorema de Fermat:
(a + b)p a + b
a p + bp

(mod p).

ii) Pelo teorema de Fermat,


pq + q p 0 + q p + q
q

p +q

p+0p+q

(mod p)
(mod q)

22. Veja que (a b)(a + b) 0 (mod p) e assim a b 0 (mod p) ou a + b 0 (mod p).


25. Suponha, por abusurdo, que seja possvel. Sejam ai e bj tais que ai bj 0
(mod 101). Se i 6= j, o conjunto {a1 b1 , a2 b2 , ..., a101 b101 } teria dois inteiros com resto

POT 2012 - Teoria dos N


umeros - Nvel 2 - Aula 5 - Samuel Feitosa

0 na divisao por p e nao poderia ser um scr. Suponha sem perda de generalidade que
i = j = 101, entao:
100! (a1 b1 )(a2 b2 ) . . . (a100 b100 )
(a1 a2 . . . a100 )(b1 b2 . . . b100 )
(100!)(100!)
(100!)2

(mod 101)

Assim, 100! 1 (mod 101). Isso contradiz o teorema de Wilson.


26. Sim. Um exemplo de tal conjunto e a uni
ao de um conjunto de 2002 inteiros positivos
que deixem resto 0 com outro conjunto composto por 2002 inteiros que deixem resto
1 por 2003.

Polos Olmpicos de Treinamento


Curso de Teoria dos Nmeros - Nvel 2

Aula

Prof. Samuel Feitosa

Aula de Revis
ao e Aprofundamento
recomend
Observac
ao 1. E
avel que o professor instigue seus alunos a pensarem nos problemas abaixo antes de resolve-los na aula.
Exemplo 2. (ASHME 1990) Para quantos inteiros N entre 1 e 1990 a fraca
o
e irredutvel?

N2 + 7
n
ao
N +4

Seja d = (N + 4, N 2 + 7). Um boa estrategia e procurar um m


ultiplo de N + 4 proximo de
N 2 + 7 pois assim conseguiremos estimar d. Usando a diferenca de quadrados, d | N 2 16 e
consequentemente d | N 2 +7(N 2 16) = 23. Como 23 e primo, a fracao nao sera irredutvel
apenas quando d = 23. Para isso acontecer, basta que 23 | N +4 pois N 2 +7 = N 2 16+23.
O maior m
ultiplo de 23 menor que 1990 e 1978 = 23 86 e 1990 + 4 < 23 87. Sendo assim,
a quantidade de inteiros procurada e 86.
Exemplo 3. Dados os primos p e q satisfazendo:
q | p2 + 1 e p | q 2 1.
Prove que o n
umero p + q + 1 e composto.
Como p | q 2 1 = (q + 1)(q 1), temos que p | q + 1 ou p | q 1. No primeiro caso, p + q + 1
e um m
ultiplo de p. No segundo caso, podemos escrever q 1 = pk para algum natural k.
Usando que q | p2 + 1, conclumos que q | p2 + 1 (pk + 1) = p(p k). Como p e q nao
podem ser primos iguais, q | p k. Temos tres casos a considerar:
1. p > k. Entao:
p k kp + 1,

(p + 1)(1 k) 2

2. p < k. Entao:
k p kp + 1,

(k + 1)(1 p) 2

3. p = k e q = p2 +1. Como o u
nico primo par e 2, segue que p = 2, q = 5 e p+q +1 = 8.
Os dois primeiros casos conduzem a um absurdo. Logo, ou p + q + 1 e m
ultiplo de p ou e
igual `a 8.
Exemplo 4. (AIME 1985) Os n
umeros da sequencia
101, 104, 109, 116, . . .
s
ao da forma an = 100 + n2 , onde n = 1, 2, 3, . . . Para cada n, seja dn o m
aximo divisor
comum de an e an+1 . Encontre o valor m
aximo de dn quando n varia sobre todo o conjunto
dos inteiros positivos.
Uma boa estrategia e buscar alguma fatoracao que nos permita identificar fatores comuns
entre os termos da sequencia. Um termo generico da sequencia possui a forma an =
k + n2 . Sendo assim, a2k = k(4k + 1),a2k+1 = (k + 1)(4k + 1) e consequentemente
mdc(a2k , a2k+1 ) = 4k + 1. Nosso proximo passo sera mostrar que realmente esse e o valor
m
aximo. Considere dois termos genericos a = an = k + n2 , b = an+1 = k + (n + 1)2 e seja
d = mdc(a, b). Usando que d | b a = 2n + 1, obtemos d | (2n + 1)(b a) = 4n2 1. Como
d | 4a = 4n2 + 4k, segue que d | 4n2 + 4k (4n2 1) = 4k + 1. Assim, 4k + 1 e realmente
o maior valor possvel entre os termos da sequencia dn .
Exemplo 5. Prove que para qualquer inteiro n > 1, o n
umero n5 + n4 + 1 n
ao e um n
umero
primo.
Considere a fatoracao:
n5 + n4 + 1 = n5 + n4 + n3 n3 n2 n + n2 + n + 1

= n3 (n2 + n + 1) n(n2 + n + 1) + (n2 + n + 1)

= (n2 + n + 1)(n3 n + 1)

Como n > 1, n3 n + 1 > 1 e obtemos assim o produto de dois inteiros maiores que 1.

Exemplo 6. (Olimpada Grega) Encontre todos os inteiros n para os quais 54 + 55 + 5n e


um quadrado perfeito.
Como 54 + 55 = 2500, queremos encontrar m e n tais que:
5n = m2 2500 = (m 50)(m + 50).
Isto implica que m + 50 = 5j e m 50 = 5i , com i < j. Assim,
100 = 5i (5ji 1).
Usando a fatoracao em primos de 100, encontramos que i = 2 e j i = 1. Portanto, m = 75
e n = 5.
2

Exemplo 7. (Irlanda) Sejam p um n


umero primo, a e n e inteiros positivos. Prove que se
2p + 3p = an ,
ent
ao n = 1.
Se p = 2, claramente a = 13 e n = 1. Se p > 2, p e mpar e 5 | 2p + 3p . Consequentemente
5 divide a. Se fosse n > 1, 25 | an e teramos:
an
5
2p + 3p

5
= 2p1 2p2 3 + . . . + 2 3p2 + 3p1

2p1 + 2p1 + . . . + 2p1


p2p1

(mod 5)

Au
nica possbilidade e termos p = 5. Entretanto, 25 + 35 nao e uma potencia perfeita nao
trivial. Logo, n = 1.
Exemplo 8. Um inteiro n > 1 tem a seguinte propriedade: para todo divisor positivo d de
n, d + 1 e um divisor de n + 1. Prove que n e primo.
n
Seja p o menor fator primo de n e seja d = . Entao,
p
p(n + 1)
n+1
np + p
=
=
n+p
p(d + 1)
d+1
e um n
umero inteiro. Como n + p tambem divide p(n + p), podemos concluir que n + p |
(np + p2 ) (np + p) = p2 p. Em particular,
n + p p2 p

n p2 2p

n p2 2p + 1 = (p 1)2

n < p2
d < p.

Em virtude da minimalidade de p, d nao possui fatores primos e consequentemente n = p.


Exemplo 9. (Olimpada Russa) Mostre que qualquer natural pode ser escrito como a diferenca de dois n
umeros naturais tendo o mesmo n
umero de fatores primos.
Se n e par, podemos escreve-lo como 2n n e e facil verificar que 2n e n possuem o
mesmo n
umero de fatores primos. Seja d o menor primo mpar que nao divide n. Escreva
n = dn (d 1)n. O termo dn contem exatamente um primo a mais que n. Pela escolha
de d, todos os outros fatores primos diferentes 2 do n
umero d 1 s
ao divisores de n e assim
(d 1)n tambem contem extamente um primo a mais que n, a saber, o primo 2.
3

Exemplo 10. Os n
umeros naturais a e b s
ao tais que
a+1 b+1
+
b
a
e um n
umero inteiro. Mostre que o m
aximo divisor comun de a e b n
ao e maior que

a + b.

Seja d = mdc(a, b), com a = md e b = nd. Entao:


m2 d + m + n2 d + n
md + 1 nd + 1
+
=
nd
md
mnd
e um inteiro. Em particular, d | m2 d + m + n2 d + n e consequentemente d | m + n. Da,

d m+n

d
m+n
p
d
d(m + n)

=
a+b

Exemplo 11. Encontre todos os conjuntos A N de pelo menos dois elementos tais que
x+y
A.
mdc(x, y)

x, y A =

Vamos dividir o problema em dois casos: Primeiro caso: Se 1 6 A.


Mostremos que nesse caso devemos ter A = {2, 3, . . .}. Facamos isso seguindo as seguintes
afirmacoes:
1. 2 A.
Para ver isso, basta tomar dois elementos iguais.
2. Existe elemento mpar em A.
Suponha, por absurdo, que nao existe elemento mpar em A. Seja 2k o menor elemento
de par de A maior que 2. Logo,
k+1=

2k + 2
A.
mdc(2k, 2)

Como k + 1 < 2k, k + 1 e mpar. Absurdo! Note que todos os mpares maiores que
k + 1 pertencem a A. Para tal, basta escolhermos 2 e k + 1 para obtermos k + 3 e
aplicar isso sucessivamente.
3. 3 A.

Tome 2l 1 > 2k + 1. Assim,


(2l 1)(2k + 1), (2k + 1) A
2l 1, 2l + 1 A
2l, 4l A

(2l 1)(2k + 1) + (2k + 1)


= 2l A;
2k + 1
2l 1 + 2l + 1
= 4l A;
mdc(2l 1, 2l + 1)
4l + 2l
=3A
mdc(4l, 2l)

Isso mostra que todos os mpares maiores ou iguais a 3 estao presentes. Para isso,
basta tomar cada mpar e o 2.
4. Todos os n
umeros pares estao em A.
Para isso, tome 2k 1 e (2k 1)2 :
(2k 1)2 + (2k 1)
= 2k A,
2k 1
para todo o k 2.
Segundo caso: Se 1 A.
Nesse caso, afirmamos que A = N. Veja que:
1A
xA

1+1
=2A
1
x+1
=x+1A
mdc(1, x)

Sendo assim, repetindo esse processo, seguir


a por inducao que A = N.
Exemplo 12. (Olimpada Matem
atica Argentina) Sejam p1 , p2 , . . . , pn n
umeros primos.
Bruno deve escolher n + 1 inteiros positivos que utilizem apenas estes primos em sua decomposica
o. Bernardo deve escolher alguns desses n
umeros de modo que o produto deles
seja um quadrado perfeito. Determine se e possvel, para algum n, que Bruno escolha seus
n + 1 n
umeros de maneira que Bernardo n
ao consiga cumprir seu objetivo.
Vamos mostrar que Bernardo sempre consegue cumprir seu objetivo. Para decidirmos se
um n
umero natural e quadrado perfeito, basta analisarmos a paridade dos expoentes de seus
mn
1 m2
fatores primos. Para cada n
umero pm
1 p2 . . . pn escolhido por Bruno, associe a n-upla de
zeros e uns, (r1 , r2 , . . . , rn ), onde cada ri e o resto na divisao por 2 de mi . A multiplicacao
de dois inteiros se traduz na soma m
odulo 2 de tais uplas, i.e., a n-upla associada ao
m n p l1 p l2 . . . pln
1 m2
e
igual
`a (r1 + q1 (mod 2), r2 + q2 (mod 2), . . . , rn + qn
.
.
.
p
p
produto pm
n
n
1 2
2
1
(mod 2)) onde ri e qi s
ao os restos na divisao por 2 de mi e li , respectivamente. Nosso
problema pergunta se e possivel Bernardo encontrar algumas uplas que somadas deem a
upla (0, 0, 0, . . . , 0). Como temos n + 1 uplas, podemos formar 2n+1 1 > 2n somas de
subconjuntos nao vazios de uplas. Cada soma corresponde a uma nova upla, como existem
5

apenas 2n tipos de uplas distintas, alguma delas se repetira dentre as somas (pelo princpio
da casa dos pombos). Suponha que para dois conjuntos de uplas A e B tenhamos a mesma
upla associada como soma, entao a soma dos elementos de A e B que nao pertencem `a
A B corresponde `
a upla (0, 0, . . . , 0).

Problemas Propostos

Problema 13. Sejam p > 2 um primo mpar e n um inteiro positivo. Prove que p divide
1p + 2p + . . . + (p 1)p .
n

Problema 14. (Olimpada Romena) Sejam a, b, c, d inteiros n


ao nulos com a 6= c e tais que
a
a2 + b2
= 2
.
c
c + b2
Prove que a2 + b2 + c2 n
ao pode ser um n
umero primo.
Problema 15. Encontre todos os n tais que n! e um quadrado perfeito.
Problema 16. (Hungria) O produto de alguns primos e dez vezes maior que a sua soma.
Quais s
ao esses primos?(n
ao necessariamnete distintos).
Problema 17. Qual o m
aximo divisor comum entre dois n
umeros de Fibonacci consecutivos?
Observaca
o: Os n
umeros de Fibonacci s
ao os n
umeros da sequencia definida por F1 = F2 =
1 e Fn+1 = Fn + Fn1 .
Problema 18. Mostre que a soma de dois primos consecutivos nunca e o dobro de um
primo.
Problema 19. (Israel) Se Sn e a soma dos n primeiros n
umeros primos, prove que h
a ao
menos um quadrado perfeito entre Sn e Sn+1
Problema 20. (Olimpada Balc
anica) Prove que, para todo natural dado n, existe um natural m > n tal que a representaca
o decimal de 5m e obtida da representaca
o decimal de
n
5 pelo acrescimo de algarismos `
a esquerda de 5n .
Problema 21. (Inglaterra 1995)
a) Encontre o primeiro inteiro positivo cujo quadrado termina em tres quatros.
b) Encontre todos os inteiros positivos cujo quadrado termina em tres quatros.
c) Mostre que nenhum quadrado perfeito termina em quatro quatros.

Exemplo 22. (Olimpada Indiana) Seja n um inteiro positivo tal que n e um divisor da
soma
1 + (1n1 + 2n1 + . . . + (n 1)n1 ).
Prove que n n
ao e divisvel por qualquer quadrado maior que 1.
Problema 23. Seja S um conjunto de primos tal que a, b S (a e b n
ao precisam ser
distintos) implicam ab + 4 S. Mostre que S tem que ser vazio

Dicas e Solu
c
oes

1. Em breve!

Polos Olmpicos de Treinamento


Curso de Teoria dos Nmeros - Nvel 2

Aula

Prof. Samuel Feitosa

Equaco
es Diofantinas I

Exemplo 1. Em Gugul
andia, o jogo de basquete e jogado com regras diferentes. Existem
possvel um time fazer 39
apenas dois tipo de pontuaco
es para as cestas: 5 e 11 pontos. E
pontos em uma partida?
Sejam x e y os n
umeros de cestas de 5 e 11 pontos, respectivamente. O problema se resume
em descobrirmos se existem inteiros nao negativos x e y tais que 5x + 11y = 39. Ao inves
de testarmos os valores de x e y, somemos 11 + 5 em ambos os lados da equacao:
5(x + 1) + 11(y + 1) = 55.
Como 5 | 55 e 5 | 5(x + 1), segue que 5 | 11(y + 1) e, com mais razao, 5 | y + 1 pois
mdc(5, 11) = 1. Do mesmo modo, 11 | x + 1. Assim,
55 = 5(x + 1) + 11(y + 1) 5 11 + 11 5 = 110,
pois x + 1, y + 1 1. Obtemos uma contradicao.
Exemplo 2. Qual o menor inteiro positivo m para o qual todo n
umero maior que m pode
ser obtido como pontuaca
o no jogo de basquete mencionado anteriormente?
Como ja sabemos que 39 nao e possvel, e natural comecarmos procurando os n
umeros
maiores que 39 que nao podem ser pontuacoes. Veja que:
40 = 5 8 + 11 0
41 = 5 6 + 11 1
42 = 5 4 + 11 2
43 = 5 2 + 11 3
44 = 5 0 + 11 4

Ao somarmos 5 a cada uma dessas representacoes, obteremos representacoes para os proximos


5 n
umeros. Repetindo esse argumento, poderemos escrever qualquer n
umero maior que 39
na forma 5x + 11y com x e y inteiros nao negativos. Conclumos assim que m = 39. Poderamos mostrar que todo n
umero maior que 44 e da forma 5x + 11y com x e y inteiros
nao negativos de outro modo. Se n > 44, considere o conjunto:
n 11 0, n 11 1, n 11 2, n 11 3, n 11 4.
Como mdc(11, 5) = 1, o conjunto anterior e um sistema completo de restos m
odulo 5 e
consequentemente existe y {0, 1, 2, 3, 4} tal que
n 11 y = 5x
Como n > 44, segue que x > 0.
Exemplo 3. Quais e quantos s
ao os inteiros positivos n que n
ao podem ser obtidos como
pontuaca
o nesse jogo de basquete?
Precisaremos relembrar um teorema da aula 03:
Teorema 4. (Bachet-B`ezout) Se d = mdc(a, b), ent
ao existem inteiros x e y tais que
ax + by = d.
A primeira observacao que fazemos e que uma vez encontrados inteiros x e y, qualquer
m
ultiplo de d pode ser representado como uma combinacao linear de a e b:
a(kx) + b(ky) = kd.
Isso e particularmente interessante quando mdc(a, b) = 1, onde obtemos que qualquer inteiro e uma combinacao linear de a e b. Veja que isso nao entra em conflito com os exemplos
anteriores pois os inteiros x e y mencionados no teorema podem ser negativos.
A proxima propopsicao conter
a o que procuramos:
Proposic
ao 5. Todo inteiro positivo k pode ser escrito(de modo u
nico) de uma e, somente
uma, das seguintes formas:
11y 5x, ou 11y + 5x, com 0 y < 5 e x 0
Pelo teorema de Bachet-B`ezout, existem m e n tais que 5m + 11n = 1. Sejam q e r o
quociente e resto da divisao de kn por 5, i.e., kn = 5q + r, 0 r < 5. Assim,
k = 5(km) + 11(kn)
= 5(km) + 11(5q + r)
= 5(km + 11q) + 11r.
2

Basta fazer x = km + 11q e r = y.


Para ver a unicidade, suponha que 11m 5n = 11a 5b com 0 m, a < 5. Entao
11(m a) = 5(b n). Usando que mdc(11, 5) = 1, segue que 5 | m a. A u
nica opcao
e termos m = a pois o conjunto {0, 1, 2, 3, 4} e um scr. Consequentemente 5n = 5b e
n = b.
Sendo assim, os elementos do conjunto
B(5, 11) = {11y 5x Z+ ; 0 y < 5 e x > 0}
constituem o conjunto das pontuacoes que nao podem ser obtidas. Seus elementos s
ao:
y = 1 11y 5x = 1, 6
y = 2 11y 5x = 2, 7, 12, 17
y = 3 11y 5x = 3, 8, 13, 18, 23, 28
y = 4 11y 5x = 4, 9, 14, 19, 24, 29, 34, 39
A quantidade de tais inteiros e
20 =

(5 1) (11 1)

.
2
2

Vale o resultado geral:


Proposic
ao 6. Dados os inteiros positivos a e b com mdc(a, b) = 1, existem exatamente
(a 1) (b 1)

2
2
n
umeros inteiros n
ao negativos que n
ao s
ao da forma ax + by com x, y 0.
Provaremos tal resultado em uma aula futura fazendo o uso da funcao parte inteira.
Exemplo 7. Suponha agora que as pontuaco
es das cestas do basquete de Gugul
andia tenham
mudado para a e b pontos com 0 < a < b. Sabendo que existem exatamente 35 valores
impossveis de pontuaco
es e que um desses valores e 58, encontre a e b.
Perceba que devemos ter mdc(a, b) = 1 pois caso contrario qualquer valor que nao fosse
m
ultiplo de mdc(a, b) nao seria uma pontuacao possvel e sabemos que existe apenas um
n
umero finito de tais valores. Em virtude da proposicao anterior, (a1)(b1) = 235 = 70.
Analisemos os possveis pares de divisores de 70 tendo em mente que a < b:
(a 1)(b 1) = 1 70 (a, b) = (2, 71)
(a 1)(b 1) = 2 35 (a, b) = (3, 36)
(a 1)(b 1) = 5 14 (a, b) = (6, 15)
(a 1)(b 1) = 7 10 (a, b) = (8, 11)

N
ao podemos ter (a, b) = (2, 71) pois 58 = 2 29. Excluindo os outros dois casos em que
mdc(a, b) 6= 1, temos a = 8 e b = 11.
A equacao ax + by = c e um exemplo de uma equancao diofantina, i.e., uma equacao
em que buscamos valores inteiros para as variaveis. Tais equacoes recebem esse nome em
homenagem ao matem
atico grego Diofanto.
Exemplo 8. Determine todas as soluco
es inteiras da equaca
o 2x + 3y = 5.
Por paridade, 3y e mpar, donde y = 2k + 1 para algum inteiro k. Da,
x=

5 3(2k + 1)
= 1 3k,
2

e consequentemente todas as solucoes da equacao s


ao da forma (x, y) = (1 3k, 2k + 1).
Exemplo 9. Determine todas as soluco
es inteiras da equaca
o 5x + 3y = 7.
Analisando agora m
odulo 3, 5x 7 1 (mod 3). Essa condicao impoe restricoes sobre o
resto de x na divisao por 3. Dentre os possveis restos na divisao por 3, a saber {0, 1, 2}, o
u
nico que satisfaz tal congruencia e o resto 2. Sendo assim, x e da forma 3k + 2 e
y=

7 5(3k + 2)
= 1 5k,
3

consequentemente, todas as solucoes da equacao s


ao da forma (x, y) = (3k + 2, 1 5k).
Notemos que para a solucao da congruencia x = 2, obtemos a solucao (x, y) = (2, 1) da
equacao. Baseado nesses exemplos, e natural imaginarmos que conhecendo uma solucao da
congruencia consigamos descrever todas as outras.
Teorema 10. A equaca
o ax + by = c, onde a, b, c s
ao inteiros, tem uma soluca
o em inteiros
(x, y) se, e somente se, d = mdc(a, b) divide c. Nesse caso, se (x0 , y0 ) e uma soluca
o, ent
ao
os pares


ak
bk
, kZ
(xk , yk ) = x0 + , y0
d
d
s
ao todas as soluco
es inteiras da equaca
o.
Dada a discuss
ao anterior, resta apenas encontrarmos a forma das solucoes. Se (x, y) e
outra solucao, podemos escrever:
ax + by = ax0 + by0
a(x x0 ) = b(y0 y)
a
b
(x x0 ) =
(y0 y)
d
d
Como mdc(a/d, b/d) = 1, temos b/d | x x0 e assim podemos escrever x = x0 + bk/d.
Substituindo na equacao original obtemos y = y0 ak/d.
4

Exemplo 11. Encontre todas as soluco


es inteiras da equaca
o 21x + 48y = 6
O sitema e equivalente `
a 7x + 16y = 2. Uma solucao e (x, y) = (2, 1). Pelo teorema
anterior, todas as solucoes s
ao da forma:
(xk , yk ) = (2 + 16k, 1 7k).
Exemplo 12. Resolva nos inteiros a equaca
o 2x + 3y + 5z = 11
Podemos transformar esse problema isolando qualquer uma das variaveis no problema que
ja sabemos resolver. Por exemplo, podemos resolver 2x + 3y = 11 5z. Supondo z fixo,
podemos encontrar a solucao particular (x, y) = (4 z, 1 z). Assim, todas as solucoes
s
ao da forma:
(x, y) = (4 z + 3k, 1 z 2k),
ou seja, as solucoes da equacao original s
ao da forma (x, y, z) = (4 z + 3k, 1 z 2k, z)
com k e z inteiros.
Vamos estudar agora alguns outros exemplos de equacoes diofantinas nao lineares:
Exemplo 13. Prove que a equaca
o 2n + 1 = q 3 n
ao admite soluco
es (n, q) em inteiros
positivos.
facil ver que a equacao nao admite solucoes se n = 1, 2, 3. Assim, podemos supor que
E
n > 3. Fatorando, temos:
(q 1)(q 2 + q + 1) = 2n ,
e consequentemente q = 2 ou q = 2k + 1, para algum k N. Claramente, q = 2 nao produz
solucao. Entao q = 2k + 1 e q 3 1 = 8k 3 + 12k 2 + 6k e uma potencia de 2, maior ou igual
a 16. Entretanto:
8k 3 + 12k 2 + 6k = 2k(4k 2 + 6k + 3),
nao e uma potencia de 2, pois 4k 2 + 6k + 3 e mpar. Assim, a equacao 2n + 1 = q 3 nao
admite solucoes inteiras positivas.

xz 2yt = 3
Exemplo 14. (URSS 1991) Encontre todas as soluco
es inteiras do sistema
xt + yz = 1.
Uma boa estrategia sera aplicar alguma manipulacao algebrica, como somar as equacoes,
multiplic
a-las, somar um fator de correcao, entre outras para obtermos alguma fatoracao
envolvendo esses n
umeros. Nesse problema, vamos elevar ambas as equacoes ao quadrado.
 2 2
x z 4xyzt + 4y 2 t2 = 9
x2 t2 + 2xytz + y 2 z 2 = 1.
Multiplicando a segunda por dois e somando com a primeira, temos:
x2 (z 2 + 2t2 ) + 2y 2 (z 2 + 2t2 ) = 11
(x2 + 2y 2 )(z 2 + 2t2 ) = 11.
5

Como cada uma das parcelas acima e um inteiro nao-negativo, temos dois casos:
 2
x + 2y 2 = 11
(x, y, z, t) = (3, 1, 1, 0).
z 2 + 2t2 = 1
ou


x2
z2

2y 2

+
+ 2t2

= 1
(x, y, z, t) = (1, 0, 3, 1).
= 11

Logo, as u
nicas solucoes possveis s
ao as quadruplas (1, 0, 3, 1) e (3, 1, 1, 0).

Problemas Propostos

Problema 15. Encontre todas as soluco


es de 999x 49y = 5000.
Problema 16. Encontre todos os inteiros x e y tais que 147x + 258y = 369.
Problema 17. Encontre todas as soluco
es inteiras de 2x + 3y + 4z = 5.
Problema 18. Encontre todas as soluco
es inteiras do sistema de equaco
es:
20x + 44y + 50z = 10
17x + 13y + 11z = 19.
Problema 19. (Torneio das Cidades 1997) Sejam a,b inteiros positivos tais que a2 + b2 e
divisvel por ab. Mostre que a = b.
Problema 20. Encontre uma condica
o necess
aria e suficiente para que
x + b1 y + c 1 z = d 1 e x + b2 y + c 2 z = d 2
tenham pelo menos uma soluca
o simultanea em inteiros x, y, z, assumindo que os coeficientes s
ao inteiros com b1 6= b2 .
Problema 21. (AMC 1989) Seja n um inteiro positivo. Se a equaca
o 2x + 2y + n = 28 tem
28 soluco
es em inteiros positivos x, y e z, determine os possveis valores de n.

Polos Olmpicos de Treinamento


Curso de Teoria dos Nmeros - Nvel 2

Aula

Prof. Samuel Feitosa

O Teorema de Euler
Nesta aula, obteremos uma generalizacao do teorema de Fermat.
Definic
ao 1. Dado n N, denotaremos o n
umero de naturais menores ou iguais a n e
relativamente primos com n por (n).
Segue imediatamente da definicao de (n) que (1) = 1, (2) = 1, (3) = 2, (5) = 4 e
(6) = 2. Se p e primo, (p) = p 1.
Lema 2. Se p e um n
umero primo e k um n
umero natural, ent
ao:
(pk ) = pk1 (p 1).
Os u
nicos n
umeros do conjunto {1, 2, . . . , pk } que nao s
ao relativamente primos com pk s
ao
pk
aqueles que s
ao divisveis por p. A quantidade de tais n
umeros e
= pk1 . Sendo assim,
p
(pk ) = pk pk1 = pk1 (p 1).
Nosso proximo objetivo sera encontrar uma formula para calcular explicitamente (m) em
funcao da fatoracao em primos de m. Precisaremos relembrar um exemplo estudado na
aula 6:
Lema 3. Sejam m um n
umero natural, l um n
umero natural relativamente primo com m
e r um inteiro arbitr
ario. Ent
ao, o conjunto:
r, l + r, 2l + r, . . . , (m 1)l + r;
e um sistema completo de restos m
odulo m.
Suponha, por absurdo, que existem dois inteiros i e j com 0 i < j < m e para os quais
tenhamos r + il r + jl (mod m). Assim, (j i)l 0 (mod m). Como l e relativamente
primo com m, devemos ter j i 0 (mod m). Obtemos um absurdo pois 0 < j i < m.
Consequentemente, temos um conjunto de m inteiros todos incongruentes m
odulo m e,
portanto, tal conjunto e um sistema completo de restos.

POT 2012 - Teoria dos N


umeros - Nvel 2 - Aula 9 - Samuel Feitosa

Teorema 4. Se l e m s
ao n
umeros naturais primos entre si, ent
ao:
(ml) = (m)(l).
Demonstraca
o. Como (1) = 1, o teorema anterior e valido quando m = 1 ou n = 1.
Suponha entao que m, l > 1. Facamos uma contagem dupla. Primeiramente, usando a
definicao, (mn) e o n
umero de inteiros da tabela abaixo que s
ao relativamente primos
com ml.
1,
1 + l,
21 + l,
...,
(m 1)1 + l,

2,
l + 2,
2l + 2,
...,
(m 1)l + 2,

...,
...,
...,
...,
...,

r,
l + r,
2l + r,
...,
(m 1)l + r,

...,
...,
...,
...,
...,

l,
2l,
3l,
...,
ml,

Seja r m um n
umero natural qualquer. Considerando a r-esima coluna da tabela, se
mdc(r, l) > 1, nenhum de seus elementos e relativamente primo com l. Entao, se buscamos
os elementos que nao possuem nenhum fator em comum com ml, devemos nos ater `as
colunas com mdc(r, l) = 1. O n
umero de tais colunas e (l). Considerando agora a r-esima
coluna e supondo que mdc(r, l) = 1, em virtude do lema anterior, sabemos que os restos de
seus elementos na divisao por m formam exatamente o conjunto {0, 1, . . . , m} e dentre eles
existem exatamente (m) n
umeros relativamente primos com m. Sendo assim, podemos
contar os n
umeros relativamente primos com ml atr
aves do n
umero de colunas boase do
n
umero de bonselementos em cada uma delas, obtendo: (m)(l).
o em primos de n, ent
ao:
Corol
ario 5. Se n = p1 1 p2 2 . . . pk k e a fatoraca



 
1
1
1
(n) = n 1
1
... 1
p1
p2
pk
De fato, pelo teorema anterior,
(n) = (p1 1 p2 2 . . . pk k )

= (p1 1 )(p2 2 ) . . . (pk k )

= p11 1 (p1 1)p2 2 1 (p2 1) . . . pk k 1 (pk 1)

= p11 1 p22 1 . . . pk k 1 (p1 1)(p2 1) . . . (pk 1)





 
1
1
1
= n 1
1
... 1
p1
p2
pk
Exemplo 6. Mostre que qualquer n 7 pode ser escrito na forma a + b, com a e b naturais
primos entre si, ambos maiores que 1.
Podemos escrever b = n a e nosso objetivo e encontrar a com 1 < a < n 1 tal que
mdc(a, n a) = 1. Para isso, basta que mdc(a, n) = 1. Pelo corolario anterior,
(n) = p1 1 1 p2 2 1 . . . pk k 1 (p1 1)(p2 1) . . . (pk 1)

POT 2012 - Teoria dos N


umeros - Nvel 2 - Aula 9 - Samuel Feitosa

Se a expressao anterior e igual `


a 2, necessariamente devemos ter i = 1 e pi = 2 ou 3 para
todo i. Sendo assim, n 6. Logo, (n) > 2 e existe pelo menos outro n
umero natural
diferente de 1 e n 1 que e relativamente primo com n.
Exemplo 7. Prove que existem infinitos inteiros positivos n tais que
(n) =

n
.
3

Basta tomar n = 2 3m , onde m e um inteiro positivo. Entao:


(n) = (2 3m ) = (2)(3m ) = 2 3m1 =

n
.
3

Exemplo 8. Se n e um inteiro positivo composto, ent


ao

(n) n n
Se n = p1 1 p2 2 . . . pk k , usando que n e composto, podemos garantir que existe um fator

primo pi tal que pi n. Assim,





 
1
1
1
(n) = n 1
1
... 1
p1
p2
pk


1
n 1
pi


1

n 1
n

= n n
Teorema 9. (Teorema de Euler) Se mdc(a, m) = 1, ent
ao
a(m) 1 (mod m)
Demonstraca
o. A prova deste teorema sera muito similar `a prova do teorema de Fermat.
Sejam r1 , r2 , . . . , r(m) os restos em {0, 1, 2, . . . , m 1} que s
ao relativamente primos com
m. Considere o conjunto {ar1 , ar2 , . . . , ar(m) }. Se dois de seus membros deixam o mesmo
resto por m, digamos:
ari arj (mod m);
temos ri rj (mod m) pois mdc(a, m) = 1. Claramente isso e uma contradicao. Alem
disso, mdc(ari , m) = mdc(m, ri ) = 1. Analisando os restos na divisao por m dos membros
desse novo conjunto, podemos concluir que tal conjunto coincide com o conjunto dos restos
iniciais. Assim,
r1 r2 . . . r(m) ar1 ar2 . . . ar(m)

a(m) r1 r2 . . . r(m)

Como mdc(r1 r2 . . . r(m) , m) = 1, podemos cancelar esse termo em ambos os membros


da congruencia anterior obtendo assim o teorema de Euler.
3

POT 2012 - Teoria dos N


umeros - Nvel 2 - Aula 9 - Samuel Feitosa

Exemplo 10. Encontre os u


ltimos tres dgitos de 79999
Como (1000) = 400, usando o Teorema de Euler, obtemos:
710000 = (7400 )25
1 (mod 1000)
Note que 7 143 = 1001 1 (mod 1000). Assim,
79999 79999 7 143
710000 143
143

(mod 1000)

Logo, 79999 termina em 143.


Exemplo 11. (Putnam 1972) Prove que n
ao existe um inteiro n > 1 tal que n|2n 1.
Se existem tais inteiros positivos, denotemos por m o menor deles. Claramente m e mpar,
pelo teorema de Euler, podemos garantir que:
m | 2(m) 1.
Seja d = mdc(m, (m)). Pelo problema 27 da aula 3, temos 2d 1 = mdc(2m 1, 2(m) 1).
Como m | mdc(2m 1, 2(m) 1), d > 1. Alem disso, d (m) < m e d | 2d 1. Isso e
um absurdo pois m e o menor inteiro maior que 1 com tal propriedade.
Exemplo 12. (Olimpada de Matem
atica Argentina) Demostre que para cada n
umero natural n, existe uma potencia de 2 cuja expans
ao decimal tem entre seus u
ltimos n dgitos
2n
(da direita) mais de
dgitos que s
ao iguais a 0.
3
Se 2k tiver um resto muito pequeno m
odulo 10n , poderemos garantir que existir
ao muitos zeros consecutivos entre seus u
ltimos dgitos. Para obtermos a equacao 2k r (mod 10n ) com
r pequeno, e interessante comecarmos analisando 2k (mod 5n ) uma vez que mdc(2, 5n ) = 1.
Facamos isso. Pelo teorema de Euler, temos:
2(n) 1 (mod 5n )

2(n)+n 2n

(mod 10n ).
n
dgitos e, conse3
2n
n
dgitos
existem pelo menos n =
3
3

Como 2n = 8n/3 < 10n/3 , podemos concluir que 2n possui menos que
quentemente, entre os u
ltimos n dgitos de 2(n)+n
consecutivos iguais `
a zero.

Exemplo 13. (IMO 1971) Prove que a sequencia 2n 3(n > 1) contem uma subsequencia
de n
umeros primos entre si dois a dois.

POT 2012 - Teoria dos N


umeros - Nvel 2 - Aula 9 - Samuel Feitosa

Uma boa estrategia e construir uma sequencia recursivamente. Suponha que ja tenhamos
escolhido os termos a1 , a2 , . . . , ak na sequencia de modo que mdc(ai , aj ) = 1. Como poderemos escolher o proximo termo ak+1 da forma 2n 3? Claramente mdc(2, ai ) = 1. Desde
que (ai ) | n, poderemos usar o teorema de Euler para obter:
2n 3 1 3

6 0 (mod ai )

Sendo assim, pelo teorema 4, basta escolhermos:


n = (a1 a2 . . . ak ) = (a1 )(a2 ) . . . (ak );
que naturalmente sera um m
ultiplo de cada (ai ). Logo, podemos definir
ak+1 = 2(a1 a2 ...ak ) 3
e assim temos uma sequencia de termos infita satisfazendo as condicoes do enunciado.

Problemas Propostos
Problema 14. Encontre todos os n
umeros naturais n para os quais (n) n
ao e divisvel por
4.
Problema 15. Prove que se p > 2 e 2p + 1 s
ao ambos n
umeros primos, ent
ao para n = 4p
vale que
(n + 2) = (n) + 2.
Problema 16. Encontre todas as soluco
es nos n
umeros naturais da equaca
o (n) = (2n).
Problema 17. Encontre todas as soluco
es nos n
umeros naturais da equaca
o (2n) = (3n).
Problema 18. Se n possui k fatores primos distintos, prove que 2k | (n).
Problema 19. Prove que para qualquer n
umero natural k, existe pelo menos um n
umero
natural n tal que
(n + k) = (n).
Dica: Considere o menor divisor primo p que n
ao e um divisor de k e estude o n
umero
n = (p 1)k.
Problema 20. Mostre que se a e b s
ao inteiros primos entre si, ent
ao existem inteiros m e
n tais que am + bn 1 (mod ab).

Problema 21. (Alemanha) Se n e um n


umero natural tal que 4n + 2n + 1 e primo, prove
que n e potencia de 3.

Problema 22. (USAMO 1991) Mostre que para qualquer inteiro fixo n 1, a sequencia
2

22

2, 22 , 22 , 22 , . . .

(mod n);

e eventualmente constante, isto e, a partir de um certo termo da sequencia todos os restos


obtidos na divis
ao por n ser
ao iguais.

Dica: Tente considerar os casos em que n e par ou n e mpar em separado e use induca
o.
Problema 23. Encontre os u
ltimos 8 dgitos da expans
ao bin
aria de 271986
Problema 24. Mostre que, para qualquer inteiro positivo n com n 6= 2 e n 6= 6 temos:

(n) n.

Refer
encias
[1] F. E. Brochero Martinez, C. G. Moreira, N. C. Saldanha, E. Tengan - Teoria dos
N
umeros ? um passeio com primos e outros n
umeros familiares pelo mundo inteiro,
Projeto Euclides, IMPA, 2010.
[2] E. Carneiro, O. Campos and F. Paiva, Olimpadas Cearenses de Matem
atica 1981-2005
(Nveis J
unior e Senior), Ed. Realce, 2005.
[3] S. B. Feitosa, B. Holanda, Y. Lima and C. T. Magalhaes, Treinamento Cone Sul 2008.
Fortaleza, Ed. Realce, 2010.
[4] D. Fomin, A. Kirichenko, Leningrad Mathematical Olympiads 1987-1991, MathPro
Press, Westford, MA, 1994.
[5] D. Fomin, S. Genkin and I. Itenberg, Mathematical Circles, Mathematical Words, Vol.
7, American Mathematical Society, Boston, MA, 1966.
[6] I. Niven, H. S. Zuckerman, and H. L. Montgomery, An Introduction to the Theory of
Numbers.

Polos Olmpicos de Treinamento


Aula

Curso de Teoria dos Nmeros - Nvel 2

10

Prof. Samuel Feitosa

Divisores
Suponha que n = p1 1 p2 2 . . . pk k e a fatoracao em primos do inteiro n. Todos os divisores
umeros, aparece
de n s
ao da forma m = p1 1 p2 2 . . . pk k , onde 0 i i . Cada um desses n
exatamente uma vez no produto:
(1 + p1 + p21 + . . . + p1 1 )(1 + p2 + p22 + . . . + p2 2 ) . . . (1 + pn + p2n + . . . + pkk ),
quando o mesmo e expandido usando a distributividade. Como existem i + 1 termos em
cada parenteses, O n
umero de termos dessa expans
ao e:
(1 + 1)(2 + 1) . . . (k + 1).
Alem disso, sabemos que:
1 + pi + p2i + . . . + pi i =

pi i +1 1
.
pi 1

Sendo assim, podemos concluir que:


Teorema 1. Se n = p1 1 p2 2 . . . pk k e a fatoraca
o em primos de n, ent
ao:
a) O n
umero de divisores de n, denotado por d(n), e: (1 + 1)(2 + 1) . . . (n + 1).
b) A soma dos divisores de n, denotada por (n), e:
(1 + p1 + p21 + . . . + p1 1 )(1 + p2 + p22 + . . . + p2 2 ) . . . (1 + pn + p2n + . . . + pnn )
ou, de forma mais sucinta,
p1 1 +1 1
p1 1

p2 2 +1 1
p2 1

...

pnn +1 1
pn 1

POT 2012 - Teoria dos N


umeros - Nvel 2 - Aula 10 - Samuel Feitosa
n
Observac
ao 2. (Pareamento de divisores) Se d e um divisor de n, ent
ao
tambem e um
d
divisor de n.

n
Portanto, pelo menos um dentre {d, } e um divisor de n menor ou igual a n.
d
Exemplo 3. Determine o n
umero de divisores positivos de 20088 que s
ao menores que
20084 .
O n
umero de divisores de 20088 = 224 2518 e 225. Como n e um quadrado
perfeito e em

virtude da observacao anterior, 112 desses divisores s


ao menores que 20088 = 20084 e 112
s
ao maiores.
Exemplo 4. Encontre a soma dos inversos dos divisores postivos de 496.
Sejam d1 , d2 , . . . , dn os divisores de 496 e K a soma de seus inversos. Usando a observacao
496 496
496
anterior, o conjunto {
+
+. . .+
} coincide com o conjunto {dn + dn1 + . . . + d1 }
d1
d2
dn
e da:
1
1
1
+
+ ... +
d1 d2
dn
496 496
496
+
+ ... +
d1
d2
dn
dn + dn1 + . . . + d1
25 1 312 1

2 1 31 1
960
496
Portanto, k =

= K
= 496K
= 496K
= 496K
= K.

60
.
31

Exemplo 5. Um n
umero natural n possui exatamente dois divisores e n + 1 possui exatamente 3 divisores. Encontre o n
umero de divisores de n + 2.
Se n possui exatamente dois divisores, entao n = p e um n
umero primo. Se n + 1 possui
2
um n
umero mpar de divisores, entao n + 1 = x e um quadrado perfeito, para algum x
inteiro positivo. Logo, x2 1 = (x 1)(x + 1) = p. Como p e primo, a u
nica possibilidade
e x 1 = 1 e consequentemente n = 3. O n
umero de divisores de n + 2 = 5 e 2.

Exemplo 6. Encontre todos os inteiros n que possuem exatamente n divisores positivos.

Para n ser inteiro, n deve ser um quadrado perfeito e assim podemos escrever:
2k
2
n = p121 p2
2 . . . pk .

A condicao do problema e equivalente a`:


p1 1 p2 2 . . . pk k = (21 + 1)(22 + 1) . . . (2k + 1).
2

POT 2012 - Teoria dos N


umeros - Nvel 2 - Aula 10 - Samuel Feitosa

Analisando o lado direito, podemos concluir que cada pi e mpar e consequentemente


pi i 3i 2i + 1.
Como devemos ter a igualdade, p1 = 3 e 31 = 21 + 1. Se 1 > 1, vale a desigualdade
estrita(veja o problema 13). Logo, a u
nica solucao e n = 9.
Exemplo 7. (Suica 2011) Encontre todos os inteiros positivos n para o qual n3 e o produto
de todos os divores de n
Claramente n = 1 e solucao. Suponha que n > 1 e sejam d1 < d2 < . . . < dk os divisores
de n. Pela observacao 2, podemos agrupar os divisores em pares cujo produto e n, assim:
n6 = (d1 d2 . . . dk )(d1 d2 . . . dk )
= (d1 dk )(d2 dk1 ) . . . (dk d1 )
= nd(n)
Consequentemente, 6 = d(n) e n = p5 ou n = pq 2 com p e q primos distintos. Fica a cargo
do leitor verificar que essas solucoes satisfazem o enunciado.
Exemplo 8. (Irlanda 1995) Para cada inteiro positivo n tal que n = p1 p2 p3 p4 , onde p1 , p2 ,
p3 e p4 s
ao primos distintos, sejam:
d1 = 1 < d2 < d3 < . . . < d16 = n,
os 16 inteiros positivos que dividem n. Prove que se n < 1995, ent
ao d9 d8 6= 22.
Suponha que n < 1995 e d9 d8 = 22. Note inicialmente que d8 nao pode ser par pois n
seria divisvel por 4 contradizendo o fato de que n possui quatro fatores primos distintos.
Consequentemente d8 , d9 e n s
ao mpares. Tambem temos a fatoracao: 35 57 = 1995 =
3 5 7 19. Entao, usando a observacao 2, d8 d9 = n. Se d8 35 teramos d9 < d8 para
manter n < 1995 e isso seria um absurdo. Logo, d8 < 35. Os divisores d1 , d2 , . . . , d8 s
ao
produtos de primos mpares distintos. Como 3 5 7 > 35, nenhum dentre d1 , d2 , . . . , d8
e grande o suficiente para possuir tres fatores primos distintos. Como n possui somente
quatro fatores primos distintos, quatro desses di s devem ser o produto de dois primos
mpares. Os menores n
umeros que s
ao o produto de dois primos s
ao:
15, 21, 33, 35, . . .
e consequentemente devemos ter d8 35, uma contradicao.
Exemplo 9. Prove que n
ao existe inteiro positivo n tal que (n) = nk para algum inteiro
positivo k.
Afirmamos que n = 1 e a u
nica solucao. Suponha que n > 1 seja solucao e sejam
d1 = 1 < d2 < . . . < dk = n,
3

POT 2012 - Teoria dos N


umeros - Nvel 2 - Aula 10 - Samuel Feitosa

os divisores de n. Entao
(n) = d1 + d2 + . . . + dk < 1 + 2 + . . . + n =

n(n + 1)
< n2 .
2

Alem disso,
Da,

n < n + 1 d1 + d2 + . . . + dk = (n).
n < nk < n2 ,

e obtemos um absurdo.
Exemplo 10. (Olimpada de Leningrado 1989) Duas pessoas jogam um jogo. O n
umero 2
est
a inicialmente escrito no quadro. Cada jogador, na sua vez, muda o n
umero atual N
no quadro negro pelo n
umero N + d, onde d e um divisor de N com d < N . O jogador que
escrever um n
umero maior que 19891988 perde o jogo. Qual deles ir
a vencer se ambos os
jogadores s
ao perfeitos.
Nesse problema, basta determinarmos apenas aquele que possui a estrategia vencedora.
Note que o incio do jogo e estritamente determinado: 2 3 4. Suponha que o segundo
jogador vence o jogo. Ap
os o movimento 4 5 do primeiro jogador, o segundo s
o pode
jogar 5 6. Isto significa que 6 e uma posicao vencedora. Entretanto, o primeiro jogador
pode obter a posicao 6 jogando 4 6, uma contradicao. Logo, o primeiro jogador possui
a estrategia vencendora.
Exemplo 11. (Olimpada de Leningrado) Duas pilhas de palitos sobre uma mesa contem
100 e 252 palitos, respectivamente. Dois jogadores jogam o seguinte jogo: Cada jogador em
sua vez pode remover alguns palitos de uma das pilhas de modo que o n
umero de palitos
retirados seja um divisor do n
umero de palitos da outra pilha. O jogador que fizer o u
ltimo
movimento vence. Qual dos dois jogadores ir
a vencer se ambos s
ao perfeitos?
O primeiro jogador perde. Em cada momento do jogo, podemos registrar o expoente
da maior potencia de 2 que divide os n
umeros de palitos em cada pilha. Por exemplo,
no incio os n
umeros s
ao (2, 2). A estrategia do segundo jogador e manter esse n
umeros
m
sempre iguais. Suponha que, em um dado momento, as pilhas possuem 2 a e 2m b
palitos com a e b mpares. O par registrado sera (m, m). Vejamos o que acontece quando
retiramos um divisor d da segunda pilha do n
umero de palitos da primeira. Se 2m e a
m+1
maior potencia de 2 que divide d, entao 2
dividira o n
umero de palitos da primeira
pilha e consequentemente o par registrado ter
a n
umeros diferentes. Se 2k , com k < m,
e a maior potencia de 2 que divide d, entao 2k sera a maior potencia de 2 que divide o
n
umero de palitos da primeira pilha e novamente o par registrado ter
a n
umeros diferentes.
Assim, sempre que um jogador receber um par registrado com n
umeros iguais, ele ir
a passar
um par registrado com n
umeros diferentes para o outro jogador. Suponha agora que, na
sua vez, as pilhas possuem 2m a e 2n b palitos, com m < n e a b 1 (mod 2).
Basta o jogador retirar 2m palitos da segunda pilha para passar um par registrado com
n
umeros iguais a (m, m). Como inicialmente as pilhas possuem n
umeros registrados iguais,
o segundo jogador pode sempre manter essa propriedade e consequentemente o u
nico que
pode passar uma pilha com zero palitos pela primeira vez e o primeiro jogador.
4


POT 2012 - Teoria dos N
umeros - Nvel 2 - Aula 10 - Samuel Feitosa
REFERENCIAS

Problemas Propostos
Problema 12. Mostre que se k e um inteiro positivo ent
ao 3k 2k +1 e vale a desigualdade
estrita quando k > 1.
Problema 13. (R
ussia 2001) Encontre todos os n tais que quaisquer divisores primos distintos a e b de n o n
umero a + b 1 tambem e um divisor de n
Problema 14. O n
umero 332 1 tem exatamente dois divisores que s
ao maiores que 75 e
menores que 85. Qual o produto desses dois divisores?

Problema 15. (Ir


a 2012) Sejam a e b inteiros positivos de modo que o n
umero de divisores
positivos de a,b, ab e 3,4 e 8, respectivamente. Encontre o n
umero de divisores positivos
de b2 .
Problema 16. (Olimpada de S
ao Petesburgo) Enconte todos os inteiros positivos n tais
que 3n1 + 5n1 divide 3n + 5n .
Problema 17. Sejam 1 = d1 < d2 < .... < dk = n o conjunto de todos os divisores de um
inteiro positivo n. Determine todos os n tais que:
d26 + d27 1 = n.
Problema 18. Um divisor d > 0 de um inteiro positivo n e dito ser um divisor unit
ario
n
se mdc(d, ) = 1. Suponha que n e um inteiro positivo tal que a soma de seus divisores
d
unit
arios e 2n. Prove que n n
ao pode ser mpar.

Refer
encias
[1] F. E. Brochero Martinez, C. G. Moreira, N. C. Saldanha, E. Tengan - Teoria dos
N
umeros ? um passeio com primos e outros n
umeros familiares pelo mundo inteiro,
Projeto Euclides, IMPA, 2010.
[2] E. Carneiro, O. Campos and F. Paiva, Olimpadas Cearenses de Matem
atica 1981-2005
(Nveis J
unior e Senior), Ed. Realce, 2005.
[3] S. B. Feitosa, B. Holanda, Y. Lima and C. T. Magalhaes, Treinamento Cone Sul 2008.
Fortaleza, Ed. Realce, 2010.
[4] D. Fomin, A. Kirichenko, Leningrad Mathematical Olympiads 1987-1991, MathPro
Press, Westford, MA, 1994.
[5] D. Fomin, S. Genkin and I. Itenberg, Mathematical Circles, Mathematical Words, Vol.
7, American Mathematical Society, Boston, MA, 1966.
[6] I. Niven, H. S. Zuckerman, and H. L. Montgomery, An Introduction to the Theory of
Numbers.

Polos Olmpicos de Treinamento


Curso de Teoria dos Nmeros - Nvel 2

Aula

11

Prof. Samuel Feitosa

O Teorema Chin
es dos Restos
Iremos estudar um antigo teorema descoberto pelos chineses no incio seculo XIII. Comecemos recordando um lema da aula 06:
Lema 1. Se mdc(a, m) = 1, ent
ao existe um inteiro x tal que:
ax 1 (mod m).
Tal inteiro e u
nico m
odulo m. Se mdc(a, m) > 1, n
ao existe x satisfazendo tal equaca
o.
Demonstraca
o. Pelo teorema de Bachet-Bezout, existem inteiros x e y tais que ax+my = 1.
Analisando essa congruencia m
odulo m, obtemos ax 1 (mod m). Se y e outro inteiro
que satisfaz a mesma congruencia, temos ax ay (mod m). Pelo primeiro lema, x y
(mod m). Se d = mdc(a, m) > 1, n
ao podemos ter d | m e m | ax 1 pois d ax 1.
Exemplo 2. Encontre x inteiro tal que:
x 1

(mod 11);

x 2

(mod 7).

A primeira congruencia nos diz que x = 11k + 1 para algum k Z. Sejam q e r o quociente
e o resto da divis
ao de k por 7, respectivamente. Assim, k = 7q + r e x = 77q + 11r + 1.
Para x satisfazer a segunda congruencia, devemos encontrar r {0, 1, 2, 3, 4, 5, 6} tal que
11r + 1 2 (mod 7), ou seja, 4r 1 (mod 7). Como o inverso de 4 (mod 7) e 2, obtemos
r = 2 e x = 77q + 23. Veja que para qualquer q inteiro, tal x e solucao do sistema de
congruencias.
Exemplo 3. Encontre x inteiro tal que:
x 1 (mod 11)
x 2 (mod 7)
x 4 (mod 5)

POT 2012 - Teoria dos N


umeros - Nvel 2 - Aula 11 - Samuel Feitosa

Pelo exemplo anterior, para x satisfazer as duas primeiras equacoes, devemos ter
x = 77q +23. Dividindo q por 5, obtemos q = 5l+s com 0 s < 5. Da, x = 385l+77s+23.
Para satisfazer a u
ltima congruencia, devemos ter 77s + 23 4 (mod 5), ou seja, 2s 1
(mod 5). Como 3 e o inverso de 2 (mod 5), s = 3 e consequentemente x = 385l + 254.
Perceba que nos dois exemplos anteriores, o problema foi reduzido `a encontrarmos o inverso
de um inteiro. No u
ltimo exemplo, a solucao geral possui a forma: x = 1175l+231+22+1.
Essencialmente, o trabalho de encontrar esses inversos foi possvel pois os inteiros 5, 7 e 11
s
ao primos entre si dois a dois.
Veremos agora um mecanismo levemente diferente para resolver tais sistemas equacoes.
Teorema 4. (Teorema Chin
es dos Restos) Sejam m1 , m2 , . . . , mr , inteiros positivos
primos entre si, dois a dois, e sejam a1 , a2 , . . . , ar ; r inteiros quaisquer. Ent
ao, o sistema
de conguencias:
x a1

(mod m1 )

x a2
..
.

(mod m2 )

x ar

(mod mr )

admite uma soluca


o x. Alem disso, as soluco
es s
ao u
nicas m
odulo m = m1 m2 . . . mr .
m
e um inteiro e
Demonstraca
o. Escrevendo m = m1 m2 . . . mr , vemos que
m
j


m
mdc
, mj = 1. Ent
ao, pelo lema inicial, para cada j, existe um inteiro bj tal que
 
 mj
m
m
bj 1 (mod mj ). Claramente
bj 0 (mod mi ) para i 6= j. Definamos
mj
mj
x0 =

m
m
m
b1 a1 +
b2 a 2 + . . . +
br a r
m1
m2
mr

m
bj aj ai (mod mi ). Logo, x0 e uma solucao do
mi
nosso sistema. Se x0 e x1 tambem o s
ao, podemos escrever x0 x1 (mod mi ) para cada i.
Como mdc(mi , mj ) = 1, para i 6= j, pbtemos x0 x1 (mod m).
Consideremos x0 m
odulo mi : x0

Observac
ao 5. Se cada uma das equaco
es do sistema anterior fosse do tipo
bi x ai (mod m)i , com mdc(bi , m) = 1, ainda poderamos us
a-lo. Bastaria reescrever
bi x ai (mod m)i como x bi ai (mod m)i , onde bi e o inverso de bi (mod m)i .
Exemplo 6. Encontre o menor inteiro positivo x tal que x 5 (mod 7), x 7 (mod 11) e
x 3 (mod 13).
Usando o teorema anterior com m1 = 5, m2 = 7, m3 = 11, a1 = 5, a2 = 7 e a3 = 3 podemos
achar x 887 (mod 1001) = 7.11.13. Como a solucao e u
nica m
odulo m, isso significa
que, dentre os n
umeros 1, 2, , 1001 a menor solucao positiva e 887.
2

POT 2012 - Teoria dos N


umeros - Nvel 2 - Aula 11 - Samuel Feitosa

Exemplo 7. (OBM 2009) Sejam m e n dois inteiros positivos primos entre si. O Teorema
Chines dos Restos afirma que, dados inteiros i e j com 0 i < m e o j < n, existe
exatamente um inteiro a, com 0 a < mn, tal que o resto da divis
ao de a por m e igual
a i e o resto da divis
ao de a por n e igual a j. Por exemplo, para m = 3 e n = 7, temos
que 19 e o u
nico n
umero que deixa restos 1 e 5 quando dividido por 3 e 7, respectivamente.
Assim, na tabela a seguir, cada n
umero de 0 a 20 aparecer
a exatamente uma vez.
0
0
1
2

1
A

5
B

C
E

6
D

Qual a soma dos n


umeros das casas com as letras A, B, C, D, E e F ?
Usando o teorema chines dos restos, podemos encontrar A = 15, B = 12, C = 10, D =
13, E = 8 e F = 11. Assim, A + B + C + D + E + F = 69.
Exemplo 8. (Est
onia 2000) Determine todos os restos possveis da divis
ao do quadrado de
um n
umero primo com 120 por 120.
Seja n tal que mdc(n, 120) = 1. Como 120 = 3 5 8, temos que n 6 0 (mod 3), (mod 5)
(mod 2). Da, n2 1 (mod 3), n2 1 (mod 8) e n2 1 ou 4 (mod 5). Sendo assim, n2
satisfaz o sistema:
x 1 (mod 3)
x 1 (mod 8)
x 1 (mod 5)

cujas solucoes s
ao x 1 (mod 120) e x 49 (mod 120).
Aconselhamos ao leitor a resoluc
ao de alguns exemplos numericos ate adquirir pr
atica com o
algoritmo usado para encontrar x0 . Provamos, no teorema passado, que todas as solucoes
daquele sistema de congruencias s
ao os termos de uma P.A de raz
ao m. Geralmente
usaremos aquele teorema apenas para garantir que um sistema de congruencias admite
uma solucao. Os pr
oximos exemplos podem deixar isso mais claro.
Exemplo 9. Para cada n
umero natural n, existe uma sequencia arbitrariamente longa de
n
umeros natu rais consecutivos, cada um deles sendo divisvel por uma s-esima potencia
de um n
umero natural maior que 1.
Demonstraca
o. Dado m N, considere o conjunto {p1 , p2 , . . . , pm } de primos distintos.
Como mdc(psi , psj ) = 1, ent
ao pelo teorema 3, existe x tal que x i (mod psi ) para
i = 1, 2, . . . m. Cada um dos n
umeros do conjunto {x + 1, x + 2, . . . , x + m} e divisvel por
um n
umero da forma psi .

POT 2012 - Teoria dos N


umeros - Nvel 2 - Aula 11 - Samuel Feitosa

Exemplo 10. (USAMO 1986)


(a) Existem 14 inteiros positivos consecutivos tais que, cada um e divisvel por um ou
mais primos p do intervalo 2 p 11?
(b) Existem 21 inteiros positivos consecutivos tais que, cada um e divisvel por um ou
mais primos p do intervalo 2 p 13?
Demonstraca
o. (a) Nao. Suponha que existam tais inteiros. Da nossa lista de 14 inteiros
consecutivos, 7 s
ao n
umeros pares. Vamos observar os mpares: a, a + 2, a + 4, a + 6, a +
8, a + 10 e a + 12. Podemos ter no m
aximo tres deles divisveis por 3, dois por 5, um por
7 e um por 11. Veja que 3 + 2 + 1 + 1 = 7. Pelo Princpio da Casa dos Pombos, cada um
desses mpares e divisvel por exatamente um primo do conjunto {3, 5, 7, 11}. Alem disso,
note que os m
ultiplos de 3 s
o podem ser {a, a + 6, a + 12}. Dois dos n
umeros restantes em
(a + 2, a + 4, a + 8, e a + 10) s
ao divisveis por 5. Mas isso e impossvel. (b) Sim. Como
os n
umeros {210, 11, 13} s
ao primos entre si, dois a dois, pelo teorema 3 existe um inteiro
positivo n > 10 tal que:
n 0( mod 210 = 2 3 5 7)
n 1( mod 11)
n 1( mod 13)
Veja que o conjunto {n 10, n 9, . . . , n + 9, n + 10} satisfaz as condicoes do item (b).
Exemplo 11. Sejam a e b inteiros positivos tais que, para qualquer n natural, an +n | bn +n.
Prove que a = b.
Seja p um primo maior que a e b. Ent
ao mdc(p, a) = mdc(p, b) = 1. Como mdc(p, p1) = 1,
existe um inteiro positivo n tal que n 1 (mod p 1) e n a (mod p). Pelo teorema de
Fermat, an + n 0 (mod ) e bn + n b a (mod p). Assim, p | |b a|. Como |b a| < p,
segue que |b a| = 0 e a = b.
Exemplo 12. (Olimpada N
ordica 1998)
(a) Para quais inteiros positivos n existe um sequencia x1 , x2 , . . . , xn contendo cada um
dos inteiros 1, 2, . . . , n exatamente uma vez, e tal que k divide x1 + x2 + + xk para
k = 1, 2, , n?
(b) Existe uma sequencia infinita x1 , x2 , . . . contendo todo inteiro positivo exatamente
uma vez, e tal que para cada inteiro positivo k, k divide x1 + x2 + + xk ?
a) Suponha que n e um inteiro que satisfaz o enunciado. Naturalmente n divide a soma:
x1 + x2 + . . . xn =
Da,

n(n + 1)
.
2

n+1
n+1
e um inteiro e n deve ser mpar. Seja m =
. Usando que
2
2
(n 1) | x1 + x2 + . . . xn1 = mn xn ,

POT 2012 - Teoria dos N


umeros - Nvel 2 - Aula 11 - Samuel Feitosa

temos xn m (mod n 1) se n 3 e, consequentemente, xn = m. Repetindo a mesma


an
alise para n 2 no lugar de n 1, obtemos xn1 = m para n 5. Como n
ao podem
existir dois termos iguais, temos um absurdo. Analisando os casos quando n 4, encontramos n = 1 e n = 3 como u
nicas solucoes.
b) Iremos construir a sequencia indutivamente. Suponha que ja tenhamos definido os termos
x1 , x2 , . . . , xn satisfazendo a condic
ao k | x1 + x2 . . . xk para todo k n. Seja m o menor inteiro positivo que ainda n
ao apareceu na sequencia. Pelo Teorema Chines dos Restos, existe
x tal que x (x1 +x2 +. . .+xn ) (mod n+1) e x (x1 +x2 +. . .+xn )m (mod n+2).
Escolha l, inteiro positivo, tal que l > x1 , x2 , . . . , xn , m e l x (mod (n+1)(n+2)). Defina
xn+1 = l e xn+2 = m. Veja que a condicao k | x1 + x2 . . . xk agora e verdadeira para todo
k n + 2. Para o incio, basta definir x1 = 1.
Exemplo 13. (Olimpada de S
ao Petesburgo 1990) Dado um polin
omio F (x) com coeficientes inteiros, tal que, para cada inteiro n, o valor de F (n) e divisvel por pelo menos
um dos inteiros a1 , a2 , , am . Prove que podemos encontrar um ndice k tal que F (n) e
divisvel por ak para cada inteiro positivo n.
Demonstraca
o. Suponha que n
ao exista tal ndice. Para cada ndice k (k = 1, 2, . . . , m),
existe um inteiro xk tal que F (xk ) n
ao e divisvel por ak . Assim, existem n
umeros
k
ao n
umeros primos), tais que dk divide ak mas n
ao divide F (xk ). Se
dk = pk (onde pk s
existem potencias do mesmo primo entre esses n
umeros, podemos apagar aquelas repetidas deixando apenas uma que tem expoente mnimo. Caso F (x) n
ao seja divisvel por
uma potencia apagada, n
ao ser
a pela potencia que tem expoente mnimo. Essas delecoes
garatem que nossa nova colec
ao d1 , d2 , . . . , dj de potencias de primos contenham apenas
inteiros primos entre si, dois a dois. Pelo teorema chines dos restos, exite um inteiro N
tal que N xk (mod d)k , para k {1, 2, . . . , j}. Suponhamos que dk | F (N ). Sabemos
que x y | F (x) F (y) e consequentemente N xk | F (N ) F (xk ). Como dk | N xk ,
devemos ter dk | F (xk ). Uma contradicao! Logo, F (N ) n
ao e divisvel por nenhum dk e
isso contradiz a hip
otese sobre os ai .

Problemas Propostos
Problema 14. Encontre o menor inteiro positivo (com a exceca
o de x = 1) que satisfaca o
seguinte sistema de congruencias:
x 1

(mod 3)

x 1

(mod 5)

x 1

(mod 7)

POT 2012 - Teoria dos N


umeros - Nvel 2 - Aula 11 - Samuel Feitosa

Problema 15. Encontre todas as soluco


es do sistema:
x 2

(mod 3)

x 3

(mod 5)

x 5

(mod 2)

Problema 16. Encontre todos os inteiros que deixam restos 1, 2 e 3 quando divididos por
3, 4 e 5, respectivamente.
Problema 17. Encontre todas as soluco
es do sistema:
3x 1 (mod 4)
2x 1 (mod 3)
4x 5 (mod 7)

Problema 18. Encontre todas as soluco


es das congruencias:
a) 20x 4 (mod 30).
b) 20x 30 (mod 4).
c) 353x 254 (mod 400).
Problema 19. Se a e escolhido ao acaso no conjunto {1, 2, 3, . . . , 14} e b e escolhido ao
acaso no conjunto {1, 2, . . . , 15}, qual a probabilidade de que a equaca
o ax b (mod 15)
possua pelo menos uma soluca
o?
Problema 20. Sejam a e b inteiros tais que mdc(a, b) = 1 e c > 0. Prove que existe um
inteiro x tal que mdc(a + bx, c) = 1.
Problema 21. Existem n inteiros consecutivos tal que cada um contem um fator primo
repetido k vezes?
Problema 22. Seja n um n
umero natural arbitr
ario. Prove que existe um par de naturais
(a, b) tais que mdc(a + r, b + s) > 1 r, s = 1, 2, . . . , n.
Problema 23. Um ponto (x, y) Z 2 e legal se mdc(x, y) = 1. Prove ou disprove: Dado
um inteiro positivo n, existe um ponto (a, b) Z 2 cuja dist
ancia a todo ponto legal e pelo
menos n?
Problema 24. Sejam mo , m1 , ..., mr inteiros positivos que s
ao primos entre si, dois a dois.
Mostre que existem r+1 inteiros consecutivos s, s+1, ..., s+r tal que mi divide s+i para i =
0, 1, ..., r.
Problema 25. (Romenia 1995) Seja f : N{0, 1} N definida por f (n) = mmc[1, 2, ..., n].
Prove que para todo n 2, existem n n
umeros consecutivos para os quais f e constante.

Problema 26. (OBM 2005) Dados os inteiros positivos a, c e o inteiro b, prove que existe
um inteiro positivo x tal que ax + x b (mod c).
Problema 27. (Cone Sul 2003) Demonstrar que existe uma sequencia de inteiros positivos
x1 , x2 , . . . que satisfaz as duas condico
es seguintes:
(a) contem exatamente uma vez cada um dos inteiros positivos,
(b) a soma parcial x1 + x2 + . . . xn e divisvel por nn .
Problema 28. (Rep
ublica Tcheca e Eslovaca 1997) Mostre que existe uma sequencia crescente {an }
de
n
u
meros naturais tais que para k 0 , a sequencia {an + k} contem um
n=1
n
umero finito de primos.
Problema 29. Considere o inteiro c 1 e a sequencia definida por a1 = c e ai+1 = cai .
Mostre que esta sequencia se torna eventualmente constante quando a reduzimos m
odulo n
para algum inteiro positivo n (isto significa que am aj (mod n) se m j).
Problema 30. (Coreia 1999) Encontre todos os inteiros n tais que 2n 1 e um m
ultiplo de
2n 1
e um divisor de 4m2 + 1 para algum inteiro m.
3e
3
Problema 31. (OBM 2006) Prove que, para todo inteiro n 2, o n
umero de matrizes
quadradas 2 2 com entradas inteiras e pertencentes ao conjunto {0, 1, 2, . . . , n 1} que
tem determinante da forma kn + 1 para algum k inteiro e dado por


Y
1
1 2 .
p
p primo
p|n

Problema 32. Encontre todos os subconjuntos S Z+ tais que todas as somas de uma
quantidade finita de elementos de S(com possveis repetico
es de elementos) s
ao n
umeros
compostos.
Problema 33. Existe algum natural n para o qual existem n 1 progress
oes aritmeticas
com raz
oes 2, 3, . . . , n tais que qualquer natural est
a em pelo menos uma das progress
oes?
Problema 34. Seja P (X) um polin
omio com coeficientes inteiros e k e um inteior qualquer.
Prove que existe um inteiro m tal que P (m) tem pelo menos k fatores primos distintos.
Acompanhe as dicuss
oes dos problemas propostos no f
orum do POTI:
www.poti.impa.br/forum/

Refer
encias
[1] F. E. Brochero Martinez, C. G. Moreira, N. C. Saldanha, E. Tengan - Teoria dos
N
umeros ? um passeio com primos e outros n
umeros familiares pelo mundo inteiro,
Projeto Euclides, IMPA, 2010.
[2] E. Carneiro, O. Campos and F. Paiva, Olimpadas Cearenses de Matematica 1981-2005
(Nveis J
unior e Senior), Ed. Realce, 2005.
[3] S. B. Feitosa, B. Holanda, Y. Lima and C. T. Magalhaes, Treinamento Cone Sul 2008.
Fortaleza, Ed. Realce, 2010.
[4] D. Fomin, A. Kirichenko, Leningrad Mathematical Olympiads 1987-1991, MathPro
Press, Westford, MA, 1994.
[5] D. Fomin, S. Genkin and I. Itenberg, Mathematical Circles, Mathematical Words, Vol.
7, American Mathematical Society, Boston, MA, 1966.
[6] I. Niven, H. S. Zuckerman, and H. L. Montgomery, An Introduction to the Theory of
Numbers.

Polos Olmpicos de Treinamento


Curso de Teoria dos Nmeros - Nvel 2

Aula

12

Prof. Samuel Feitosa

Equac
oes Diofantinas II
Continuaremos nosso estudo das equacoes diofantinas abordando agora algumas equacoes
quadr
aticas. Comecaremos peloo cl
assico problema das ternas pitag
oricas.
Desejamos encontrar todas as soluc
oes (x, y, z) da equacao:
x2 + y 2 = z 2 ,
em inteiros positivos. Seja d = mdc(x, y). Como d2 | z 2 , segue que d | z e que
tambem e soluc
ao. Alem disso, podemos concluir que:

x y z 
, ,
d d d

mdc(x/d, y/d) = mdc(x/d, z/d) = mdc(y/d, z/d) = 1.


Uma terna que e soluc
ao e possui a propriedade de que quaisquer dois de seus termos
s
ao primos entre si, ser
a chamada de solucao primitiva. Assim, toda solucao (x, y, z) e da
forma (dx1 , dy1 , dz1 ) onde (x1 , y1 , z1 ) e uma solucao primitiva. Para cumprimirmos nosso
objetivo, bastara nos concentrarmos em encontrar todas as solucoes primitivas. Analisando
a equacao m
odulo 4 e lembrando que todo quadrado perfeito pode deixar apenas os restos 0
ou 1, conclumos que exatamente um dentre x e y e par. Suponha sem perda de generalidade
que y seja par. Fatorando a equac
ao, obtemos:
 y 2
z+x zx

=
2
2
2
Como mdc((z + x)/2, (z x)/2) = 1, conclumos que (z + x)/2 e (z x)/2 devem ser ambos
quadrados perfeitos, i.e., existem inteiros positivos r e s, com r > s e mdc(r, s) = 1, tais que
(z + x)/2 = r 2 e (z x)/2 = s2 (veja o primeiro problema proposto). Consequentemente,
x = r 2 s2 , y = 2rs e z = r 2 + s2 . Reciprocamente, se (x, y, z) = (r 2 s2 , 2rs, r 2 + s2 ),
temos:
x2 + y 2 = (r 2 s2 )2 + (2rs)2 = (r 2 + s2 )2 = z 2 .
O pr
oximo teorema resume nossa discussao original:

POT 2012 - Teoria dos N


umeros - Nvel 2 - Aula 12 - Samuel Feitosa

Teorema 1. Todas as soluco


es primitivas de x2 +y 2 = z 2 com y par s
ao da forma x = r 2 s2 ,
2
2
y = 2rs e z = r + s , onde r e s s
ao inteiros de paridade oposta com r > s > 0 e
mdc(r, s) = 1.
Exemplo 2. Encontre todas as ternas pitag
oricas (a, b, c) tais que a + b + c = 1000.
Seja k = mdc(a, b, c) e suponha sem perda de generalidade que b/k e par. Pelo teorema
anterior, (a, b, c) = (k(x2 y 2 ), k(2xy), k(x2 + y 2 )), onde x > y, mdc(x, y) = 1 e pelo menos
um dentre x e y par. Assim, (x2 y 2 ) + 2xy + (x2 + y 2 ) = 2x(x + y) e um divisor de 1000.
Com mais raz
ao, x(x + y) | 500. Usando que mdc(x, x + y) = 1 e a fatoracao em primos de
500, podemos concluir que um deles e uma potencia de 5 e o outro uma potencia de 2. Veja
que x n
ao pode ser uma potencia de 5 pois nesse caso y deveria ser mpar para garantir
que x + y seja uma potencia de 2. Assim, x | 500 e x = 2k , produzindo como possibilidades
x = 1, 2 ou 4. Analisando cada um desses casos e levando em conta que y < x, e facil
encontrar que x = 4 e y = 1 s
ao as u
nicas opcoes possveis. Nesse caso, x = 15, y = 8 e
z = 17. Consequentemente, (a, b, c) = (20 15, 20 8, 20 17).
Exemplo 3. Mostre que se a, b e c s
ao inteiros positivos tais que a2 + b2 = c2 , ent
ao
4
4
4
(ab) + (bc) + (ca) e um quadrado perfeito.
Veja que:
(ab)4 + (bc)4 + (ca)4 = (a2 b2 + b4 )2 + (a2 b2 )2 + (a2 b2 + a4 )2 = (a4 + a2 b2 + b4 )2 .
Exemplo 4. Encontre todas as soluco
es de x2 + 2y 2 = z 2 em inteiros positivos com
mdc(x, y, z) = 1.
Como 2y 2 0 (mod 2), devemos ter x z (mod 2). Alem disso, se fosse x z 0
(mod 2) teramos 4 | z 2 x2 = 2y 2 e consequentemente 2 | y, contradizendo a hip
ostese
mdc(x, y, z) = 1. Fatorando a express
ao, temos:
2y 2 = (z x)(z + x).
Como mdc(x, z) = 1 e ambos s
ao mpares; mdc(z x, z + x) = 2 e apenas um deles e
congruo `a 2 (mod 4). Temos dois casos a considerar: 1) z + x 0 (mod 4) e z x 2
(mod 4). Nesse caso, y 2 = (z x)/2 (z + x) com mdc((z x)/2, (z + x)) = 1. Da,
existem inteiros positivos r e s tais que (z x)/2 = r 2 e (z + x)/2 = s2 , produzindo a
solucao (x, y, z) = ((s2 2r 2 )/2, rs, (2r 2 + s2 )/2) com mdc(r, s) = 1 e s 0 (mod 2). Um
raciocnio an
alogo para o caso z + x 2 (mod 4) e z x 0 (mod 4) produz (x, y, z) =
((2s2 r 2 )/2, rs, (r 2 + 2s2 )/2) com mdc(r, s) = 1 e r 0 (mod 2).
Problema 5. (USAMO 1976) Encontre todas as soluco
es naturais da equaca
o
a2 + b2 + c2 = a2 b2 .
A equacao pode ser reescrita como:
c2 = (a2 1)(b2 1) 1.
2

POT 2012 - Teoria dos N


umeros - Nvel 2 - Aula 12 - Samuel Feitosa

Se pelo menos um dentre a ou b e mpar, teremos c2 3 (mod 4). Como os quadrados


perfeitos s
o podem deixar resto 0 ou 1 (mod 4), temos um absurdo. Portanto, a, b e
consequentemente c s
ao n
umeros pares. Seja k o maior inteiro tal que 2k divida esses tres
x
n
umeros. Assim, a = 2 , b = 2k y, c = 2k z onde pelo menos um dentre x, y e z mpar.
Assim,
x2 + y 2 + z 2 = 22r x2 y 2 .
Como r > 0, x2 + y 2 + z 2 0 (mod 4). Entretanto, isso n
ao e possvel se um dentre os
x, y, z e mpar pois a soma s
o poderia ser congruente a` 1, 2, 3 (mod 4).
Exemplo 6. (Extrado de [1]) Determine todas as ternas (a, b, c) de inteiros positivos tais
que a2 = 2b + c4 .
Como a2 = 2b + c4 (a c2 )(a + c2 ) = 2b , pelo Teorema Fundamental da Aritmetica
existem dois naturais m > n tais que m + n = b, a c2 = 2n e a + c2 = 2m . Subtraindo as
duas u
ltimas equac
oes, obtemos que 2c2 = 2m 2n , assim c2 = 2n1 (2mn 1). Como 2n1
e 2mn 1 s
ao primos entre si e o seu produto e um quadrado perfeito (i.e. os expoentes
das potencias de primos distintos s
ao pares), novamente pelo Teorema Fundamental da
Aritmetica 2n1 e 2mn 1 devem ser ambos quadrados perfeitos, logo n 1 e par e
2mn 1 = (2k1)2 para algum inteiro positivo k. Como 2mn = (2k1)2 +1 = 4k(k1)+2
e divisvel por 2 mas n
ao por 4, temos m n = 1. Assim, fazendo n 1 = 2t, temos que
todas as soluc
oes s
ao da forma (a, b, c) = (3 22t , 4t + 3, 2t ) com t N e e facil verificar que
todos os n
umeros desta forma s
ao solucoes.
O pr
oximo exemplo ilustrara o metodo da descida de Fermat que faz uso do princpio
da boa ordenac
ao: todo subconjunto n
ao vazio de inteiros positivos possui um elemento
mnimo.
Exemplo 7. Determine todas as soluco
es da equaca
o x4 + y 4 = z 2 em inteiros positivos
com mdc(x, y) = 1.
Como (x2 )2 + (y 2 )2 = z 2 e mdc(x2 , y 2 ) = 1, podemos usar o primeiro teorema para concluir
que existem u e v tais que x2 = u2 v 2 , y 2 = 2uv, z = u2 + v 2 , u > v > 0 e mdc(u, v) = 1
(Estamos assumindo sem perda de generalidade que x e mpar). Se u e par, entao v ser
a
mpar e teremos x2 3 (mod 4). Como isso e um absurdo, u deve ser mpar e v deve ser
par. Sendo assim, (y/2)2 = u v/2 com mdc(u, v/2) = 1. Devemos ter u = r 2 , v/2 = s2 ,
com mdc(r, s) = 1, r, s > 0, r mpar e y = 2rs. Alem disso, como x2 + v 2 = u2 , obtemos
x2 + 4s2 = r 4 . Como mdc(r, 2s) = 1, novamente pelo primeiro teorema, existem m e n tais
que x = m2 n2 , 2s2 = 2mn e r 2 = m2 + n2 com mdc(m, n) = 1 e m > n > 0. Como
mn = s2 , podemos escrever m = f 2 e n = g2 com f, g > 0 e mdc(f, g) = 1. Portanto,
r 2 = f 4 + g4 . Note que dada a soluc
ao em inteiros positivos (x, y, z), obtivemos outra
solucao (f, g, r), tambem nos inteiros positivos, com 0 < r < z. Isso nos diz que existe
uma infinidade decrescente de possveis valores para o inteiro positivo z e naturalmente
obtemos uma contradic
ao do princpio da boa ordenacao. Sendo assim, a equacao anterior
n
ao possui soluc
ao nos inteiros positivos.
Observac
ao 8. Outra maneira de formalizar o argumento anterior e escolher dentre as
ternas nos inteiros positivos que s
ao soluco
es, aquela com z mnimo. A nova terna (f, g, r)
caracterizaria um absurdo.
3

POT 2012 - Teoria dos N


umeros - Nvel 2 - Aula 12 - Samuel Feitosa

Exemplo 9. Prove que para todo inteiro n > 2, existem inteiros positivos p e q tais que
n 2 + q 2 = p2 .
Fatorando a express
ao, obtemos n2 = (p q)(p + q). Se n e mpar, podemos encontrar p e
2
q tais que p + q = n e p q = 1, bastando para isso resolver o sistema originado, obtendo
2
2
(n, q, p) = (n, n 21 , n 2+1 ). Se n e par, podemos fazer algo semelhante e encontrar p e q tais
2
2
que p + q = n2 /2 e p q = 2, cuja solucao e (n, q, p) = (n, n4 1, n4 + 1).
Exemplo 10. (Extrado de [3]) Prove que a equaca
o
x2 + y 2 + z 2 + w2 = 2xyzw

(1)

n
ao possui soluco
es inteiras positivas.
Por contradic
ao, suponha que (1) possua pelo menos uma solucao n
ao-trivial, digamos
(x0 , y0 , z0 , w0 ). Se x0 , y0 , z0 , w0 forem todos mpares, o lado esquerdo e um m
ultiplo de 4
e o lado direito n
ao. Se apenas um ou tres deles forem pares, o lado esquerdo e mpar e o
direito e par. Se dois deles forem pares e dois forem mpares, o lado direito e um m
ultiplo
de quatro e o esquerdo n
ao. Desse modo, x0 , y0 , z0 , w0 s
ao todos pares, ou seja, x0 = 2x1 ,
y0 = 2y1 , z0 = 2z1 e w0 = 2w1 . Substituindo em (1) e dividindo por quatro, conclumos
que x1 , y1 , z1 , w1 satisfazem a igualdade
x21 + y12 + z12 + w12 = 8x1 y1 z1 w1 .
Com uma an
alise de paridades an
aloga `a acima, obtemos x1 = 2x2 , y1 = 2y2 , z1 = 2z2 e
w1 = 2w2 , e da
x22 + y22 + z22 + w22 = 32x2 y2 z2 w2 .
Procedendo dessa maneira, x0 , y0 , z0 , w0 devem ser todos m
ultiplos de 2n , qualquer que seja
n 1. Entao x0 = y0 = z0 = w0 = 0, absurdo.
Exemplo 11. (Extrado de [3]) Encontre todas as quadr
uplas (x, y, z, k) de n
umeros inteiros,
com x, y, z > 0 e k 0, tais que
x6 + y 6 + z 6 = 4826 7k .
Vamos mostrar o seguinte fato:
(x, y, z, k) e soluc
ao, com k 1

(x/7, y/7, z/7, k 6) e solucao,


e nesse caso k 6.

(=) Temos x6 + y 6 + z 6 0 (mod 7). Como x6 , y 6 , z 6 0 ou 1 (mod 7), devemos ter


x, y, z m
ultiplos de 7. Da, 76 |4826 7k 76 |7k k 6. Ademais, vale a igualdade
 x 6
7

 y 6
7

 z 6
7

= 4826 7k6 ,

ou seja, (x/7, y/7, z/7, k 6) tambem e solucao.


(=) Claro.
O fato acima garante que podemos ir subtraindo 6 de k e retirando um fator 7 de x, y,
z enquanto k 1, ate que o expoente de 7 no lado direito da igualdade seja 0. Em outras
palavras, existe n 0 tal que k = 6n, com x = 7n x0 , y = 7n y0 , z = 7n z0 , e
x0 6 + y0 6 + z0 6 = 4826.
A equacao acima s
o tem a soluc
ao (1, 3, 4) e suas permutacoes. Assim, as solucoes da
equacao original s
ao (7n , 3 7n , 4 7n , 6n), n 0, e suas permutacoes nas tres primeiras
coordenadas.

Problemas Propostos
Problema 12. Mostre que se a b = x2 e mdc(a, b) = 1 ent
ao existem r e s tais que a = r 2
e b = s2 .
1
1
1
Problema 13. Prove que todas as soluco
es positivas da equaca
o 2 + 2 = 2 com
x
y
z
mdc(x, y, z) = 1 s
ao dadas por
(x, y, z) = (r 4 s4 , 2rs(r 2 + s2 ), rs(r 2 s2 ))
ou
(x, y, z) = (2rs(r 2 + s2 ), r 4 s4 , rs(r 2 s2 )),
onde r > s > 0, mdc(r, s) = 1 e r e s de paridades opostas.
Problema 14. Encontre todos os pares de racionais (x, y) tais que x2 + y 2 = 1.
Problema 15. Resolva simultaneamente em inteiros positivos:
a2 + b2 = c2
a2 + c2 = d2
onde a, b, c e d s
ao inteiros positivos relativamente primos entre si dois dois.
Problema 16. (Torneio das Cidades 1997) Prove que a equaca
o
x2 + y 2 z 2 = 1997
tem infinitas soluco
es inteiras (x, y, z).
Problema 17. Encontre todas as soluco
es inteiras de x2 + y 2 + z 2 = t2 .
Problema 18. Encontre todas as soluco
es de 5m2 + n2 = 52011
Problema 19. Encontre todas as soluco
es em n
umeros naturais m e n da equaca
o:
m2 = 1 + 2 + . . . + n.

Refer
encias
[1] F. E. Brochero Martinez, C. G. Moreira, N. C. Saldanha, E. Tengan - Teoria dos
N
umeros: um passeio com primos e outros n
umeros familiares pelo mundo inteiro,
Projeto Euclides, IMPA, 2010.
[2] E. Carneiro, O. Campos and F. Paiva, Olimpadas Cearenses de Matematica 1981-2005
(Nveis J
unior e Senior), Ed. Realce, 2005.
[3] S. B. Feitosa, B. Holanda, Y. Lima and C. T. Magalhaes, Treinamento Cone Sul 2008.
Fortaleza, Ed. Realce, 2010.
[4] D. Fomin, A. Kirichenko, Leningrad Mathematical Olympiads 1987-1991, MathPro
Press, Westford, MA, 1994.
[5] D. Fomin, S. Genkin and I. Itenberg, Mathematical Circles, Mathematical Words, Vol.
7, American Mathematical Society, Boston, MA, 1966.
[6] I. Niven, H. S. Zuckerman, and H. L. Montgomery, An Introduction to the Theory of
Numbers.

Polos Olmpicos de Treinamento


Aula

Curso de Teoria dos Nmeros - Nvel 2

13

Prof. Samuel Feitosa

Equaco
es Diofantinas III
Ja estudamos as equacoes diofantinas lineares e equacoes em que alguma fatoracao conveniente poderia facilitar a busca por solucoes. Nesta aula, estaremos interessados em encontrar
m
odulos convenientes para analisar os termos de uma equacao.
Exemplo 1. Encontre todas as soluco
es em inteiros da equaca
o x2 7y = 1004.
Analisando os restos na divisao por 7, obtemos x2 3 (mod 7). Entretando, os u
nicos
inteiros que s
ao restos de quadrados perfeitos na divisao por 7 s
ao 0, 1, 2 e 4. Como
3 1004 (mod 7) nao faz parte dessa lista, nao existem soulcoes inteiras para a equacao.
Exemplo 2. Encontre todas as soluco
es em inteiros da equaca
o x3 + 98y 2 + 5 = 0.
Analisemos os possveis restos de x3 (mod 7) fazendo uma tabela dos restos correspondentes
de x e x3 :
x
x3

0
0

1
1

2
1

3
6

4
1

5
6

6
6

Como os u
nicos restos possveis s
ao 0, 1, 1 (mod 7), o lado esquerdo da equacao s
o pode
deixar resto 5, 6, 4 (mod 7). Como o resto do lado direito nao faz parte dessa lista, nao
existem solucoes em inteiros.
Exemplo 3. Prove que a equaca
o x2 = 3y 2 + 8 n
ao possui soluco
es em inteiros x e y.
Analisando o resto na divisao por 3, obtemos x2 2 (mod 3). Como os u
nicos restos de
um quadrado por 3 s
ao 0 e 1, nao existem solucoes em inteiros.
Nos proximo problema, usaremos congruencias para encontrarmos informacoes sobre as
incognitas envolvidas nos expoentes e buscaremos alguma fatoracao apropriada para reduzir
o problema `a resolucao de um sistema de equacoes.
Exemplo 4. Encontre todas as soluco
es em inteiros positivos da equaca
o 3m + 7 = 2n

POT 2012 - Teoria dos N


umeros - Nvel 2 - Aula 13 - Samuel Feitosa

Analisando o resto m
odulo 3 do lado esquerdo, podemos concluir que 2n 1 (mod 3).
n
n
Como 2 (1) (mod 3), conclumos que n e par, ou seja, n = 2k, para algum k N.
Assim, como o lado direito e m
ultiplo de 4, podemos concluir que:
3m 7 (mod 4)

(1)m 1 (mod 4)

Logo, m e par, ou seja, m = 2t, para algum t N. Usando diferenca de quadrados,


podemos escrever:
7 = (2k 3t )(2k + 3t ).
Como 7 e primo, temos as seguintes opcoes:
7 = 2k + 3t 2k 3t = 1

1 = 2k + 3t 2k 3t = 7

Em ambos os casos, 8 = 2k+1 e da k = 2. Substituindo nas equacoes, obtemos solucao


apenas no primeiro caso com t = 1. Assim, (m, n) = (2, 4).
Exemplo 5. Encontre todas as soluco
es em inteiros positivos da equaca
o 3 2m + 1 = n2 .
Analisandoa equacao m
odulo 3, n2 1 (mod 3) e assim, n 1 (mod 3). No primeiro
caso, se n = 3k + 2, temos 3 2m + 1 = n2 = 9k 2 + 12k + 4 e da 2m = (3k + 1)(k + 1).
Como o lado esquerdo possui apenas um fatores 2, temos 3k + 1 = 2i , k + 1 = 2j ,com
j i. Da, 3 2j 2i = 2. Se j = i, temos 2i+1 = 2 e consequentemente i = 0 produzindo
k = 0 e (m, n) = (0, 2). Se j < i, temos j = 1 pois o lado esquerdo possui um u
nico fator
2 e por conseguinte, i = 2, (m, n) = (3, 5). No segundo caso, quando n = 3k + 1, e tratado
analogamente produzindo apenas a nova solucao (m, n) = (4, 7).
Exemplo 6. Encontre todas as soluco
es da equaca
o x2 xy + y = 3 em inteiros x, y.
Fixado o valor de y, podemos encontrar os valores de x usando a formula de B
askara. Como
x e inteiro, o discriminante y 2 4(y 3) = (y 2)2 + 8 deve ser um quadrado perfeito,
digamos z 2 . Assim,
z 2 (y 2)2 = (z y + 2)(z + y 2) = 8.
Como z y + 2 e z + y 2 possuem a mesma paridade, o produto anterior dever (2) (4).
Em qualquer caso, somando ambos os termos, obtemos 2z = 6 e z = 3. Logo, y2 = 1.
Substituindo os valores de y na equacao original, obtemos os valores correspondentes para
x. As solucoes s
ao: (x, y) = (2, 1), (1, 1), (0, 3), (3, 3).

Exemplo 7. (Hungria 1969) Seja n um inteiro positivo. Prove que se 2 + 2 28n2 + 1 e um


inteiro, ent
ao e um quadrado perfeito.

POT 2012 - Teoria dos N


umeros - Nvel 2 - Aula 13 - Samuel Feitosa

Necessariamente
perfeito. Assim,

28n2 + 1 deve ser racional e para isso 28n2 + 1 deve ser um quadrado
28n2 + 1 = t2


t1
t+1
2
7n =
2
2

Como 7 e primo, 7 |

t+1
t1
ou que 7 |
. No primeiro caso,
2
2



t1
t+1
2
n +1 =
14
2

Alem disso, como mdc((t 1)/2, (t + 1)/2) = 1, existem a e b tais que


t+1
14
t1
2

a2 =
b2 =

Da, 7a2 b2 = 1 e b2 1 (mod 7). Como quadrados perfeitos s


o podem deixar restos
0, 1, 2, 4 (mod 7), esse caso nao gera solucoes. No segundo caso,
t+1
2
t1
.
14

a2 =
b2 =

Logo, 2 + 2 28n2 + 1 = 2 + 2t = 2 + 2(2a2 1) = (2a)2 .


Exemplo 8. (Reino Unido 1996) Encontre todas as soluco
es em inteiros n
ao negativos
x, y, z da equaca
o:
2x + 3y = z 2 .
Se y = 0, entao 2x = z 2 1 = (z 1)(z + 1). Analisando a fatoracao em primos, existem
i, j, com i > j, tais que z + 1 = 2i e z 1 = 2j . A diferenca das duas equacoes produz
2 = 2i 2j = 2j (2ij 1). Como o lado esquerdo possui apenas um fator 2, j = 1 e
i j = 1. Nossa primeira solucao encontrada e (x, y, z) = (3, 0, 3). Se y > 0, 2x z 2
(mod 3). Como 2x 1 (mod 3) e z 2 0, 1 (mod 3) temos, 2x z 2 1. Isso implica
que x e par, ou seja, x = 2m. Fatorando, obtemos:
3y = z 2 22m

= (z 2m )(z + 2m )

Novamente, analisando a fatoracao em primos, existem l e k, com l < k, tais que z 2m =


3l , z + 2m = 3k . A diferenca das duas equacoes produz 2m+1 = 3l (3kl 1). Novamente
3

POT 2012 - Teoria dos N


umeros - Nvel 2 - Aula 13 - Samuel Feitosa

analisando a fatoracao em primos, l = 0 e 2m+1 = 3k 1. Se m = 0, temos k = 1 e


(x, y, z) = (0, 1, 2). Se m > 0,
3k = 1 (mod 4)
(1)k = 1 (mod 4).
e devemos ter k par, ou seja, existe t tal que k = 2t. Fatorando novamente, 2m+1 =
(3k 1)(3k + 1). Escrevendo 3k + 1 = 2p e 3k 1 = 2q , temos 2 = 2q (2pq 1). Veja
que ja tratamos essa equacao no incio e assim podemos concluir que q = 1 e p q = 1.
Produzindo a solucao (x, y, z) = (4, 2, 5).
Nos proximos dois problemas, contruiremos solucoes indutivamente.
Exemplo 9. (Bulg
aria) Prove que para qualquer n
umero natural n 3, existem n
umeros
naturais mpares xn e yn tais que 7x2n + yn2 = 2n .
Para n = 3, basta tomar x1 = y1 = 1. Suponha que tenhamos encontrado xk e yk mpares,
satisfazendo
7x2k + yk2 = 2k .
Um dos n
umeros (xk + yk )/2, (xk yk )/2 e mpar e assim podemos escolher um deles de
modo a satisfazer o enunciado para k + 1:




x k yk 2
x k yk 2
7
+
= 2(7x2k + yk2 ) = 2k+1 .
2
2
Exemplo 10. Mostre que existe uma sequencia infinita de inteiros positivos a1 , a2 , . . . tais
que a21 + a22 + . . . + a2n e um quadrado perfeito para todo n natural.
Definamos a1 = 3. Suponha que a sequencia ja esteja definida para a1 , a2 , . . . , ak com
a21 + a22 + . . . + a2k = (2t + 1)2 .
Vejamos que podemos definir o proximo termo de modo que a soma de todos os primeiros
k + 1 termos ao quadrado ainda seja um quadrado perfeito de um inteiro mpar. Basta
fazer ak+1 = 2t2 + 2t. Veja que:
a21 + a22 + . . . + a2k + a2k+1 = (2t + 1)2 + (2t2 + 2t)2
= (2t2 + 2t + 1)2 .
que e novamente o quadrado de um mpar.

Problemas Propostos
Problema 11. Encontre todas as soluco
es em inteiros x, y, z, t da equaca
o:
x2 + y 2 + z 2 = 8t 1.
4

POT 2012 - Teoria dos N


umeros - Nvel 2 - Aula 13 - Samuel Feitosa

Problema 12. Encontre todas as soluco


es em inteiros positivos da equaca
o
1 1 1
+ + = 1.
a b
c
Problema 13. Encontre todas as soluco
es em inteiros de x2 y 2 1988
Problema 14. Mostre que para todo inteiro z, existem inteiros x e y satisfazendo x2 y 2 =
z3
Problema 15. Encontre todas as soluco
es de 1 + x + x2 + x3 = 2y em inteiros positivos x
e y.
Problema 16. Mostre que a equaca
o diofantina 5m2 6mn + 7n2 = 1988, n
ao possui
soluca
o nos inteiros.
Problema 17. (R
ussia 1996) Sejam x, y, p, n, k n
umeros naturais tais que
x n y n = pk .
Prove que se n > 1 e mpar, e p e um primo mpar, ent
ao n e uma potencia de p.
Problema 18. (R
ussia 1997) Encontre todas as soluco
es inteiras da equaca
o
(x2 y 2 )2 = 1 + 16y.
Problema 19. (OBM 2009) Prove que n
ao existem inteiros positivos x e y tais que x3 +y 3 =
2009
2
.

Ap
endice: A conjectura de Catalan
Em alguns dos problemas anteriores, nos deparamos com a quest
ao de encontrarmos duas
potencias perfeitas consecutivas nao triviais. As u
nicas solucoes que apareceram foram
23 = 8 e 32 = 9. Em 1844, Eug`ene Catalan conjecturou que essa seria a u
nica solucao.
Recentemente, tal conjectura se mostrou verdadeira atr
aves do:
Teorema 20. (Mihalnescu - 2002)Existe uma u
nica soluca
o nos n
umeros naturais de
xa y b = 1,
com x, a, y, b > 1 que e (x, y, a, b) = (3, 2, 2, 3).
Problema 21. Encontre toda as soluco
es em inteiros positivos da seguinte equaca
o diofantina:
2y 2 = x4 + x.

Polos Olmpicos de Treinamento


Curso de Teoria dos Nmeros - Nvel 2

Aula

14

Prof. Samuel Feitosa

A Func
ao Parte Inteira - I

O jogo de Wythoff

O objetivo da aula de hoje e resolver o seguinte problema:


Exemplo 1. Dois jogadores jogam alternadamente removendo pedras de duas pilhas sobre
uma mesa. Na sua vez, cada jogador pode remover qualquer quantidade de pedras de uma
pilha ou igual n
umero de pedras de ambas as pilhas. O ganhador e aquele que retirar a
u
ltima pedra. Determine todas posico
es perdedoras.
Uma boa estrategia para identificar as posicoes perdedoras nesse jogo, e associar o movimento dos jogadores ao movimento de uma peca em um tabuleiro. Suponha que inicialmente as duas pilhas possuem 5 e 7 pedras. Colocaremos uma peca no canto direito
superior em um tabuleiro 8 6. O movimento de retirar x pedras da coluna de 5 sera
traduzido como um deslocamento vertical de x casas para baixo, enquanto que a mesma
retirada da outra coluna sera traduzido como um movimento horizontal para a esquerda
de mesmo deslocamento. Um movimento de retirada de x pedras de ambas as colunas
sera traduzido como um deslocamento diagonal da direita para a esquerda dessa mesma
quantidade de casas. O jogo terminar
a quando a peca chegar na casa do canto extremo
esquerdo simbolizando que ambas as colunas estao com 0 pedras.

A posicao (0, 0) e perdedora porque uma vez que um jogador a receba, ele ter
a perdido
o jogo. Qualquer posicao do tipo (x, 0), (0, x) ou (x, x), com x > 0, sera uma posicao
vencedora. Marquemos essas posicoes no tabuleiro:

EFeitosa
POT 2012 - Teoria dos N
umeros - Nvel 2 - Aula
2 14
DEFINIC
- Samuel
AO
PROPRIEDADES
+
+
+
+
+
+
+
+
++
+++++++
As proximas posicoes perdedoras que encontramos s
ao (1, 2),(2, 1). A partir dessa nova
posicao, podemos preencher o tabuleiro com as novas posicoes vencedoras.
+
+
+
+
+

+
+
+
+

+
+
+

+
+

+
+
+
+
+

+
+
+
+
+
+

++
+
+++
+++
+++

As proximas posicoes perdedoras que encontramos s


ao (3, 5) e (5, 3). Como existe simetria
entre as duas pilhas, basta procurarmos as posicoes perdedoras (x, y) com x < y. Repetindo
o processo anterior, podemos listar as primeiras posicoes perdedoras ordenadas (xn , yn ) com
x n < yn .
n
xn
yn

0
0
0

1
1
2

2
3
5

3
4
7

4
6
10

5
8
13

6
9
15

7
11
18

8
12
20

9
14
23

10
16
26

11
17
28

12
19
31

As proximas secoes nos ajudarao a estabelecer alguma padr


ao entre os valores de (xn , yn )
em funcao de n.

Definicao e Propriedades

Definic
ao 2. A parte inteira de um n
umero real x e o maior inteiro x que n
ao e maior
que x. Definimos a parte fracion
aria {x} de x por {x} = x x. (exemplos: 3 =
3 , 3, 5 = 3 e 4, 7 = 5)
Teorema 3. Sejam x e y n
umeros reais. Ent
ao:
1. x x < x + 1 e 0 {x} < 1.
2. x + m = x + m se m e um inteiro.
3. x + y x + y x + y + 1.
j k  
x
se m e um inteiro positivo.
4. x
= m
m
5. Se n e a s
ao inteiros positivos,
divisveis por a.

jnk
a

e o n
umero de inteiros entre 1, 2, ..., n que s
ao

EFeitosa
POT 2012 - Teoria dos N
umeros - Nvel 2 - Aula
2 14
DEFINIC
- Samuel
AO
PROPRIEDADES

Demonstraca
o. Os primeiros dois itens decorrem facilmente da definicao e serao deixados
a cargo do leitor.
Para provar (3), veja que:
x + y x + y + {x} + {y}
= x + y + {x} + {y}
= x + y.

Como {x} + {y} < 2, {x} + {y} 1 e da:


x + y = x + y + {x} + {y}
x + y + 1.

Para provar (4), seja x = qm + r com 0 r < m 1, entao:



 j
x
rk
= q.
= q+
m
m
Como 0 {x} < 1,
 
 j k
qm + r + {x}
x
r + {x}
=
=
.
q=q+
m
m
m


Finalmente, para provar (5), sejam a, 2a, ..., ja todos os inteiros positivos n que s
ao
divisveis por a. Entao,
n
<j+1
jan k
.
j=
a

ja n < (j + 1)a j

Observac
ao 4. Em alguns dos problemas desta seca
o, ser
a usada a notaca
o de somat
orio.
Recomenda-se que o professor escrevar por extenso os primeiros somat
orios ate que os
alunos se sintam confort
aveis com a manipulaca
o dos ndices.
Teorema 5 (F
ormula de Polignac). Seja p um primo. Ent
ao o maior expoente p na fatoraca
o em primos de n! e:
 
X
n
tp (n) =
.
pi
i=1

n
? Ele conta o n
umero de inteiros positivos menores ou
pi
iguais a n divisveis por pi . Cada m
ultiplo de p contribui com um expoente 1 para p em n!,
cada m
ultiplo de p2 contribui com expoente 2 para p em n! e assim sucessivamente. Entao,
 
X
n
e a soma de todas essas contribuicoes (veja que um m
ultiplo de pi e contado i
pi
i=1
j k j k
j k
vezes em np , pn2 , ..., pni ).
Demonstraca
o. O que significa

EFeitosa
POT 2012 - Teoria dos N
umeros - Nvel 2 - Aula
2 14
DEFINIC
- Samuel
AO
PROPRIEDADES

Observac
ao 6. Se p e primo e p e a maior potencia de p que divide n, usaremos a notaca
o

p k n.
Exemplo 7. Em quantos zeros termina a representaca
o decimal de 1000!?
Para determinarmos o n
umero de zeros, basta determinarmos a maior potencia de 10 que
divide 1000!. Existem mais fatores 2 do que fatores 5 e assim bastar
a encontrarmos o
expoente de 5 na fatoraca
o de 1000!. Pelo teorema anterior, tal n
umero e:


X
1000
5i

i=1

Exemplo 8. Mostre que

k1 
X
i=0

i
x+
k

= 200 + 40 + 8 + 1 = 249.

= kx.

j+1
j
b<
. Ent
ao kb = j.
k
k
Alem disso, b + i/k = 0 se i < k j e b + j/k = 1 se k j i k 1. Da,
Sejam a = x, b = {x} e j {0, 1, . . . , k 1} tal que
k1
X
i=0

b + i/k =

k1
X

i=kj

b + i/k

= j
= kb
Exemplo 9. Mostre que x + y + x + y 2x + 2y.
Sejam a = {x} e b = {y}. A desigualdade e equivalente `
a
2x + 2y + a + b + a + b 2x + 2y + 2a + 2b.
que pode ser reescrita como:
a + b + a + b 2a + 2b.
Temos que 0 a + b < 2 e a = b = 0. Se a + b = 1 segue que pelo menos um
dentre a, b e maior ou igual `
a 1/2. Da,
a + b + a + b = 1

2a + 2b.

Caso contr
ario, a + b + a + b = 0 e a desigualdade segue.
Exemplo 10. Mostre que se m e n s
ao inteiros positivos, ent
ao
inteiro.

(2m)!(2n)!
e um
(m)!(n)!(m + n)!

EFeitosa
POT 2012 - Teoria dos N
umeros - Nvel 2 - Aula
2 14
DEFINIC
- Samuel
AO
PROPRIEDADES

Pelo teorema anterior, basta mostrarmos que:


       


2n
m
n
m+n
2m
+ k k + k +
,
pk
p
p
p
pk
para todo primo p e todo inteiro k. A desigualdade segue do exemplo anterior.

Exemplo 11. Prove que 2n
n divide M M C{1, 2, ..., 2n}.


+1 , pelo teorema anterior, temos:


Seja p um primo. Se p k 2n
n e p 2n < p
=



X
2n
j=1

pj



X
2n
j=1

pj

n
2 j
p
n
2 j
p

,
pois 2x 2x = 0 ou 1. Como p k M M C{1, 2, ..., 2n} e , segue o resultado.
Exemplo 12. Mostre que


X
n
k=1

2i


1
= n.
2

O n
umero de inteiros que s
ao m
ultiplos de 2k , mas n
ao de 2k+1 e n/2k n/2k+1 . Se
n = 2k+1 q + r, esse n
umero e q + r/2k . Para x [0, 1), e verdade que 2x = x + 1/2.
Assim,
n/2k n/2k+1 = q + r/2k

= r/2k + 1/2 + q

= r/2k+1 + 1/2

Somando a quantidade de n
umeros que s
ao m
ultiplos de 2k mas n
ao de 2k+1 entre 1 e n
para k = 0, 1, . . . obtemos a quantidade total de n
umeros, ou seja,

X
n
k=1


1
+
= n.
2i 2

Teorema 13. Seja vp (n) a soma dos dgitos da representaca


o de n na base p. Mostre que
n vp n
.
o expoente de p na fatoraca
o em primos de n! e
p1
Demonstraca
o. Seja kp (n!) o maior expoente de p que divide n!. Considere a representacao
de n na base p: n = d0 + d1 p + ... + dr pr onde 0 di < p. Entao,

EFeitosa
POT 2012 - Teoria dos N
umeros - Nvel 2 - Aula
2 14
DEFINIC
- Samuel
AO
PROPRIEDADES

 
n
= d1 + d2 p + . . . + dr pr1 ,
p
 
n
= d2 + d3 p + . . . + dr pr2 ,
p2
  ...
n
= dr .
p
Somando tudo, obtemos:
kp (n!) = d1 + d2 (p + 1) + d3 (p2 + p + 1) + + dr (pr1 + + 1)

1
=
d1 (p 1) + d2 (p2 1) + + dr (pr 1)
p1
n vp (n)
=
p1
Exemplo 14. Seja B(m) o conjunto dos inteiros r tais que 2r e um termo na representaca
o
na base 2 de n. Por exemplo, B(100) = {2, 5, 6} pois 100 = 22 + 25 + 26 . Prove que nk e
mpar se, e somente se, B(k) B(n).
Aproveitando a notacao dos teoremas anteriores, t2 (n) = n v2 (n). Assim,
 
n
1 (mod 2) [n v2 (n)] [(k v2 (k)) ((n k) v2 (n k))] = 0
k
v2 (k) + v2 (n k) v2 (n) = 0
v2 (k) + v2 (n k) = v2 (n).

Au
ltima equacao nos diz que na soma das representacoes na base 2 de n k e k nao ocorre
a operacao de vai um, ou seja, B(k) B(n).

Observac
ao 15. Esse exemplo tambem mostra que nk 0 (mod 2) para todo k {1, 2, . . . , n
1} se, e somente se, n e uma potencia de 2.
Exemplo 16. (Olimpada Rioplatense) Seja r um real tal que

 



19
20
92
r+
+ r+
+ + r +
= 554.
100
100
100
Calcule 100r .
Sejam r = a e {x} = b. Podemos reescrever a equaca
o como:
74a +

73 
X
i=0


19 + i
b+
= 554.
100

EFeitosa
POT 2012 - Teoria dos N
umeros - Nvel 2 - Aula
2 14
DEFINIC
- Samuel
AO
PROPRIEDADES



19 + i
Como 0 b +
< 2 para todo i {0, 1, . . . , 73}, segue que 74a 554 74a + 74.
100
N
ao podemos ter 554 = 74(a + 1) pois 74 554. Logo, a < 554/74 < a + 1 e por conseguinte
a = 554/74 = 7. Assim,

73 
X
19 + i
= 554 74 7 = 36.
b+
100
i=0

Pelo primeiro exemplo,


99 
X


i
= j.
100
i=0


j+1
19
j
b<
. N
ao podemos ter r +
= 1 pois nesse caso teramos
onde
100
100
100

73 
X
19 + i
r+
73 6= 36.
100
i=0




i
i
= 0 para todo i 19. Como r +
= 1 se 100 j i 99, temos:
Logo, r +
100
100
b+

100b = j

99 
X
i
r+
=
100
i=0
 



92 
X
i
93
99
r+
=
+
r+
... r +
100
100
100
i=19

= 36 + 7
= 43.
Finalmente,
100r = 100a + 100b
= 700 + 43
= 743.
Exemplo 17. Prove que existe um natural n tal que a representaca
o decimal de n2 comeca
( da esquerda para a direita ) com o n
umero 201120112011. . . 2011 ( 2011 vezes).
Podemos encontrar n tal que n2 comece com qualquer sequencia de dgitos (c1 c2 ...cr ) = m.

ao,
Tome k suficientemente grande tal que 2 m < 10k1 . Seja n = 10k m + 1. Ent

0 < 10k m < n 10k m + 1

102k m < n2 102k m + 2.10k m + 1


102k m < n2 102k m + 102k1 + 1
102k m < n2 < 102k (m + 1).

Assim, n2 comeca com a sequencia de dgitos m.


7

EFeitosa
POT 2012 - Teoria dos N
umeros - Nvel 2 - Aula
2 14
DEFINIC
- Samuel
AO
PROPRIEDADES

Exemplo 18. (OBM 1999) Prove que h


a pelo
menos um algarismo diferente de zero entre
os a vrgula.
a 1000000a e a 3000000a casa decimal de 2 ap

6
6
6
6
Suponha que n
ao, ent
ao 10210 1010 2 = 10310 2. Se k = 1010 2, temos:
6
6
6
10210 k 10310 2 < 10210 k + 1

1
k
k
<
2 < 106 + 3106
6
10
10
10
10
k2
2k
1
k2
6 < 2 <
6 +
6 +
210
210
410
610
10
10
10
10 6

6
2k
2 21010
1
Como 2106 <
,
6
210
2
10
10
6

k 2 < 2 10210 < k 2 + 1


6

0 < 2 10210 k 2 < 1.


6

Isso e um absurdo pois 2 10210 k 2 Z.


Exemplo 19. ( Sequencia de Beatty) Se e s
ao irracionais satisfazendo
as sequencias
, 2, 3, . . . ;

1
1
+

= 1 ent
ao,

e
, 2, 3, . . . ;
incluem todos os n
umeros naturais exatamente uma vez.
Primeiramente, provemos a unicidade. Suponha que k = l = n, como e s
ao
irracionais, n < k < n + 1 e n < l < n + 1. Assim,
l
k
+
n+1 n+1

<
<

1
1
+

l
k
+
n n

k+l
k+l
<1<
. Temos um absurdo pois a desigualdade anterior diz
n+1
n
que o inteiro k + l est
a entre dois inteiros consecutivos.

Isso nos diz que

Mostremos agora que todo natural aparece nas sequencias. Dado n N, existe k Z+ tal
que
1
k
k1
< < .
n

Samuel
POT 2012 - Teoria dos N
umeros - Nvel 23- Aula
A SOLUC
14-AO
DO PROBLEMA
Feitosa INICIAL

Dividamos o intervalo [k/(n+1), k/n] em duas partes. Se


1
k
k1
< <
, temos:
Se por outro lado,
n

n+1

k1
1
k
< < , temos k = n.
n

k1
1
k
<1 <

n+1
1
n+1k
n+1k
< <

n+1

n
(n + 1 k) = n
Em qualquer caso, n faz parte da sequencia.

A solucao do problema inicial

Voltemos ao exemplo inicial. Veja que toda linha ou coluna do tabuleiro deve possuir no
m
aximo uma posica
o perdedora. Se a k-esima coluna n
ao possuir nenhuma posica
o perdedora, para cada uma de suas casinhas, poderemos encontrar um posica
o perdedora na
mesma linha ou diagonal e isso implicaria na existencia de uma infinidade de posico
es
perdedoras entre as k 1 primeiras colunas. O mesmo argumeto se aplica para as linhas.
Consequentemente, cada natural aparece exatamente uma vez dentre os termos da sequencia
x0 , x1 , . . . ; y0 , y1 , . . .. Alem disso, tambem e f
acil concluir que as sequencias (xn ) e (yn ) s
ao
crescentes. Induzidos do exemplo anterior, isso nos leva a conjecturar a existencia de dois
irracionais e que possam de alguma forma gerar os termos das posico
es perdedoras.
A inserca
o dos pontos (xn , yn ) em um gr
afico sugere que esses pontos est
ao pr
oximos a
alguma reta. Isso poderia ser
traduzido
dizendo
que
o
quociente
y
/x

e
pr
o
ximo
a algum
n
n

1+ 5
valor. De fato, se =
, xn n e yn ( + 1)n. Como as posico
es s
ao n
umeros
2
inteiros, podemos conjecturar que:

1+ 5
, ent
ao (xn , yn ) = (n , n ( + 1))
Conjectura 20. Se =
2
Veja que e irracional e que:
1
1
+
+1

2 + 1
2 +
2 + 1
=
2 + 1
= 1.
=

Provemos a afirmaca
o anterior por induca
o. Ela e facilmente verific
avel para os casos
iniciais apresentados na primeira tabela. Suponha sua validade para todos os inteiros no
conjunto {0, 1, 2, . . . , k}. Provemos que o mesmo tambem pe v
alido para k + 1. Seja t o
menor natural que n
ao est
a no conjunto {x0 , x1 , . . . , xk , y0 , y1 , . . . , yk }. Como as sequencias
9

Samuel
POT 2012 - Teoria dos N
umeros - Nvel 23- Aula
A SOLUC
14-AO
DO PROBLEMA
Feitosa INICIAL

xn e yn s
ao crescentes e xn < yn , se xk+1 6= t, o inteiro t n
ao aprecer
a entre os termos
das sequencias e isso contradiz nossa observaca
o inicial. Em virtude da unicidade de repsentaca
o da sequencia de Beatty, o inteiro (k + 1) ainda n
ao apareceu dentre os termos
das k + 1 primeiras posico
es perdedoras. Se t < (k + 1), como xn e crescente, deve
existir j tal que ( + 1)j = t com j > k. Nesse caso,
t = ( + 1)j

( + 1)(k + 1)

= (k + 1) + k + 1

> (k + 1)

Contrariando a suposica
o inicial sobre t. Logo, devemos ter xk+1 = (k + 1). Seja
l = yk+1 xk+1 . Se l < k + 1, O movimento diagonal
(xk+1 , yk+1 ) (xl , yk+1 xk+1 + xl ) = (xl , yl ),
passa uma posica
o perdedora para contradizendo o fato de que (xk+1 , yk+1 ) era uma posica
o
perdedora. Se l > k + 1, yk+1 > ( + 1)(k + 1) e o jogador naquela posica
o pode remover
pedras de apenas uma pilha obtendo:
(xk+1 , yk+1 ) ((k + 1), ( + 1)(k + 1)).
Usando novamente hip
otese de induca
o e lembrando que yi xi 6= k + 1 para todo i
{0, 1, . . . , k}, qualquer movimento do pr
oximo jogador, conduzir
a a uma posica
o em que exatamente uma das pilhas possui um n
umero de pedras igual a um dos n
umeros x0 , y0 , . . . , xk , yk .
Assim, o oponente poder
a passar uma posica
o perdedora. Novamente temos um absurdo
pois (xk+1 , yk+1 ). Logo, yk+1 xk+1 = k + 1 e consequentemente yk+1 = ( + 1)(k + 1)
concluindo a prova da conjectura para k + 1.

Problemas Propostos
Problema 21. Mostre que a parte fracion
aria do n
umero

4n2 + n n
ao e maior que 0, 25.

Problema 22. Sejam {ai }0ir , inteiros n


ao negativos com n = a1 + a2 + + ar . Mostre
n!
que
e um inteiro.
a1 !a2 !...ar !


X
n + 2i1
Problema 23. Prove que, para qualquer n natural,
= n.
2i
i=1
 

2n
, com p 2n. Ent
ao o expoente
Problema 24. Seja p um divisor primo do n
umero
  n
2n
e igual a 1.
de p na fatoraca
o em primos do do n
umero
n
10

Samuel
POT 2012 - Teoria dos N
umeros - Nvel 23- Aula
A SOLUC
14-AO
DO PROBLEMA
Feitosa INICIAL

Problema 25. (Coreia 1997) Expresse

Pn

k=1

k em termos de n e n.

Problema 26. (Canad


a 1998) Determine o n
umero de soluco
es reais da equaca
o
jak jak jak
+
+
= a.
2
3
5

Problema 27. Encontre todos o reais tais que a igualdade n+ n + = 4n + 1


e verdadeira para todos os naturais n.
Problema 28. Se a, b, c s
ao reais e na + nb = nc para todo n natural, ent
ao a Z
ou b Z.
Problema 29. Sejam a, b, c e d n
umeros reais. Suponha que an + bn = cn + dn
para todos os inteiros positivos n. Mostre que pelo menos um dentre a + b, a c, a d e
inteiro.
Problema 30. Seja n 3 um inteiro positivo. Mostre que e possvel eliminar no m
aximo
dois elementos do conjunto {1, 2, . . . , n} de modo que a soma dos n
umeros restantes seja
um quadrado perfeito.
m1
X  ax + b 
Problema 31. Sejam a, m, b inteiros dados, com mdc(a, m) = 1. Calcule
.
m
x=0

Problema 32. Encontre todos os naturais n tais que

2n1

| n!.

Problema 33. Determine os pares (a, b) de reais tais que abn = ban para todo inteiro
positivo n.
 k
p
Problema 34. Se p e primo , ent
ao
0 (mod p)( para 1 i pk 1).
i



Problema 35. Prove que ( 3 n + 3 n + 2)3 e divisvel por 8.


jnk
jnk jnk
+
+ ... +
, onde tn e o n
umero
Problema 36. Prove que, t1 + t2 + . . . + tn =
1
2
n
de divisores do natural n.
   
n
n
e divisvel

Problema 37. Prove que, se p e um n


umero primo, ent
ao a diferenca
p
p
por p.

11

Polos Olmpicos de Treinamento


Aula

Curso de Teoria dos Nmeros - Nvel 2

15

Prof. Samuel Feitosa

Equaco
es de Pell

Uma Equacao Famosa


Vejamos um problema que servira de motivacao ao nosso estudo.
Exemplo 1. Sejam Fn e Ln as seq
uencias de Fibonacci e Lucas, respectivamente, definidas
por
F1 = 1, F2 = 1

L1 = 1, L2 = 3

Fn+1 = Fn + Fn1 , n 2,

Ln+1 = Ln + Ln1 , n 2.

Mostre que a equaca


o 5x2 y 2 = 4 admite uma soluca
o (x, y) em inteiros positivos se, e
somente se, (x, y) = (F2n1 , L2n1 ) para algum natural n.
Note que (1, 1) e a u
nica solucao com y 2. Podemos supor, portanto, que y 3. Sendo
, as duas razes da equacao x2 x 1 = 0, e conhecido que
Fn =

n n

Ln = n + n .

trivial verificar que os pares (F2n1 , L2n1 ) satisfazem nossa equacao. A parte difcil e
E
mostrar que essas s
ao as u
nicas solucoes. Seja
S = {(F2n1 , L2n1 ), n 1}.
Por absurdo, suponha que exista uma solucao (x, y) 6 S, e tome aquela que minimiza o
valor de x. Como x e y tem a mesma paridade, as fracoes (3x y)/2 e (3y 5x)/2 s
ao
inteiros positivos, pois
x2 > 1 9x2 > 5x2 4
2

y >5

9x2 > y 2
e

3x > y

9y > 5y + 20 9y 2 > 25x2 3y > 5x.

POT 2012 - Teoria dos N


umeros - Nvel 2 - Aula 15 - Samuel Feitosa

Afirmamos que ((3x y)/2, (3y 5x)/2) e uma solucao da equacao que nao esta em S. De
fato,




3x y 2
3y 5x 2 20x2 4y 2
5

=
= 4,
2
2
4
e, se ((3x y)/2, (3y 5x)/2) S, existiria n para o qual

3x y
= F2n1
2
x = F2n+1

e
e

3y 5x
= L2n1
2
y = L2n+1 ,

o que contraria o fato de (x, y) nao estar em S. Para terminar, note que (3x y)/2 < x,
ou seja, obtemos uma solucao cuja primeira coordenada e menor que x. Pelo metodo da
Descida de Fermat, conclumos que todas as solucoes est
ao em S.
Exemplo 2. (Vietn
a 1999) A seq
uencia an e definida por a1 = 1, a2 = 2 e an+2 =
3an+1 an , n 1. A seq
uencia bn e definida por b1 = 1, b2 = 4 e bn+2 = 3bn+1 bn ,
n 1. Mostre que os inteiros positivos (a, b) satisfazem 5a2 b2 = 4 se, e somente se,
mdc(an , bn ) = mdc(a, b).
Veja que encontramos uma famlia infinita de solucoes para o problema anterior. Curi
osamente, essa famlia satisfaz uma recorrencia linear bem simples. Note ainda que 5
apareceu na formula que encontramos para F2n+1 e L2n+1 . Ser
a que tudo isso foi coincidencia? Nosso proximo objetivo sera estudar mais detalhadamente equacoes como a do
problema anterior.
A equacao x2 dy 2 = N , onde d e natural e N e inteiro, e chamada equaca
o de Pell. Jonh
Pell contribuiu muito pouco para a an
alise desta equacao. Ela recebeu seu nome apenas
por um engano de Euler. Lagrange foi o primeiro a provar que, se d nao e um quadrado
perfeito, entao x2 dy 2 = 1 tem infinitas solucoes. Estamos interessados em descrever
todas as possveis solucoes desta equacao, caso existam, e tentar obter alguns criterios para
dizer quando ela nao tem solucao. Trataremos apenas do caso em que d nao e um quadrado
perfeito. O outro caso e deixado como exerccio para o leitor.
Observac
ao 3. (Para professores) Caso os alunos n
ao estejam preparados para o formalismo nas pr
oximas demonstraco
es, o professor poder
a ater-se apenas aos problemas. Bastaria o aluno entender como se caracterizam as soluco
es de uma equaca
o de Pell. Ap
os
alguma amadurecimento,ficaria mais f
acil estudar as provas e entender porque as soluco
es
s
o podem ser daquela forma.
A proxima proposicao e um conhecido exerccio de princpio da casa dos pombos.
Proposic
ao 4. Se e um n
umero irracional, existem infinitos n
umeros racionais x/y, com
mdc(x, y) = 1, tais que




x < 1

y y2
2

POT 2012 - Teoria dos N


umeros - Nvel 2 - Aula 15 - Samuel Feitosa

Demonstraca
o. Considere a particao


 


1
1 2
n1
[0, 1) = 0 ,
,
, 1 .


n
n n
n
Pelo princpio da casa dos pombos, dois dentre os n
umeros 0, {}, {2}, . . ., {n} estao em
um mesmo intervalo da particao, digamos


i i+1
,
, 0 k < j n e 0 i < n.
{k}, {j}
n
n
Entao
|{j} {k}| <
|(j k) (j k)| <




x <

y

1
n
1
n
1
,
ny

onde x = j k e y = j k. Seja d = mdc(x , y ), com x = dx e y = dy,


mdc(x, y) = 1. Temos






x = x < 1 < 1 < 1

y
y ny
y2
y 2

Para obter infinitas solucoes, tome m > 1/ | x/y| 6= 0. Com o mesmo argumento acima,
particionando agora o intervalo [0, 1) em m intervalos de mesmo tamanho, obtemos um par
de inteiros (x1 , y1 ) primos entre si, 0 < y1 < m, tais que








x 1 < 1 < x

y1 my1
y
e portanto x1 /y1 e uma aproximacao racional de melhor do que x/y.
Dizemos que um inteiro n e livre de quadrados se nao existe nenhum natural k > 1 tal que
k 2 |n.

Proposic
ao 5. Seja d um inteiro positivo livre de quadrados. Existe uma constante M tal
que a desigualdade
2

x dy 2 < M
tem infinitas soluco
es inteiras positivas (x, y).

claro que, se d e livre de quadrados, entao d e irracional. Pela proDemonstraca


o. E
posicao anterior,
existem infinitos pares de inteiros (x, y), com y > 0 e mdc(x, y) = 1, tais

que |x y d| < 1/y. Aplicando a desigualdade triangular, temos


1

x + y d x y d + 2y d < + 2y d.
y
3

POT 2012 - Teoria dos N


umeros - Nvel 2 - Aula 15 - Samuel Feitosa

Multiplicando as duas desigualdades encontradas, obtemos





2

x dy 2 < 1 1 + 2y d 2 d + 1,
y y
o que conclui a demonstracao.

Se (x, y) e solucao da equacao de Pell, e claro que (x, y) tambem e. Suporemos, a partir
daqui, que x, y > 0.
Teorema 6. Se d e um inteiro positivo livre de quadrados, ent
ao a equaca
o x2 dy 2 = 1 tem
infinitas soluco
es inteiras. Ademais, existe uma soluca
o (x1 , y1 ), chamada
ofun de soluca
damental, tal que toda soluca
o e da forma (xn , yn ), n 1, onde xn + yn d = (x1 + y1 d)n .
Demonstraca
o. Vamos mostrar a existencia de uma
2 soluc2a o. Pela proposicao anterior,
x dy < M . Existe, portanto, um
existem infinitos pares de inteiros (x, y) tais
que

2
odulo
inteiro m, 1 m M , tal que a equacao x dy 2 = m tem infinitas solucoes. M
m, cada solucao esta no conjunto finito



Zm Zm = 0, 1, . . . , m 1 0, 1, . . . , m 1 ,

e da, com um argumento an


alogo ao anterior, existe um par (a, b) Zm Zm tal que
uma quantidade infinita de solucoes (x, y) satisfaz (x, y) = (a, b). Em particular, existem
solucoes distintas (x1 , y1 ) e (x2 , y2 ) tais que (x1 , y 1 ) = (x2 , y 2 ). Temos




d
d
x
+
y
x

y
1
1
2
2
x 1 + y1 d

=
m
x 2 + y2 d

(x1 x2 dy1 y2 ) + (x2 y1 x1 y2 ) d


=
m

= u + v d,

onde u, v s
ao inteiros, pois
x1 x2 dy1 y2 x1 2 dy1 2
0
x 2 y 1 x 1 y2 x 1 y 1 x 1 y 1 0

(mod m)
(mod m) .

Ademais,

 

u+v d uv d

x 1 + y1 d x 1 y1 d

=
x 2 + y2 d x 2 y2 d
x1 2 dy1 2
=
x2 2 dy2 2
= 1,

u2 dv 2 =

POT 2012 - Teoria dos N


umeros - Nvel 2 - Aula 15 - Samuel Feitosa

mostrando a existencia de solucao. Vejamos agora como s


ao as solucoes.

Diremos que uma solucao (x1 , y1 ) e maior que uma solucao (x2 , y2 ) se x1 +y1 d > x2 +y2 d.
Seja (x1 , y1 ) a menor solucao, com x1 , y1 > 0. Chamaremos esta soluc
ao de fundamental.

Dada outra solucao (x, y), vamos mostrar que existe n para o qual x + y d = (x1 + y1 d)n .
De fato, se esse nao e o caso, para algum n valem as seguintes desigualdades


n+1
n
,
x 1 + y1 d < x + y d < x 1 + y1 d

e da
1 <
1 <
1 <

n

x + y d x 1 + y1 d
<

n

x + y d x 1 y1 d

A+B d

<

x 1 + y1 d

x 1 + y1 d

< x1 + y1 d,

onde (A, B) e uma solucao formada por inteiros positivos (prove!). Issocontraria a minimi
lidade do par (x1 , y1 ). Assim, toda solucao (x, y) deve satisfazer x + y d = (x1 + y1 d)n ,
para algum n 1, e uma facil verificacao mostra que esses pares s
ao de fato solucoes da
equacao.

Exemplo 7. Encontre todos os tri
angulos cujos lados s
ao inteiros consecutivos e cuja a
rea
e inteira.
Sejam a = n 1, b = n, c = n + 1 os lados do tri
angulo. Pela formula de Herao, a area do
tri
angulo e
A=

np 2
1p
(a + b + c)(b + c a)(a + c b)(a + b c) =
3(n 4).
4
4

A e inteiro se e somente se n e par e 3(n2 4) e um quadrado perfeito. Substituindo n


por 2x e A por ny, obtemos a equacao 3x2 3 = y 2 . Entao y e divisvel por 3, digamos
y = 3z, e da a equacao anterior equivale `a equacao de Pell x2 3z 2 = 1. A solucao
fundamental dessa u
ltima e (x1 , z1 ) = (2, 1), donde todas as outras solucoes s
ao geradas
pelas recorrencias xn+1 = 2xn + 3zn e zn+1 = xn + 2zn , n 1. Os tri
angulos procurados
s
ao os que tem lados de medidas 2xn 1, 2xn e 2xn + 1, cuja area e 3xn zn .
Exemplo 8. Encontre o menor inteiro positivo n para o qual 19n + 1 e 95n + 1 sejam ambos
quadrados perfeitos.
Sejam 19n + 1 = x2 e 95n + 1 = y 2 . Entao 5x2 y 2 = 4, que e a equacao analisada no
problema 45. Suas solucoes s
ao os pares (F2m1 , L2m1 ), m 1. Para que n seja mnimo,
5

POT 2012 - Teoria dos N


umeros - Nvel 2 - Aula 15 - Samuel Feitosa

basta calcular o menor n


umero da seq
uencia de Fibonacci m
ultiplo de 19, que e F17 = 1597.
Assim,
F17 2 1
n=
= 134232.
19
Exemplo 9. Dado n > 1, mostre que existe um conjunto S de n pontos no plano tal que:
1. N
ao existem tres pontos de S colineares;
2. A dist
ancia entre quaisquer dois pontos de S e um inteiro.
Como a equacao de Pell x2 2y 2 = 1 admite a solucao (1, 1), ela admite infinitas. Sejam
entao a1 , a2 , . . . , an inteiros satisfazendo as seguintes condicoes:
(i) ai 2 + 1 = 2bi 2 , i = 1, 2, . . . , n;
(ii) a1 < a2 < < an .
Tome S = {P1 , P2 , . . . , Pn }, onde P1 , P2 , . . . , Pn s
ao pontos sobre a circunferencia unit
aria
definidos por


ai 2 1
2ai
,
, i = 1, 2, . . . , n.
Pi =
ai 2 + 1 ai 2 + 1
Um calculo simples mostra que a dist
ancia entre quaisquer dois pontos de S e racional.
Multiplicando as coordenadas desses pontos pelo mnimo m
ultiplo comum dos denominadores de todas as fracoes das dist
ancias entre dois pontos, obtemos um conjunto de n
pontos sobre uma mesma circunferencia, em particular nao existindo tres colineares, tais
que a dist
ancia entre quaisquer dois deles e inteira.
Problema 10. (IMO 1975) Existem 1975 pontos sobre uma circunferencia de raio 1 de
modo que a dist
ancia entre quaisquer dois desses pontos e racional?
Problema 11. Seja n 3 um inteiro. Mostre que existe um conjunto S de n pontos no
plano tal que a dist
ancia entre quaisquer dois pontos de S e irracional e a a
rea de qualquer
tri
angulo com vertices em S e racional.
Problema 12. Prove que a equaca
o x2 dy 2 = 1 n
ao tem soluca
o se d e divisvel por um
primo da forma 4k + 3.
Problema 13. Sejam d, k inteiros positivos tais que d n
ao e um quadrado perfeito. Mostre
que existem infinitos pares de inteiros positivos (x, y) tais que k|y e x2 dy 2 = 1.

POT 2012 - Teoria dos N


umeros - Nvel 2 - Aula 15 - Samuel Feitosa

Outros Resultados
Para o leitor familiarizado
com fracoes contnuas, basta sabermos
as n-esimas convergencias

da expans
ao de d para determinarmos as solucoes de x2 dy 2 = 1, como diz a proposicao
abaixo:

Proposic
ao 14. Todas as soluco
es de x2 dy 2 = 1 podem ser encontradas em xn = hn
hn
s
ao as n-esimas convergencias da expans
ao em fraco
es contnuas de
, yn = kn , onde
kn

d. Se r e o perodo da expans
ao em fraco
es contnuas de d temos:

ao tem soluca
o e todas soluco
es positivas de
i) Se r
e par ent
ao x2 dy 2 = 1 n
2
2
x dy = 1 s
ao dadas por x = hnr1 , y = knr1 para n = 1, 2, 3, . . ..

2 dy 2 = 1
ii) Se r mpar ent
ao x = hnr1 , y = knr1 produzem todas
as
solu
c
o

es
de
x

quando n = 1, 3, 5, . . .., e todas as soluco


es de x2 dy 2 = 1 quando n = 2, 4, 6 . . ..
Proposic
ao 15. (Equacao de Pell generalizada)Consideremos ax2 by 2 = c, onde a e b n
ao
s
ao simultaneamente iguais a 1 e eles tambem n
ao s
ao divisveis por nenhum quadrado,
ent
ao as soluco
es s
ao obtidas da seguinte maneira: Se c = 1 , e ambos a e b s
ao diferentes
de 1, determine primeiro a soluca
o fundamental se existir.
Todas as outras
soluco
es s
ao

obtidas da soluca
o fundamental (x0 , y0 ) por xn a + yn b = (x0 a + y0 b)2n+1 . Se c 6= 1
, primeiro determine a soluca
o fundamental da equaca
o ax2 by 2 = 1. Se a ou b e igual
a 1, ent
ao esta equaca
o tem no m
aximo um soluca
o fundamental (x0 , y0 ). Se (x0 ,
y0 ) e

2
2
uma soluca
ofundamental deax by = c todas as outras s
ao obtidas de xn a + yn b =

(x0 a + y0 b) (x0 a + y0 b). Se nem a oub for igual a 1, as soluco


es s
ao geradas por

xn a + yn b = (x0 a + y0 b)2n (x0 a + y0 b).


Corol
ario 16. Suponha que N e um inteiro n
ao nulo e que d seja livre de quadradados. Se
x2 dy 2 = N tem uma soluca
o, ent
ao tem infinitas.

hn
as n-esimas
Proposic
ao 17. Seja d um inteiro que n
ao e um quadrado perfeito, sejam
kn

convergencias da expans
ao em fraca
o contnua de d. SejaN um inteiro tal que |N | < d.
Ent
ao qualquer soluca
o positiva de x = s, y = t de x2 dy 2 = N com mdc(s, t) = 1
satisfaz s = hn , t = kn para algum n.

Problemas Propostos
Problema 18. Mostre que existem infinitos inteiros n para os quais n2 + (n + 1)2 e um
quadrado perfeito.
Problema 19. Dado um inteiro positivo k, mostre que n
ao existem inteiros (x, y) tais que
2
2
2
x (k 1)y = 1.

POT 2012 - Teoria dos N


umeros - Nvel 2 - Aula 15 - Samuel Feitosa

Problema 20. Mostre que, se d 1 (mod 4), ent


ao a equaca
o x2 dy 2 = 1 tem soluca
o.
Problema 21. (Banco IMO 2002) Existe um inteiro positivo m para o qual a equaca
o
1 1 1
1
m
+ + +
=
a b
c abc
a+b+c
tem infinitas soluco
es inteiras positivas a, b, c?
Problema 22. Determine todos os pares (k, n) de inteiros positivos tais que
1 + 2 + + k = (k + 1) + (k + 2) + + n.
Problema 23. Encontre todos os n
umeros da forma m(m+1)/3 que s
ao quadrados perfeitos.
Problema 24. Encontre todos os n
umeros triangulares que s
ao quadrados perfeitos.
Problema 25. Resolva a equaca
o (x + 1)3 x3 = y 2 em inteiros positivos.
Problema 26. Encontre todos os inteiros positivos n para os quais 2n + 1 e 3n + 1 sejam
ambos quadrados perfeitos e mostre que todos esses inteiros s
ao divisveis por 40.
Problema 27. Seja n um inteiro positivo tal que 3n + 1 e 4n + 1 s
ao ambos quadrados
perfeitos. Mostre que n e divisvel por 56.
Problema 28. Prove que existem infinitos inteiros positivos n para os quais n2 +1 divide n!.
Problema 29. Prove que a equaca
o
x2 + y 2 + z 2 + 2xyz = 1
admite infinitas soluco
es inteiras positivas (x, y, z).
Problema 30. Encontre todos os inteiros positivos n para os quais existem inteiros positivos
distintos a1 , a2 , . . . , an tais que
1
2
n
a1 + a2 + + an
+
+ +
=

a1 a2
an
n
Problema 31. (Vietn
a 1992) Encontre todos os pares de inteiros positivos (x, y) satisfazendo a equaca
o
x2 + y 2 5xy + 5 = 0.
8


POT 2012 - Teoria dos N
umeros - Nvel 2 - Aula 15 - Samuel Feitosa
REFERENCIAS

Problema 32. Encontre todos os naturais n para os quais n + 1 e 3n + 1 s


ao ambos quadrados perfeitos.
Problema 33. Encontre todos os pares de naturais (m, n) satisfazendo a igualdade
1 + 2 + 3 + + n = m2 .
Problema 34. (Banco IMO 1967)
o p/q, onde p, q s
ao inteiros positivos menores
Qual fraca
que 100, e a mais pr
oxima de 2? Encontre todos os dgitos ap
os a vrgula da representa
ca
o

decimal dessa fraca


o que coincidem com os dgitos da representaca
o decimal de 2.
Problema 35. (Banco IMO 2003) Seja b > 5 um inteiro. Para cada natural n, seja xn a
representaca
o na base b do n
umero
11
1} 22
2} 5.
| {z
| {z
n

n1

Prove que a seguinte condica


o e verdadeira se e somente se b = 10:
Existe um inteiro positivo n0 tal que, para cada n > n0 , o n
umero xn e um quadrado
perfeito.
Problema 36. (Torneio das Cidades 1997) Prove que a equaca
o
x2 + y 2 z 2 = 1997
tem infinitas soluco
es inteiras (x, y, z).
Problema 37. (Irlanda 1995) Determine todos os inteiros a para os quais a equaca
o
x2 + axy + y 2 = 1
tem infinitas soluco
es inteiras positivas (x, y) tais que x 6= y.

Refer
encias
[1] F. E. Brochero Martinez, C. G. Moreira, N. C. Saldanha, E. Tengan - Teoria dos
N
umeros ? um passeio com primos e outros n
umeros familiares pelo mundo inteiro,
Projeto Euclides, IMPA, 2010.
[2] E. Carneiro, O. Campos and F. Paiva, Olimpadas Cearenses de Matem
atica 1981-2005
(Nveis J
unior e Senior), Ed. Realce, 2005.
[3] S. B. Feitosa, B. Holanda, Y. Lima and C. T. Magalhaes, Treinamento Cone Sul 2008.
Fortaleza, Ed. Realce, 2010.
9


POT 2012 - Teoria dos N
umeros - Nvel 2 - Aula 15 - Samuel Feitosa
REFERENCIAS

[4] D. Fomin, A. Kirichenko, Leningrad Mathematical Olympiads 1987-1991, MathPro


Press, Westford, MA, 1994.
[5] D. Fomin, S. Genkin and I. Itenberg, Mathematical Circles, Mathematical Words, Vol.
7, American Mathematical Society, Boston, MA, 1966.
[6] I. Niven, H. S. Zuckerman, and H. L. Montgomery, An Introduction to the Theory of
Numbers.

10

Polos Olmpicos de Treinamento


Curso de Teoria dos Nmeros - Nvel 2

Aula

18

Prof. Samuel Feitosa

Resduos Quadr
aticos
Definic
ao 1. Para todos a tais que mdc(a, m) = 1, a e chamado resduo quadr
atico m
odulo
m se a congruencia x2 a (mod m) tem soluca
o. Se ela n
ao tem soluca
o, ent
ao a e
chamado de resduo n
ao quadr
atico m
odulo m.

ao e um
Exemplo 2. Seja n um inteiro. Prove que se 2 + 2 28n2 + 1 e um inteiro, ent
quadrado perfeito.

Se 2+2 28n2 + 1 e inteiro, o n


umero 28n2 + 1 e um racional e consequentemente devemos
ter que 28n2 + 1 e o quadrado de um inteiro mpar, digamos:
28n2 + 1 = (2k + 1)2

28n2 + 1 = 4k 2 + 4k + 1
7n2 = k(k + 1)

Devemos considerar dois casos: 7 | k ou 7 | k+1. Alem disso, lembremo-nos do seguinte fato:
Se mdc(a, b) = 1, e a b = n2 entao existem inteiros x e y tais que a = x2 e y = b2 .
Como mdc(k, k + 1) = 1, temos os dois casos para analisar:
Primeiro caso:

k
= x2
(k + 1)/7 = y 2
Assim, 1 = (k + 1) k = 7y 2 x2 . Analisando essa equacao m
odulo 7, temos x2 1
(mod 7). Entretanto, analisando os quadrados dos restos da divisao por 7, podemos notar
que 1 nao e um resduo quadratico e consequentemente temos um absurdo.
Segundo caso:


k/7 = x2
k + 1 = y2

POT 2012 - Teoria dos N


umeros - Nvel 2 - Aula 18 - Samuel Feitosa

Da, 2 + 2 28n2 + 1 = 2 + 2(2k + 1) = 4(k + 1) = (2y)2 e isso conclui o problema.


Em geral, se p e um primo da forma 4k + 3, 1 nunca e resduo quadratico. Para ver isso,
suponha que existe x tal que:
x2 1 (mod p)

(x2 )(p1)/2 (1)(p1)/2


x

p1

(mod p)

1 (mod p).

Isso contradiz o teorema de Fermat.


 
a
Definic
ao 3. Se p denota um primo mpar, ent
ao o smbolo de Legendre
e definido
p
por 1 se a e um resduo quadr
atico, 1 se a n
ao e um resduo quadr
atico m
odulo p, e 0 se
p|a.
Teorema 4. Se p e um primo mpar. Ent
ao
 
p1
a
a)
a 2 (mod p)
p
    
b
ab
a
=
b)
p
p
p
   
a
b
c) a b (mod p) implica que
=
p
p
 2
 2   
a
a b
b
d) Se mdc(a, p) = 1 ent
ao
=1e
=
p
p
p
 
 
p1
1
1
e)
= 1,
= (1) 2
p
p
Provemos inicialmente o item a) quando mdc(a, p) = 1. Em virtude do teorema de Fermat,
perceba que se mdc(a, p) = 1 entao:

Da, ap1/2

p | ap1 1 = (ap1/2 + 1)(ap1/2 1).


 
a
1 (mod p). Suponha que
= 1, entao existe x tal que
p

(x2 )

x2 a

p1
2

xp1 a

(mod p)

p1
2
p1
2

p1

Pelo teorema
de Fermat, a u
ltima congruencia nos diz que a 2 1 (mod p). Suponha
 
a
= 1, ou seja, que nao existe x tal que x2 a (mod p). Assim, podemos
agora que
p
2

POT 2012 - Teoria dos N


umeros - Nvel 2 - Aula 18 - Samuel Feitosa

separar os n
umeros do conjunto {1, 2, . . . , p 1} em pares (i, j) onde ij a (mod p) e
i 6= j. Da, o produto de todos esses pares e
1 2 3 . . . (p 1) a a . . . a (mod p)
a

p1
2

Usando o teorema de Wilson, conclumos que a

p1
2

p1
2

(mod p)
 
a
1 (mod p). Se p | a,
0
p

(mod p). Os demais itens seguem de a).

Exemplo 5. Suponha que p e um primo mpar. Seja n o menor n


ao-resduo quadr
atico

positivo m
odulo p. Prove que n < 1 + p.
Seja m o maior inteiro positivo tal que mn > p, ou seja, (m 1)n < p < mn. Assim,
0 < mn p < n e consequentemente:


mn p
1 =
p


mn
=
p
   
m
m
=

p
p
 
m
=
p
Da, m n e
(n 1)2 < n(n 1)

n(m 1)
< p.

Portanto, n 1 <

p.

Teorema 6. (Lema de Thue) Sejam m um n


umero natural e a um inteiro primo com m,
ent
ao existem inteiros x e y tais que:

1. 0 < |x|, |y| < m;


2. ax y (mod m).

Demonstraca
o. Considere o conjunto {au v|u, v Z, 0 u, v p}. Como existem

ao (u1 , v1 ) 6= (u2 , v2 ) tais que


p2 > p tais pares (u, v), existir
au1 v1 au2 v2

(mod p)

Sejam x = v1 v2 e y = u1 u2 . Claramente ii) esta satisfeito. Por construcao, x, y nao


podem ser ambos nulos e, caso um deles seja, o outro tambem o sera. Logo i) tambem e
verdade.
3

POT 2012 - Teoria dos N


umeros - Nvel 2 - Aula 18 - Samuel Feitosa

Proposic
ao 7. Sejam D Z e m N inteiros relativamente primos tais que D e um
resduo quadr
atico m
odulo m. Ent
ao existem inteiros k, x, y Z com 0 < k D e

0 < |x|, |y| < p tais que:


x2 + Dy 2 = kp
Demonstraca
o. Seja a tal que a2 D (mod p) e x, y como no teorema anterior com
m = p. Entao, por um lado:
0 < x2 + Dy 2 < (1 + D)p
e por outro lado,
x2 + Dy 2 (a2 + D)y 2 0 (mod p)
 
3
= 1, existem x e y tais que
Exemplo 8. Seja p > 3 um primo mpar tal que
p
x2 + 3y 2 = p.

Pelo teorema anteiror, Exitem x, y, k tais que x2 + 3y 2 = pk com |x|, |y| p. Assim,
x2 + 3y 2 < 4p. Temos tres casos a considerar:
Primeiro caso: x2 + 3y 2 = 3p. Devemos ter x2 0 (mod 3) e x = 3x0 . Da, 3x20 + y 2 = p.
Segundo caso: x2 + 3y 2 = 2p. Como 2p e par, devemos ter x e y ambos mpares ou ambos
pares. Em qualquer caso, x2 + 3y 2 sera m
ultiplo de 4 e consequentemente 2 | p. Isso e um
absurdo.
Terceiro caso: x2 + 3y 2 = p. N
ao ha o que fazer nesse caso.
Teorema 9. (Lema de Gauss ) Seja p um primo mpar e a um inteiro tal que mdc(a, p) = 1,
a(p 1)
e seus restos m
odulo p. Se n denota o n
umero
Considere os inteiros a, 2a, . . . ,
2 
a
= (1)n
desses restos que excedem p2 ent
ao
p
Demonstraca
o. Sejam r1 , r2 , . . . rn os resduos que excedem p/2 e sejam s1 , s2 , . . . , sk os
resduos restantes. Naturalmente todos esses restos s
ao distintos e nenhum deles e nulo.
Considere agora os n
umeros da forma p ri e perceba que 0 < p ri < p/2. Se tivessemos
p ri sj (mod p) para algum par (i, j), tambem teramos ri + sj 0 (mod p) e por cona(p 1)
seguinte p dividiria a soma de dois n
umeros do conjunto {a, 2a, . . . ,
}. Entretanto,
2
isso e um absurdo porque a soma de quaisquer dois n
umero desse conjunto e da forma ak
com 0 < k < p e a nao e divisvel por p. Logo, os n
umeros da forma p rj s
ao todos
distintos dos n
umeros da forma si e todos eles pertencem ao conjunto {1, 2, . . . (p 1)/2}.

POT 2012 - Teoria dos N


umeros - Nvel 2 - Aula 18 - Samuel Feitosa

Como n + k = (p 1)/2, podemos concluir que:


p1

2
p1
(mod p)
1 2 ...
2
p1
1 2 ...
(mod p)
2
p1
1 2...
(mod p)
2
1 (mod p)

(p r1 )(p r2 ) . . . (p rn )s1 s2 . . . sk = 1 2 . . .
(r1 )(r2 ) . . . (rn )s1 s2 . . . sk
(1)n r1 r2 . . . rn s1 s2 . . . sk
p1
a
2
(1)n a(p1)/2

(1)n a 2a . . .

(1)n a(p1)/2

(mod p).

Pelo criterio de Euler, o resultado segue.


 
a
Teorema 10. Se p e um primo mpar e mdc(a, 2p) = 1, ent
ao
= (1)t onde t =
p
p1
  
2 
X
p2 1
2
ja
e
= (1) 8 .
p
p
j=1

a(p 1)
} e usaremos a
Demonstraca
o. Consideraremos novamente o conjunto {a, 2a, . . . ,
2
mesma notacao do teorema anterior. Quando o inteiro ja e dividido por p, obtemos como
quociente o n
umero ja/p. Assim, podemos escrever:
(p1)/2

(p1)/2

ja =

j=1

pja/p +

j=1

n
X

rj +

j=1

k
X

sj

j=1

e
(p1)/2

X
j=1

j =

n
X
i=1

(p ri ) +

= np

n
X

rj +

n
X

sj +

k
X

sj

k
X

sj

j=1

j=1

j=1

j=1

Substituindo na equacao anterior, obtemos:

(p1)/2
(p1)/2
n
X
X
X

(a 1)
j = p
ja/p n + 2
rj
j=1

Como

j=1

(p1)/2

j=

j=1

p2 1
,
8

j=1

POT 2012 - Teoria dos N


umeros - Nvel 2 - Aula 18 - Samuel Feitosa

temos:
(p1)/2

p2 1
(a 1)
8

j=1

ja/p n

(mod 2)

(p1)/2

Se a e mpar, n

X
j=1

ja/p (mod 2). Se a = 2, isto implica que n (p2 1)/8 (mod 2)

pois 2j/p = 0 para 1 j (p 1)/2. O resultado decorre do teorema anterior.


Exemplo 11. Encontre todos os inteiros positivos n tais que 2n 1 e divisvel por 3 e
tem um m
ultiplo da forma 4m2 + 1 para algum natural m.

2n 1
3

Teorema 12. (Lei da reciprocidade quadr


atica) Se p e q s
ao primos mpares distintos, ent
ao
  
p1 q1
p
q
= (1) 2 2
q
p
Demonstraca
o. Seja S o conjunto de todos os pares de inteiros (x, y) satisfazendo 1 x
(p 1)/2, 1 y (q 1)/2. O conjunto S possui (p 1)(q 1)/4. Suponha que exista
um par (x, y) tal que qx = py. Como mdc(p, q) = 1, segue que q | y e p | x. Entretanto,
nos internvalos mencionados nao existem tais m
ultilplos. Separemos entao esse conjunto
em dois outros mutuamente exclusivos:
S1 = {(x, y)|qx > py}

S2 = {(x, y)|qx < py}


S1

S2
b

(q1)/2

(p1)/2

Os n
umeros de pares em S1 e S2 s
ao

qx/p e

x=1

X
y=1

py/q. Fazendo a contagem

total de pares, temos:


(p1)/2

(q1)/2

x=1

qx/p +

y=1

py/q =

p1 q1

2
2

e, em virtude do teorema anterior, obtemos:


  
p1 q1
p
q
= (1) 2 2
q
p
6

POT 2012 - Teoria dos N


umeros - Nvel 2 - Aula 18 - Samuel Feitosa

Exemplo 13. Mostre que

x2 2
nunca e um inteiro quando x e y s
ao inteiros.
2y 2 + 3

Exemplo 14. Seja q = 4n + 1 onde n e um inteiro positivo. Prove que q e um primo se, e
q1
somente se, 3 2 1 (mod q)
Se q e um primo, entao q 2 (mod 3) e pela lei da reciprocidade quadratica temos:
1 = (1)(q1)/21
  
3
q
=
q
3
 
3
=
(1)
q
Em virtude dessa equacao e do criterio de Euler, temos:
 
3
1 =
q
3

q1
2

(mod q)

q1

Reciprocamente, se 3 2 1 (mod q), entao ordq 3 = 4n . Como ordq 3 | (q), teremos


(q) = q 1, ou seja, q e primo.

Problemas Propostos
Problema 15. Prove que se p e um primo maior que 3 ent
ao a soma dos resduos quadr
aticos
m
odulo p e divisvel por p.
Problema 16. Mostre que se a e um resduo quadr
atico m
odulo m, e ab 1 (mod m),
ent
ao b e tambem um resduo quadr
atico. Prove que o produto dos resduos quadr
aticos
m
odulo p e congruente a +1 ou 1 m
odulo p.
Problema 17. Prove que se p e um primo da forma 4k + 3, e se m e o n
umero de resduos
quadr
aticos menores que p2 , ent
ao:
1 3 5 . . . (p 2) (1)m+k+1 (mod p)
2 4 6 . . . (p 1) (1)m+k (mod p)

Problema 18. Seja q = 4n + 1 onde n e um inteiro positivo. Prove que q e um primo se,
q1
e somente se, 3 2 1 (mod q)
Problema 19. Os inteiros positivos a e b s
ao tais que os n
umeros 15a + 16b e 16a 15b
s
ao ambos quadrados de inteiros positivos. Qual e o menor valor possvel que pode ter o
menor desses n
umeros?

POT 2012 - Teoria dos N


umeros - Nvel 2 - Aula 18 - Samuel Feitosa

Problema 20. (Olimpada B


ulgara) Sejam m e n n
umeros naturais tais que
A=

(m + 3)n + 1
3m

e um inteiro. Prove que A e mpar.


Problema 21.
a) Prove que para qualquer primo p, o n
umero

2p
p

2 e divisvel por p2 .

b) Mostre que se p e um primo e 0 m < n < p ent


ao


np + m
(1)m+n+1 p (mod p2 )
mp + n
c) Prove que para qualquer primo p > 3, o n
umero

2p1
p1

1 e divisvel por p3 .

Problema 22. Caracterize todos os inteiros que podem ser expressos na forma:
a) a2 + ab + b2
b) a2 + 2b2
Problema 23. Se n e um inteiro tal que 7n e da forma a2 + 3b2 , prove que n tambem e
dessa forma.
Problema 24. Encontre todos os inteiros positivos n para os quais existe um inteiro m tal
que m2 + 9 e um m
ultiplo de 2n 1.
Problema 25. Mostre que dado qualquer primo p, existem inteiros x, y, z, w satisfazendo
x2 + y 2 + z 2 wp = 0 e 0 < w < p

Problema 26. Mostre que p e um divisor de ambos os n


umeros da forma m2 + 1, n2 + 2,
se e somente se e um divisor de algum n
umero da forma k 4 + 1.

Problema 27. Seja A o conjunto de todos os inteiros da forma a2 + 2b2 , onde a e b s


ao
inteiros e b 6= 0. Mostre que p e um n
umero primo e p2 A, ent
ao p A.
Problema 28. Seja p um primo da forma 4k + 1. Mostre que:

p1 
X
2k 2
k=1

k2
2
p




p1
.
2

Problema 29. Mostre que se x n


ao e divisvel por 3, ent
ao 4x2 + 3 tem pelo menos um
fator primo da forma 12n + 7. Mostre que existem infinitos primos dessa forma.
Problema 30. Suponha que (5m 1) = 5n 1 com m, n n
umeros naturais. Prove que
mdc(m, n) > 1
Problema 31. (Coreia 2001) Seja f : Z Z. Dado um primo mpar p, encontre todas as
funco
es f : Z Z satisfazendo as seguintes condico
es:
8


POT 2012 - Teoria dos N
umeros - Nvel 2 - Aula 18 - Samuel Feitosa
REFERENCIAS

1. Se m n (mod p) com m, n Z, ent


ao f (m) = f (n)
2. f (mn) = f (m)f (n) para quaisquer m, n Z.
Problema 32. Para a congruencia z 2 D (mod 2a ), onde D e mpar e a e um natural, ser
sol
uvel, e neces
ario e suficiente que D seja da forma 2k + 1, 4k + 1 ou 8k + 1 de pendendo
de a = 1, a = 2 ou a > 2.
Problema 33. (OBM 2007) Para quantos numeros inteiros c, 2007 c 2007 , existe
um inteiro x tal que x2 + c e m
ultiplo de 22007 ?
Problema 34. (Teorema de Wolstenhome) Se p 5 e um primo, mostre que o numerador
da fraca
o
1 1
1
+ + ... +
1 2
p1

e m
ultiplo de p2 .


2p
Problema 35. Se p e um primo maior que 3 e q =
, prove que
3
   
 
p
p
p
+
+ ...
1
2
q
e divisvel por p2 .
(Dica: Use a identidade de Catal
ao e o teorema de Wolstenhome)

Refer
encias
[1] E. Carneiro, O. Campos and F. Paiva, Olimpadas Cearenses de Matem
atica 1981-2005
(Nveis J
unior e Senior), Ed. Realce, 2005.
[2] S. B. Feitosa, B. Holanda, Y. Lima and C. T. Magalhaes, Treinamento Cone Sul 2008.
Fortaleza, Ed. Realce, 2010.
[3] D. Fomin, A. Kirichenko, Leningrad Mathematical Olympiads 1987-1991, MathPro
Press, Westford, MA, 1994.
[4] D. Fomin, S. Genkin and I. Itenberg, Mathematical Circles, Mathematical Words, Vol.
7, American Mathematical Society, Boston, MA, 1966.
[5] I. Niven, H. S. Zuckerman, and H. L. Montgomery, An Introduction to the Theory of
Numbers.

Polos Olmpicos de Treinamento


Curso de Teoria dos Nmeros - Nvel 2

Aula

19

Prof. Samuel Feitosa

A Func
ao Parte Inteira - II

Exemplo 1. Considere um tabuleiro T , de dimens


oes m n, onde m e n s
ao inteiros positivos. Prove que uma diagonal de T passa por exatamente m + n mdc(m, n) quadradinhos
1 1.
Suponhamos os quadradinhos de lado unit
ario. Vamos fazer primeiro o caso em que
mdc(m, n) = 1. Esse tabuleiro em Z Z pode ser reperesentado por um ret
angulo de
vertices : O = (0, 0), A = (m, 0), B = (m, n), C = (0, n). Queremos porvar que a diagonal OB passa por exatamente m + n 1 quadradinhos. Quando esta diagonal corta um
quadradinho, um segmento de reta dela, fica totalmente contido no quadradinho. Basta
contarmos em quantos segmentos esses quadradinhos dividem OB.
Se um vertice, digamos (a, b), de algum dos quadradinhos do tabuleiro esta em OB, usando
semelhanca de tri
angulos podemos concluir que:
m
b
n
an = bm
a =

Como m|an e mdc(m, n) = 1, temos m | a resultando que a = 0 ou a m. No primeiro caso (a, b) = O e no segundo, como a m pois a esta no inteiror do ret
angulo,
temos (a, b) = B. Assim, OB nao contem vertices de quadradinhos diferentes de O e B.
Consequentemente OB corta cada uma das retas x = 1, 2, ..., m 1 e y = 1, 2, ..., n 1
em pontos distintos determinando assim m + n 2 pontos sobre OB. Juntando esses
m + n 2 pontos marcados sobre a diagonal com O e B teremos m + n pontos e por
conseguinte OB corta m + n 1 quadradinhos. Agora se mdc(m, n) = d podemos escrever
m = dm1 , n = dn1 com mdc(m1 , n1 ) = 1. Divida agora o tabuleiro em d sub-tabuleiros :
Ti = {Oi = ((i 1)m1 , (i 1)n1 ), Ai = (im1 , 0), Bi = (im1 , in1 ), Ci = (0, in1 )} para
1 i d. Basta usar o que fizemos para cada um desses sub-tabuleiros.

POT 2012 - Teoria dos N


umeros - Nvel 2 - Aula 19 - Samuel Feitosa

Exemplo 2. Suponha que mdc(p, q) = 1. Ent


ao
q1  
X
ip
i=1

p1  
X
iq
i=1

(p 1)(q 1)
.
2

Prova: Considere o ret


angulo T = O = (0, 0), A = (q, 0), B = (q, p), C = (0, p). Claramente
existem (p 1)(q 1) pontos de Z Z no interior do ret
angulo T . Pelo exemplo anterior,
nao pode existir pontos de Z Z na diagonal OB. Por simetria, existem (p1)(q1)
pontos
 2
ip
no interior do ret
angulo OAB. Dado 1 i (q 1) existem exatamente
pontos da
q
q1  
X
(p 1)(q 1)
ip
=
.
forma (i, j) Z Z no interior do tri
angulo OAB. Assim
q
2
i=1

Conjugados
Suponha que seja um irracional e que estamos interessados em calcular o resto de n
mod m. Nesse caso, tentaremos encontrar tal que 0 < < 1 , + e Z. Para
entendermos o prop
osito disso, considere a equacao: x2 ax b = 0 onde a = + e
b = . Como e s
ao razes:
2 = a + b n+1 = an + bn1
2 = a + b n+1 = a n + b n1

Se Kn = n + n , temos Kn+1 = aKn + bKn1 . Como a e b s


ao inteiros e K1 = + Z,
K2 = ( + )2 2 Z, segue que Kn Z para todo natural n. Alem disso,
n

Kn Z

{ } + + { } + n Z

Consequentemente {n }+{ n } Z. Como 0 < {n }+{ n } < 2, devemos obrigatoriamente


ter {n } + { n } = 1. Usando que 0 < < 1, tambem podemos concluir que n = 0
e por conseguinte Kn = n + 1. Conhecendo-se a recurs
ao de Kn , podemos facilmente
determinar o perodo dos restos dos termos da sequencia na divisao por m. Os proximos
exemplos servirao para ilustrar essa heurstica. Tambem podemos modificar um pouco a
ideia anterior para tratar do caso 1 < < 0.
Exemplo 3. Prove que, para todo natural n temos:
$
!n %
7 + 37
3|
.
2
Prova: Sejam =

7+ 37
2

e=

7 37
.
2

Se Kn = n + n , entao:

Kn+1 = 7Kn 3Kn1 .


2

POT 2012 - Teoria dos N


umeros - Nvel 2 - Aula 19 - Samuel Feitosa

Como K1 = 7 1 (mod 3) e K2 = 43 1 (mod 3). Temos que todos os termos da


sequencia Kn s
ao inteiros e que todo deixam resto 1 na divisao por 3 pois Kn+1 7Kn Kn
(mod 3). Portanto,
n + 1 = Kn 1( mod 3) n N.

Exemplo 4. Encontre a maior potencia de 2 que divide (3 + 11))2n+1 .

Sejam = 3 + 11, = 3 11 e Kn = n + n . Entao, Kn+1 = 6Kn + 2Kn1 .


1 Lema:

2n+1 | K2n e 2n+1 k K2n+1

Provaremos o lema por inducao. Suponha que 2k+1 | K2k e que 2k+1 k K2k+1 , ou seja,
K2k = 2k+1 a e K2k+1 = 2k+1 b com b 1 (mod 2). Entao:
K2k+2 = 6 2k+1 b + 2 2k+1 a = 2k+2 (3b + a)

K2k+2 = 6 2k+2 (3b + a) + 2 2k+1 a = 2k+2 (18b + 7a)

e o resultado segue. Assim,


K2n+1 = 2n+1 + 2n+1 + { 2n+1 } + {2n+1 }
= 2n+1 + (1) + 1
= 2n+1

pois 1 < < 0 e consequentemente 2n+1 = 1. Da, em virtude do lema, a maior


potencia de 2 e 2n+1 .

Problemas Propostos
Problema 5. (Teste de Seleca
o do Brasil para
a Cone Sul)Prove que para todo inteiro
positivo k , a parte interia do n
umero (7 + 4 3)k e mpar.
Problema 6. (Olimpada Iraniana) Mostre que, k n k n = 1

1
onde k = 2 + 3.
n
k

Problema 7. (Hungria 2000) Se A = (1000 + 10002 + 1)1000 , determine o 2000-esimo


algarismo ap
os a vrgula de sua representaca
o decimal.
Problema 8. Prove que para todo inteiro m > 2 existe um irracional r que depende de m,
tal que rk 1( mod m).
Problema 9. Considere a sequecica de reais positivos a1 , a2 , ..., tal que a1 = 1 an = an+1 +
1
an+2 , para todo inteiro n > 0. Prove que o dgito das unidades de
n
ao pode ser 0,3,5
ai
ou 8 para todo i N.
1 n

a k b significa que an | b mas an+1 b


POT 2012 - Teoria dos N
umeros - Nvel 2 - Aula 19 - Samuel Feitosa
REFERENCIAS

Problema 10. (Seletiva do Brasil paraa IMO-2001) Encontre todos os naturais n tais que
1+ 5
n n2 e um inteiro onde =
.
2
Problema 11. (Revista Eureka)Seja a maior raiz da equaca
o x3 3x2 + 1 = 0. Prove
que 2004 e divisvel por 17.
Problema 12. (Taiwan 1998) Mostre que, para inteiros positivos m e n , mdc(m, n) =
m1
X  kn 
2
+ m + n mn.
m
k=0

Problema 13. (Balc


anica 2003) Seja ABCD um tabuleiro m n de quadrados unit
arios.
Assuma que mdc(m, n) = 1 e m, n s
ao mpares. Os pontos de interseca
o entre a diagonal
principal AC e os lados dos quadrados unit
arios s
ao A1 , A2 , ..., Ak , nesta ordem
(k 2) e

2
m + n2
.
A1 = A, Ak = C. Prove que A1 A2 A2 A3 + A3 A4 ... + (1)k Ak1 Ak =
mn
Problema 14. (Ge
orgia 1998) dado n > 5 , as retas x = n e y = n s
ao desenhadas no
plano cartesiano. considere os pontos com coordenadas inteiras no interior (ou bordo) do
quadrado formado por essas retas e pelos eixos. Quantos desses pontos tem a soma das
coordenadas multiplo de 5?
Problema 15. Um jogador solit
ario recebe ap
os cada jogada a ou b pontos (a e b s
ao
inteiros positivos com a < b) e estes se acumulam jogada ap
os jogada. Existem 35 valores
impossveis para a pontuaca
o acumulada e um desses valores e 58. Encontre a e b.

Refer
encias
[1] E. Carneiro, O. Campos and F. Paiva, Olimpadas Cearenses de Matem
atica 1981-2005
(Nveis J
unior e Senior), Ed. Realce, 2005.
[2] S. B. Feitosa, B. Holanda, Y. Lima and C. T. Magalhaes, Treinamento Cone Sul 2008.
Fortaleza, Ed. Realce, 2010.
[3] D. Fomin, A. Kirichenko, Leningrad Mathematical Olympiads 1987-1991, MathPro
Press, Westford, MA, 1994.
[4] D. Fomin, S. Genkin and I. Itenberg, Mathematical Circles, Mathematical Words, Vol.
7, American Mathematical Society, Boston, MA, 1966.
[5] I. Niven, H. S. Zuckerman, and H. L. Montgomery, An Introduction to the Theory of
Numbers.

Polos Olmpicos de Treinamento


Aula

Curso de Teoria dos Nmeros - Nvel 2

16

Prof. Samuel Feitosa

Ordem
Definic
ao 1. O menor inteiro positivo k para o qual ak 1 (mod m), onde mdc(a, m) = 1,
e chamado ordem de a m
odulo me ser
a denotado por ordm a.
Teorema 2. Se a e um inteiro relativamente primo com m, ent
ao an 1 (mod m) se, e
n
n
somente se, ordm a|n. Ademais, a0 a1 (mod m) se , e somente se, n0 n1 (mod ordm a)
Demonstraca
o. Sejam b = ordm a e n = qb + r com 0 r < b. Como ab 1 (mod m),
an 1 (mod m) aqb+r 1
r

a 1

(mod m)

(mod m)

Como 0 r < b, devemos ter r = 0. Usando que mdc(a, m) = 1 e supondo que n0 > n1 ,
an0 an1

(mod m) an0 n1 1

(mod m)

n0 n1 0 (mod b)
Teorema 3. Se mdc(a, m) = 1, ordm a|(m)
Demonstraca
o. Pelo teorema de Euler, a(m) 1 (mod m). O resultado segue do teorema
anterior.
Problema 4. (Putnam 1972) Prove que n
ao existe inteiro positivo n > 1 tal que n|2n 1.
Suponha, por absurdo, que existe um inteiro positivo n > 1 com essa propriedade e que k
e o menor dentre eles. Se d = ordk 2, entao d | k. Como 2d 1 (mod k), temos 2d 1
(mod d). Em virtude da minimalidade de k, temos d = 1 ou d = k. No primeiro caso,
teramos k = 1 produzindo uma contradicao. No segundo caso, em decorrencia do teorema
anterior, k | (k). Entretanto, se k > 1, (k) k 1 e obtemos assim um absurdo.
Problema 5. (Leningrado 1990) Prove que para todos os inteiros a > 1 e n , n|(an 1).
Se k = ordan 1 a, como an 1 (mod an 1), temos k | n e consequentemente k n. Nao
podemos ter k < n porque an 1 | ak 1 an 1 ak 1. Assim, k = n e usando o
teorema anterior podemos concluir que k | (an 1).
Problema 6. Mostre que
a) ord3n 2 = 2 3n1
b) Se 2m 1 (mod 3n ), ent
ao 3n1 | m.

Provaremos por induc


ao que 23 + 1 = 3k+1 mk com 3 mk . Suponha que a afirmacao vale
para k. Provemos para k + 1:
k+1

23

= (3k+1 mk 1)3
= 33k+3 m3k 32k+3 m2k + 3k+2 mk 1
= 3k+2 (32k+1 m3k 3k+1 + mk ) 1
= 3k+2 mk+1 1

Claramente 3 mk+1 . Voltemos ao problema. Seja b = ord3n 2, entao b | (3n ) = 2 3n1 .


n1
1 (mod 3n ) e 3j | 3n1
Temos duas possibilidades: ou b = 2 3j ou b = 3j . Como 23
j
j
se j n 1, devemos ter b = 2 3j . Assim, (23 1)(23 + 1) 1 (mod 3)n . Usando que
j
j
23 1 1 (mod 3), temos 23 1 (mod 3n ). Novamente pelo lema provado no incio,
3j 3n1 e assim b = 2 3n1 . Para o item b), de 2m 1 (mod 3n ), podemos concluir
que 22m 1 (mod 3)n . Da, 2 3n1 | 2m e o resultado segue.
Problema 7. (Bulg
aria 1997) Encontre todos os n
umeros inteiros m, n 2 tais que
n

1 + m3 + m23
n
e um inteiro

Claramente n e mpar, mdc(m, n) = 1 e n > 2. Se n = 3, como mdc(m, n) = 1 devemos


n
n
ter que m 1 (mod 3) pois caso contr
ario 1 + m3 + m23 1 1 + 1 1 (mod 3).
facil ver que todo par (m, n) = (3k + 1, 3) e solucao. Suponha agora n > 3 e seja
E
n+1
1
m3
n
n
n
23
3
3
segue
=
k = ordn m. Se n > 3 m 6 1 (mod n). Como 1 + m + m
m3n 1
n+1
1 k | 3n+1 . Logo, k = 3n+1 . Pelo teorema de Euler, m(n) 1 (mod n)
que n | m3
entao k (n) e 3n+1 (n) n 1, uma contradicao.
Problema 8. Prove que que se p e primo, ent
ao pp 1 tem um fator primo congruente a
1 m
odulo p
pp 1
. Como q | pp 1 segue que ordq p | p. Se ordq p = 1
p1
entao q | pp 1 e 0 pp1 + pp2 + . . . p + 1 1 + 1 + . . . + 1 + 1 p (mod q). Mas isso e
um absurdo pois p 6= q. Logo ordq p = p e obtemos p | (q) = q 1. Da, todos os divisores
pp 1
s
ao congruentes a 1 m
odulo p.
primos de
p1
Seja q um primo que divide

Problemas Propostos
Problema 9. Se orda m = h, ordm b = k e mdc(h, k) = 1 mostre que ordm ab = hk.

Problema 10. Prove que se a, b s


ao n
umeros naturais tais que a > b , n > 1, ent
ao cada
divisor primo do n
umero an bn e ou da forma nk + 1, onde k e um inteiro, ou um divisor
de um n
umero an1 bn1 , onde n1 |n e n1 < n.
Problema 11. Prove que se a, b s
ao n
umeros naturais tais que a > b , n > 1, ent
ao cada
divisor primo do n
umero an + bn e ou da forma 2nk + 1, onde k e um inteiro, ou um
divisor de um n
umero an1 + bn1 , onde n1 e o quociente obtido por dividir o n
umero n por
um n
umero mpar mairo que 1.
Problema 12. Seja p um primo que n
ao divide 10, e seja n um inteiro, 0 < n < p. Seja d
a ordem de 10 m
odulo p.
1. Mostre que o comprimento do perodo da representaca
o decimal de n/p e d.
2. Prove que que se d e par, ent
ao o perodo da representaca
o decimal de n/p pode ser
d/2
ao
dividido em duas partes cuja soma e 10 1. Por exmeplo, 1/7 = 0, 142857, ent
d = 6, e 142 + 857 = 999 = 103 1.
3. Se ordm a = h ordm ak =

h
mdc(h, k)

Problema 13. Se p e um primo maior que 3, ent


ao qualquer divisor maior que 1 do n
umero
2p + 1
e da forma 2kp + 1, onde k e um n
umero natural.
3
Teorema 14. Se p e um primo maior que 2, ent
ao qualquer n
umero natural que divida o
n
umero 2p 1 e da forma 2kp + 1, onde k e um inteiro.
Problema 15. (Bulg
aria 1995) Encontre todos os primos p e q tais que o n
umero 2p + 2q
seja divisvel por pq.
Problema 16. Mostre que se k > 1 ent
ao 2k1 6 1 (mod k)
n

Problema 17. Mostre que se 3 d 2n1 , ent


ao d (a2 +1) para qualquer inteiro positivo
a.
Problema 18. (Eureka) Prove que se p e um primo da forma 4k + 3, ent
ao 2p + 1 tambem
e primo se e somente se 2p + 1 divide 2p 1.
n

ao da
Problema 19. Prove que todos os divisores dos n
umeros de Fermat 22 + 1, n > 1, s
n+2
forma 2
k + 1.
Problema 20. (IMO 1990) Encontre todos os inteiros positivos n > 1 tais que
2n + 1
n2
e um inteiro.
Problema 21. (Teste Cone Sul 2002) Encontre o perodo na representaca
o decimal de
1
.
32002

Problema 22. (Teste de Seleca


o do Ir
a para a IMO) Seja a um natural fixo. Mostre que o
n
conjunto dos divisores primos de 22 + a, para n N, e infinito.
Problema 23. (Col
ombia 2009) Encontre todas as triplas de inteiros positivos (a, b, n) que
satisfazem a equaca
o:
ab = 1 + b + . . . + b n .
Problema 24. (IMO 2003) Seja p um n
umero primo. Demonstre que existe um n
umero
primo q tal que, para todo inteiro n, o n
umero np p n
ao e divisvel por q.

Polos Olmpicos de Treinamento


Curso de Teoria dos Nmeros - Nvel 2

Aula

17

Prof. Samuel Feitosa

Aula de Revis
ao e Aprofundamento

Exemplo 1. O mnimo multiplo comum dos inteiros a, b, c, d e d e igual a


` a + b + c + d.
Prove que abcd e divisvel por 3 ou por 5.
Solu
c
ao: Suponha inicialmente que mdc(a, b, c, d) = 1 e seja L = a + b + c + d. Como L e
facil ver
o mnimo m
ultiplo comum, existem x, y, z, w tais que aw = bx = cy = dz = L. E
que L tambem e o mnimo m
ultiplo comum de x, y, z, w e que
1
1 1 1
+ + +
x y z w

a
b
c
d
+ + +
L L L L
= 1.

Suponha sem perda de generalidade que w x y z. Da equacao anterior, o maior


valor de w e 4 e ocorrendo igualdade deveramos ter a = b = c = d = 1 que n
ao satisfaz
o enunciado. Para w = 3, o leitor poder
a facilmente verificar que a = b = 4, c = 3
e d = 1 e a u
nica soluc
ao. Para w = 2, temos as seguintes solucoes: (a, b, c, d) =
(5, 2, 2, 1), (10, 5, 4, 1), (6, 4, 1, 1) , (9, 6, 2, 1), (12, 8, 3, 1), (21, 14, 6, 1). Em todos os casos, L
e divisvel por 3 ou 5.
Exemplo 2. Seja A = {a1 < a2 < a3 < } uma seq
uencia crescente de inteiros positivos
em que o n
umero de fatores primos de cada termo, contando fatores repetidos, nunca e
maior que 2007. Prove que e sempre possvel extrair do conjunto A um subconjunto infinito
B = {b1 < b2 < b3 < }
tal que o m
aximo divisor comum entre bi e bj e sempre o mesmo para quaisquer naturais
i 6= j.
O n
umero de primos usados como fatores dos ai n
ao pode ser finito, pois se essa quantidade
e n, teramos apenas n2007 possveis elementos ai . Podem ocorrer duas situacoes:
a) Nenhum primo divide infinitos ai . Seja b1 = a1 . Como cada fator primo de a1
aparece um n
umero finito de vezes como fator dos ai , existira um termo al > a1 tal que a1
e al s
ao primos entre si. Seja b2 = al . Esse argumento pode ser repetido para gerar um
subconjunto infinito B = {b1 < b2 < b3 < } de modo que mdc(bi , bj ) = 1, quaisquer que

sejam os naturais i, j.
b) Existe um primo p que divide infinitos ai . Como o expoente de p1 = p em cada
ai que e m
ultiplo de p1 e um elemento do conjunto {1, . . . , 2007}, pelo menos um deles,
digamos r1 > 0, dever
a ocorrer infinitas vezes. Seja A1 o conjunto de todos os termos
ai para os quais o expoente de p1 em ai e r1 . Se nenhum primo p2 6= p1 divide infinitos
elementos de A1 , o caso anterior mostra que A1 possui um subconjunto tal que o m
aximo
divisor comum de quaisquer dois elementos e p1 r1 . Sen
ao, seja p2 6= p1 um primo que divide
infinitos elementos de A1 . O expoente de p2 em cada elemento de A1 que e m
ultiplo de p2
pertence ao conjunto {1, . . . , 2007 r1 }. Sejam r2 > 0 um expoente que ocorre infinitas
vezes e A2 o conjunto dos elementos de A1 para os quais p1 e p2 tem expoentes r1 e r2 ,
respectivamente. Esse processo deve terminar em i passos, i 2007, e nessa situacao Ai
e um subconjunto de A para o qual todo elemento e um m
ultiplo de P = pr11 pr22 pri i ,
digamos
Ai = {P c1 < P c2 < P c3 < },
onde C={c1 < c2 < c3 < } e um conjunto em que cada termo e o produto de n
ao mais
que 2007 (r1 + r2 + + ri ) fatores primos, nenhum dos quais ocorrendo infinitas vezes.
Pelo caso a), C possui um subconjunto B1 tal que quaisquer dois elementos s
ao primos
entre si. O conjunto
B = P B1 = {P x | x B1 }
satisfaz as condic
oes do problema, pois o m
aximo divisor comum entre quaisquer dois de
seus elementos e igual a P .
Exemplo 3. (Seletiva Rioplatense 2001) Encontre todos os pares (m, n) de n
umeros naturais
com m < n tais que m2 + 1 e um m
ultiplo de n e n2 + 1 e um m
ultiplo de m.
Afirmamos que todas as soluc
oes s
ao da forma (F2k1 , F2k+1 ), k 0 (Fn e o n-esimo
facil ver que F2k1 F2k+3 = F 2
termo da sequencia de Fibonacci). E
2k+1 + 1 e portanto
os pares anteriores s
ao soluc
oes. Seja P o conjunto das solucoes que n
ao s
ao da forma
(F2k1 , F2k+1 ). O conjunto P contem um par (a, b) tal que a + b e mnimo. Suponhamos
a < b(se a = b (a, b) = (1, 1) = (F1 , F1 ) 6 P ). Como b | a2 + 1, a2 + 1 = bb
facil ver que a | b 2 + 1 e b | a2 + 1. Logo (b , a) e uma solucao com
e b < a. E
b + a < a + b Entretanto, (b , a) 6 P e da (b , a) = (F2k1 , F2k+1 ). COnsequentemente,
2
b = F2k+3 (a, b) = (F2k+1 , F2k+3 ) 6 P . Logo P deve
F2k1 b = b b = a2 + 1 = F2k+1
ser vazio.
Exemplo 4. (URSS 1988) A sequencia de inteiros an e dada por a0 = 0, an = P (an1 ),
onde P (x) e um polin
omio cujos coeficientes s
ao inteiros positivos. Mostre que para quaisquer inteiros positivos m, k com m
aximo divisor comum d, o m
aximo divisor comum de am
e ak e ad .
Quando temos um polin
omio com coeficientes inteiros e sempre bom lembrar que a b |
P (a) P (b). Essa ser
a nossa principal ferramenta nesta solucao.

1. am | amr . Provaremos por inducao. Se am(r1) 0 (mod am ) am(r1)+1 P (0)


(mod am ) am(r1)+2 P (P (0)) (mod am ) amr = am(r1)+m P (P (. . . (P (0)) =
|
{z
}
m vezes

am 0 (mod am ).
2. Se l | at e l | af l | atf (Supondo t > f ). (Deixaremos a prova dessa afirmacao
para o leitor).
Pelo teorema de Bezout, existem inteiros positivos x, y tais que mx ky = d. Seja n =
mdc(am , ak ). Como n | am | amx e n | ak | aky , pelo item 2, n | amxky = ad . Mas ad | am
e ad | ak , ent
ao ad | n. Portanto ad = n.
Exemplo 5. Prove que existem infinitos n
umeros compostos n para os quais
n | 3n1 2n1 .
t

Lema: 2t | 32 1 t N
Vamos provar o lema por induc
ao. Para t = 1 e trivial. Suponha que a afirmacao
seja valida para t = r, provemos que tambem e valida para t = r + 1. Fatorando
t
t
t
t
t+1
t+1
ultiplo de 2t e o
= (22 33 )(22 + 33 ), como o primeiro parentesis e m
33
22
t+1
segundo e m
ultiplo de 2, o produto deles e m
ultiplo de 2 .
t

Seja n = 32 22 com t > 1. Como n 1 (32 1) 22 0 (mod 2t ), pelo lema,


t
t
obtemos n = 32 22 | 3n1 2n1 . Aqui estamos usando o fato que xk y k | xmk y mk .
Exemplo 6. Consideramos todas as sequencias (xn )n1 de inteiros positivos satisfazendo
xn+2 = mdc(xn , xn+1 ) + 2008, n 1.
Alguma dessas seq
uencias contem exatamente 102008 n
umeros distintos?
A ideia e construir a sequencia de tr
as para frente. Mostraremos que para qualquer inteiro
positivo k > 1, existe uma tal sequencia contendo exatamente k n
umeros distintos. Basta
encontrarmos uma sequencia que satisfaca:
xn+2 = mdc(xn , xn+1 ) + 2, n 1. (1)
pois a multiplicac
ao de todos os termos por 1004 produz a sequencia do problema. De(an1 2)(an2 2)
para n 5.
finamos a1 = 4, a2 = 6, a3 = 8, a4 = 2(a3 2) e an =
2
facil ver por induc
E
ao que todos os ai ser
ao pares e que a sequencia e crescente. Consequentemente, todos os ai com i 1 s
ao inteiros e distintos. Alem disso, definamos os
termos com ndices n
ao positivos por a3k = 4, a3k1 = 6, a3k2 = 4 para k 0.
. . . a6 , a5 , a4 , a3 = 8, a2 = 6, a1 = 4, a0 = 4, a1 = 6, a2 = 4, a3 = 4, a4 = 6, . . .
imediato
Afirmamos que a sequencia anterior satisfaz mdc(an+2 , an+1 )+2 = an n Z. E
verificar isso para n 1 Para n = 2, mdc(a4 , a3 ) + 2 = mdc(12, 8) + 2 = 6 = a2 . Para
n > 2,
mdc(an+2 , an+1 )+2 = mdc(

an 2
(an+1 2)(an 2)
, an+1 )+2 = mdc((an+1 )
an +2, an+1 )+2
2
2
3

Portanto,
mdc(an+2 , an+1 ) + 2 = mdc(an 2, (an 2)

an1 2
) + 2 = an 2 + 2 = an .
2

Para encontrarmos uma sequencia satisfazendo (1) com exatamente k > 1 termos distintos
basta escolhermos x1 = ak e x2 = ak1 e definirmos o resto da sequencia usando a relacao
de recorrencia xn+2 = mdc(xn , xn+1 ) + 2

Problemas Propostos

Problema 7. Resolva em inteiros a equaca


o:
xy xz yz
+
+
=3
z
y
x
Exemplo 8. Alguns inteiros positivos est
ao escritos no quadro. Podemos apagar quaisquer
dois inteiros distintos e substitu-los pelo m
aximo divisor comum e o mnimo divisor comum
dos dois n
umeros. Prove que eventualmente a operaca
o de alterar os n
umeros n
ao ser
a mais
executada.
Exemplo 9. Mostre que se k > 1 ent
ao 2k1 6 1 (mod k)
Exemplo 10. (Est
onia 2005) Sejam a, b inteiros positivos primos entre si tais que (a +
b)/(a b) e um inteiro positivo. Prove que ao menos um dentre os n
umeros ab + 1 e 4ab + 1
e um quadrado perfeito.
Exemplo 11. (Ibero 1999) Seja n um inteiro maior que 10 tal que cada um de seus dgitos
pertence ao conjunto S = {1, 3, 7, 9}. Prove que n tem algum divisor primo maior ou igual
a 11.

You might also like